SlideShare a Scribd company logo
TO GET ALL CHAPTERS EMAIL ME AT>>>>> donc8246@gmail.com
Test Bank for Huether and McCance's Understanding
Pathophysiology, Canadian Edition,
2nd Edition by Kelly Power-Kean,
Chapter 1 - 42
TO GET ALL CHAPTERS EMAIL ME AT>>>>>
donc8246@gmail.com
Table of Contents
PART ONE: BASIC CONCEPTS OF PATHOPHYSIOLOGY
Unit 1: The Cell
1. Cellular Biology
2. Genes and Genetic Diseases
3. Epigenetics and Disease
4. Altered Cellular and Tissue Biology
5. Fluids and Electrolytes, Acids and Bases
Unit 2: Mechanisms of Self-Defense
6. Innate Immunity: Inflammation and Wound Healing
7. Adaptive Immunity
8. Infection and Defects in Mechanisms of Defense
9. Stress and Disease
Unit 3: Cellular Proliferation: Cancer
10. Biology of Cancer
11. Cancer Epidemiology
12. Cancer in Children and Adolescents
PART TWO: BODY SYSTEMS AND DISEASES
Unit 4: The Neurologic System
13. Structure and Function of the Neurologic System
14. Pain, Temperature, Sleep, and Sensory Function
15. Alterations in Cognitive Systems, Cerebral Hemodynamics and Motor Function
16. Disorders of the Central and Peripheral Nervous Systems and Neuromuscular Junction
17. Alterations of Neurologic Function in Children
Unit 5: The Endocrine System
18. Mechanisms of Hormonal Regulation
19. Alterations of Hormonal Regulation
Unit 6: The Hematologic System
20. Structure and Function of the Hematologic System
21. Alterations in Hematologic Function
22. Alterations of Hematologic Function in Children
Unit 7: The Cardiovascular and Lymphatic Systems
23. Structure and Function of the Cardiovascular and Lymphatic Systems
24. Alterations of Cardiovascular Function
25. Alterations of Cardiovascular Function in Children
Unit 8: The Pulmonary System
26. Structure and Function of the Pulmonary System
27. Alterations of Pulmonary Function
28. Alterations of Pulmonary Function in Children
Unit 9: The Renal and Urologic Systems
29. Structure and Function of the Renal and Urologic Systems
30. Alterations of Renal and Urinary Tract Function
31. Alterations of Renal and Urinary Tract Function in Children
TO GET ALL CHAPTERS EMAIL ME AT>>>>>
donc8246@gmail.com
Unit 10: The Reproductive Systems
32. Structure and Function of the Reproductive Systems
33. Alterations of the Female Reproductive System
34. Alterations of the Male Reproductive System
Unit 11: The Digestive System
35. Structure and Function of the Digestive System
36. Alterations of Digestive Function
37. Alterations in Digestive Function in Children
Unit 12: The Musculoskeletal and Integumentary Systems
38. Structure and Function of the Musculoskeletal System
39. Alterations of Musculoskeletal Function
40. Alterations of Musculoskeletal Function in Children
41. Structure, Function, and Disorders of the Integument
42. Alterations of the Integument in Children
NURSINGTB.COM
TO GET ALL CHAPTERS EMAIL ME AT>>>>>
donc8246@gmail.com
Chapter 01: Cellular Biology
MULTIPLE CHOICE
1. A student is observing a cell under the microscope. It is observed to have supercoiled DNA
with histones. Which of the following would also be observed by the student?
a. A single circular chromosome
b. A nucleus
c. Free-floating nuclear material
d. No organelles
ANS: B
The cell described is a eukaryotic cell, so it has histones and a supercoiled DNA within its
nucleus; thus, the nucleus should be observed. A single circular chromosome called a
prokaryote contains free-floating nuclear material but has no organelles.
REF: p. 2
2. A nurse is instructing the staff about cellular functions. Which cellular function is the nurse
describing when an isolated cell absorbs oxygen and uses it to transform nutrients to energy?
a. Metabolic absorption
b. Communication
c. Secretion
d. Respiration
ANS: D NURSINGTB.COM
The cell’s ability to absorb oxygen is referred to as respiration while its communication ability
involves maintenance of a steady dynamic state, metabolic absorption provides nutrition, and
secretion allows for the synthesizing of new substances.
REF: p. 2
3. A eukaryotic cell is undergoing DNA replication. In which region of the cell would most of
the genetic information be contained?
a. Mitochondria
b. Ribosome
c. Nucleolus
d. Nucleus Cytoplasm
ANS: C
The region of the cell that contains genetic material, including a large amount of ribonucleic
acid, most of the DNA, and DNA-binding proteins, is the nucleolus, which is located within
the cell’s nucleus. Mitochondria is associated with cellular respiration, while ribosomes are
involved with protein manufacturing. Cytoplasm is a fluid filling that is a component of the
cell.
REF: p. 2
NURSINGTB.COM
TO GET ALL CHAPTERS EMAIL ME AT>>>>>
donc8246@gmail.com
U S N
4. Which of the following can remove proteins attached to the cell’s bilayer by dissolving the
layer itself?
a. Peripheral membrane proteins
b. Integral membrane proteins
c. Glycoproteins
d. Cell adhesion molecules
ANS: B
Proteins directly attached to the membrane bilayer can be removed by the action of integral
membrane proteins that dissolve the bilayer. Peripheral membrane proteins reside at the surface
while cell adhesion molecules are on the outside of the membrane. Glycoprotein marks cells and
does not float.
REF: p. 7
5. Which of the following can bind to plasma membrane receptors?
a. Oxygen
b. Ribosomes
c. Amphipathic lipids
d. Ligands
ANS: D
Ligands are the only specific molecules that can bind with receptors on the cell membrane.
REF: p. 9
6. A nurse is reviewing a report from a patient with metastatic cancer. What alternation in the
extracellular matrix would suN
ppoR
rt t h
Ie dG
iagn
B
o.
siC
s ofM
metastatic cancer?
a. Decreased fibronectin
b. Increased collagen
c. Decreased elastin
d. Increased glycoproteins
ANS: A
Only a reduced amount of fibronectin is found in some types of cancerous cells, allowing
them to travel or metastasize.
REF: p. 10
7. Which form of cell communication is used to relate to other cells in direct physical contact?
a. Cell junction
b. Gap junction
c. Desmosome
d. Tight junction
ANS: A
Cell junctions hold cells together and permit molecules to pass from cell to cell.
Gap junctions allow for cellular communication between cells. Neither desmosomes nor tight
junctions are associated with cellular communication.
REF: p. 11
NURSINGTB.COM
TO GET ALL CHAPTERS EMAIL ME AT>>>>>
donc8246@gmail.com
8. Pancreatic beta cells secrete insulin, which inhibits secretion of glucagon from neighboring
alpha cells. This action is an example of which of the following signaling types?
a. Paracrine
b. Autocrine
c. Neurohormonal
d. Hormonal
ANS: A
Paracrine signaling involves the release of local chemical mediators that are quickly taken up,
destroyed, or immobilized, as in the case of insulin and the inhibition of the secretion of
glucagon. None of the other options involve signaling that is associated with a local chemical
mediator like insulin.
REF: p. 12
9. In cellular metabolism, each enzyme has a high affinity for a:
a. solute.
b. substrate.
c. receptor.
d. ribosome.
ANS: B
Each enzyme has a high affinity for a substrate, a specific substance converted to a product of
the reaction. Cellular metabolism is not dependent on an attraction between an enzyme and any
of the remaining options.
REF: p. 16
NURSINGTB.COM
10. An athlete runs a marathon, after which his muscles feel fatigued and unable to contract. The
athlete asks the nurse why this happened. The nurse’s response is based on the knowledge that
the problem is result of a deficiency of:
a. GTP
b. AMP
c. ATP
d. GMP
ANS: C
When ATP is deficient, impaired muscle contraction results. None of the other options are
involved in muscle contraction.
REF: p. 16
11. Which phase of catabolism produces the most ATP?
a. Digestion
b. Glycolysis
c. Oxidation
d. Citric acid cycle
ANS: D
While some ATP is produced during the oxidation and glycolysis phases, most of the ATP is
generated during the citric acid cycle. Digestion does not produce any ATP.
NURSINGTB.COM
TO GET ALL CHAPTERS EMAIL ME AT>>>>>
donc8246@gmail.com
REF: p. 16
12. A nurse is teaching the staff about the phases of cellular catabolism. Which phases should the
nurse include?
a. Digestion, glycolysis, oxidation, and the citric acid cycle
b. Diffusion, osmosis, and mediated transport
c. S phase, G phase, and M phase
d. Metabolic absorption, respiration, and excretion
ANS: A
Only digestion, glycolysis, oxidation, and the citric acid cycle are the phases of cellular
catabolism.
REF: p. 16
13. A runner has depleted all the oxygen available for muscle energy. Which of the following will
facilitate his continued muscle performance?
a. Electron-transport chain
b. Aerobic glycolysis
c. Anaerobic glycolysis
d. Oxidative phosphorylation
ANS: C
When no oxygen is available, anaerobic glycolysis occurs. The electron-transport chain is part of
the citric acid cycle. Aerobic glycolysis involves the presence of oxygen. Oxidative
phosphorylation is the mechanism by which the energy produced from carbohydrates, fats, and
proteins is transferred to ATP. It is not part of muscle performance.
REF: p. 16 NURSINGTB.COM
14. A faculty member asks a student to identify the appropriate term for the movement of a solute
from an area of greater to lesser concentration. Which answer indicates the nursing student
understood the teaching?
a. Osmosis
b. Diffusion
c. Hydrostatic pressure
d. Active transport
ANS: B
Diffusion is the movement of a solute molecule from an area of greater solute concentration to
an area of lesser solute concentration through a permeable membrane. Osmosis is the
movement of water across a semipermeable membrane from a region of higher water
concentration to one of lower concentration. Hydrostatic pressure is the force of fluid against a
cell membrane. In active transport, molecules move up a concentration gradient.
REF: p. 19
15. Which description accurately describes electrolytes?
a. Small lipid-soluble molecules
b. Large protein molecules
c. Micronutrients used to produce ATP
NURSINGTB.COM
TO GET ALL CHAPTERS EMAIL ME AT>>>>>
donc8246@gmail.com
d. Electrically charged molecules
ANS: D
Electrolytes are electrically charged molecules. They are not lipid soluble, they are not made up
of protein, and they do not play a role in ATP production.
REF: p. 18
16. A nurse is reading a chart and sees the term oncotic pressure. The nurse recalls that oncotic
pressure (colloid osmotic pressure) is determined by:
a. the concentration of sodium.
b. plasma proteins.
c. hydrostatic pressure.
d. the availability of membrane transporter proteins.
ANS: B
Oncotic pressure is determined by the effect of colloids or plasma proteins. The concentration
of sodium plays a role in tonicity. Hydrostatic pressure is the force within a vessel. Membrane
transporter proteins are involved in active transport within a concentration gradient.
REF: p. 20
17. A patient has a body fluid of 300 mOsm/kg. This lab result is measuring:
a. osmolality.
b. osmolarity.
c. osmotic pressure.
d. oncotic pressure.
ANS: A NURSINGTB.COM
Osmolality measures the number of milliosmoles per kilogram of water, or the concentration of
molecules per weight of water, while osmolarity measures the number of milliosmoles per liter of
solution, or the concentration of molecules per volume of solution. Osmotic pressure is the
amount of hydrostatic pressure required to oppose the osmotic movement of water.
Oncotic pressure is from plasma proteins, not body fluids.
REF: p. 19
18. A nurse is discussing the movement of fluid across the arterial end of capillary membranes
into the interstitial fluid surrounding the capillary. Which process of fluid movement is the
nurse describing?
a. Hydrostatic pressure
b. Osmosis
c. Diffusion
d. Active transport
ANS: A
Blood reaching the capillary bed has a hydrostatic pressure of 25–30 mm Hg, which is sufficient
force to push water across the thin capillary membranes into the interstitial space. Osmosis
involves the movement of fluid from an area of higher concentration to an area of lower
concentration. It does not involve pressure or force. Diffusion is the passive movement of a
solute from an area of higher solute concentration to an area of lower solute concentration.
Active transport involves movement up a concentration gradient.
NURSINGTB.COM
TO GET ALL CHAPTERS EMAIL ME AT>>>>>
donc8246@gmail.com
REF: p. 19
19. How are potassium and sodium transported across plasma membranes?
a. By passive electrolyte channels
b. By coupled channels
c. By adenosine triphosphate enzyme (ATPase)
d. By diffusion
ANS: C
The transporter protein ATPase is directly related to sodium and potassium transport via active
transport. Electrolyte movements require energy and do not move passively, nor are they
transported by diffusion. Enzymes, not electrolytes, are passed via coupled channels.
REF: p. 21
20. The ion transporter that moves Na+
and Ca2+
simultaneously in the same direction is an
example of which of the following types of transport?
a. Biport
b. Uniport
c. Antiport
d. Symport
ANS: D
When ions are transported in one direction, it is termed symport. There is no such term as
biport. Uniport refers to the movement of a single molecule. Antiport refers to the movement
of molecules in the opposite direction.
NURSINGTB.COM
REF: p. 19, Figure 1-22
21. During which process are bacteria engulfed for ingestion?
a. Endocytosis
b. Pinocytosis
c. Phagocytosis
d. Exocytosis
ANS: C
Phagocytosis (cell eating) involves the ingestion of large particles, such as bacteria, through the
formation of large vesicles. Endocytosis involves the formation of vesicles to facilitate
movement into the cell. Pinocytosis is a type of endocytosis in which fluids and solute
molecules are ingested through the formation of small vesicles. Exocytosis occurs when coated
pits invaginate and internalize ligand-receptor complexes in coated vesicles.
REF: p. 22
22. Some cancer drugs work during the cell cycle phase where nuclear and cytoplasmic divisions
occur. What is this cell cycle phase called?
a. G1
b. S
c. M
d. G2
NURSINGTB.COM
TO GET ALL CHAPTERS EMAIL ME AT>>>>>
donc8246@gmail.com
ANS: C
The M phase includes both nuclear and cytoplasmic divisions. The G1 phase includes the period
between the M phase and the start of DNA synthesis. The S phase includes synthesis of DNA in
the cell nucleus. The G2 phase includes RNA and protein synthesis.
REF: pp. 25-26
23. Which causes the rapid change in the resting membrane potential that initiates an action
potential?
a. Potassium gates open, and potassium rushes into the cell, changing the membrane
potential from negative to positive.
b. Sodium gates open, and sodium rushes into the cell, changing the membrane
potential from negative to positive.
c. Sodium gates close, allowing potassium into the cell to change the membrane
potential from positive to negative.
d. Potassium gates close, allowing sodium into the cell to change the membrane
potential from positive to negative.
ANS: B
When the threshold is reached, the cell will continue to depolarize with no further stimulation.
The sodium gates open, and sodium rushes into the cell, causing the membrane potential to
reduce to zero and then become positive (depolarization). Sodium is involved in creating the
action potential, not potassium. The sodium gate and channel must be open, not closed. The
action potential is not affected by a change in the potassium gate.
REF: pp. 24-25
24. A cell is isolated, and electroN
ph
U
yR
siS
olI
og
N
yG
st
T
uB
di.
esC
re
O
vM
eal that the resting membrane potential is
70 mV. The predominant intracellular ion is Na+
, and the predominant extracellular ion is K+
.
With voltage change, which of the following would result in an action potential?
a. K+
rushing into the cell
b. Na+
rushing into the cell
c. Na+
rushing out of the cell
d. K+
rushing out of the cell
ANS: A
With voltage change, potassium rushes into, not out of, the cell. Sodium movement is not
related to this process.
REF: pp. 24-25
25. A nurse teaching the staff about platelet-derived growth factor includes information that
platelet-derived growth factor (PDGF) stimulates the production of:
a. platelets.
b. epidermal cells.
c. connective tissue cells.
d. fibroblast cells.
ANS: C
Different types of cells require different growth factors; for example, PDGF stimulates the
production of connective tissue cells, but not platelets, epidermal cells, or fibroblast cells.
NURSINGTB.COM
TO GET ALL CHAPTERS EMAIL ME AT>>>>>
donc8246@gmail.com
REF: p. 27
26. The phase of the cell cycle during which the centromeres split and the sister chromatids are
pulled apart is referred to as:
a. anaphase.
b. telophase.
c. prophase.
d. metaphase.
ANS: A
Anaphase begins when the centromeres split and the sister chromatids are pulled apart. During
telophase, a new nuclear membrane is formed around each group of 46 chromosomes, the
spindle fibers disappear, and the chromosomes begin to uncoil. During prophase, the first
appearance of chromosomes occurs. Metaphase occurs when two centrioles located at opposite
poles of the cell pull the chromosomes to opposite sides of the cell.
REF: p. 26
27. What is the role of cytokines in cell reproduction?
a. Provide growth factor for tissue growth and development.
b. Block progress of cell reproduction through the cell cycle.
c. Restrain cell growth and development.
d. Provide nutrients for cell growth and development.
ANS: A
Cytokines play a major role in the regulation of tissue growth and development but do not
oR
meI
intrG
aceB
llu.
laC
r br
M
aking mechanisms that restrain cell
restrain it. Cytokines help ovN
erc U S N
growth and promote cell growth, but they do not provide nutrients.
REF: p. 26
28. A biopsy of the lung bronchi revealed ciliated epithelial cells that are capable of secretion and
absorption. These cells are called columnar epithelium.
a. simple
b. ciliated simple
c. stratified
d. pseudostratified ciliated
ANS: B
Ciliated simple columnar epithelium is found in the lungs. Simple columnar epithelium is found
from the stomach to the anus. Stratified columnar epithelium is found in the lining of the
epiglottis, part of the pharynx, the anus, and the male urethra. Pseudostratified ciliate columnar
epithelium is found in the lining of the large ducts of some glands (parotid, salivary), male
urethra, respiratory passages, and Eustachian tubes of the ears.
REF: p. 30, Table 1-6
29. A student is reviewing functions of the cell. The student would be correct in identifying a
chief function of the nerve cell as:
a. sensory interpretation.
b. conductivity.
NURSINGTB.COM
TO GET ALL CHAPTERS EMAIL ME AT>>>>>
donc8246@gmail.com
c. maintenance of homeostasis.
d. communication.
ANS: B
Conductivity, not sensory interpretation, homeostasis, or communication, is one of the eight
chief functions of nerve cells.
REF: p. 2
MULTIPLE RESPONSE
1. A nurse recalls that the basic types of tissues are: (select all that apply)
a. nerve.
b. epithelial.
c. mucosal.
d. connective.
e. skeletal.
f. muscle.
ANS: A, B, D, F
The basic tissue types include nerve, epithelial, connective, and muscle. Mucosal is a type of
epithelial cell, while skeletal is a type of connective tissue.
REF: p. 27
2. Characteristics of prokaryotes include which of the following? (select all that apply)
a. They contain no organelles.
b. Their nuclear material isN
no
U
tR
en
S
cI
asN
ed
G
b
T
yB
a.
nu
C
cO
leM
ar membrane.
c. They contain a distinct nucleus.
d. They contain histones.
e. They contain a cellular membrane.
ANS: A, B
The prokaryotes lack a cellular membrane that encases nuclear material, thus they have no
distinct nucleus; organelles and histones are also missing.
REF: p. 1
NURSINGTB.COM
TO GET ALL CHAPTERS EMAIL ME AT>>>>>
donc8246@gmail.com
Chapter 02: Genes and Genetic Diseases
MULTIPLE CHOICE
1. A nurse recalls the basic components of DNA are:
a. pentose sugars and four phosphate bases.
b. a phosphate molecule, deoxyribose, and four nitrogenous bases.
c. adenine, guanine, and purine.
d. codons, oxygen, and cytosine.
ANS: B
The three basic components of DNA are deoxyribose; a phosphate molecule; and four types of
nitrogenous, not phosphate, bases. DNA does not contain condone.
REF: p. 38
2. Which of the following mutations have the most significant effect on protein synthesis?
a. Base pair substitutions
b. Silent mutations
c. Intron mutations
d. Frameshift mutations
ANS: D
The frameshift mutation involves the insertion or deletion of one or more base pairs of the
DNA molecule. This greatly alters the amino acid sequence, which affects protein synthesis.
The base pair substitution is N
a tU
yp
R
eS
oI
f N
mG
utT
atB
io.
n C
inO
w
M
hich one base pair replaces another. Silent
mutations do not change amino acids or protein synthesis. Intron mutations are part of RNA
sequencing.
REF: p. 39
3. The base components of DNA are:
a. A, G, C, and U.
b. P, G, C, and T.
c. A, G, C, and T.
d. X, XX, XY, and YY.
ANS: C
The four base components of DNA are cytosine, thymine, adenine, and guanine, and are
commonly represented by their first letters (A, C, T, and G) and not components identified as P
or U. X, XX, XY, and YY are components of human chromosomes.
REF: p. 38
4. A DNA strand has a region with the sequence ATCGGAT. Which of the following would be a
complementary strand?
a. CGATACGT
b. TAGCCTAG
c. TUGCCTUG
NURSINGTB.COM
TO GET ALL CHAPTERS EMAIL ME AT>>>>>
donc8246@gmail.com
d. UAGCCUAG
ANS: B
The consistent pairing of adenine with thymine and of guanine with cytosine is known as
complementary base pairing; thus, A complements to T and C to G and vice versa throughout
the strand. A complements to T; thus, the first letter must be a T. U does not represent a
complement in the sequence.
REF: p. 39
5. A biologist is explaining how RNA directs the synthesis of protein. Which process is the
biologist describing?
a. Termination
b. Transcription
c. Translocation
d. Translation
ANS: D
In translation, RNA directs the synthesis of a polypeptide, interacting with transfer RNA
(tRNA), a cloverleaf-shaped strand of about 80 nucleotides. Termination does not involve
synthesis of protein. Transcription is the process by which DNA specifies a sequence of
messenger RNA (mRNA). Translocation is the interchange of genetic material between
nonhomologous chromosomes.
REF: p. 41
6. What is the result of homologous chromosomes failing to separate during meiosis?
a. Neurofibromatosis
b. Nondisjunction
c. Polyploidy
d. Conjoined twins
ANS: B
NURSINGTB.COM
Nondisjunction is an error in which homologous chromosomes or sister chromatids fail to
separate normally during meiosis or mitosis. Neurofibromatosis is not due to chromosome
failure during meiosis. Polyploidy occurs when a euploid cell has more than the diploid number
of chromosomes. Conjoined twins are not due to chromosome failure during meiosis.
REF: p. 45
7. A cell that does not contain a multiple of 23 chromosomes is called a cell.
a. diploid
b. euploid
c. polyploid
d. haploid
ANS: C
A polyploid cell is one in which a euploid cell has more than 23 pairs of chromosomes. A
diploid cell is when the somatic cell nucleus has 46 chromosomes in 23 pairs. A euploid cell is a
cell with multiples of the normal number of chromosomes. A haploid cell has only one
member of each chromosome pair, for a total of 23 chromosomes.
NURSINGTB.COM
TO GET ALL CHAPTERS EMAIL ME AT>>>>>
donc8246@gmail.com
U S N
REF: p. 42
8. A 20-year-old pregnant female gives birth to a stillborn child. Autopsy reveals that the fetus
has 92 chromosomes. What term may be on the autopsy report to describe this condition?
a. Biploidy
b. Triploidy
c. Tetraploidy
d. Aneuploidy
ANS: C
Tetraploidy is a condition in which euploid cells have 92 chromosomes. Biploidy is a euploid cell
with two times more chromosomes, or 46. Triploidy is a zygote that has three copies of each
chromosome, rather than the usual two. Aneuploidy is when an aneuploid cell does not contain
a multiple of 23 chromosomes.
REF: p. 42
9. The condition in which an extra portion of a chromosome is present in each cell is called:
a. reciprocal translocation.
b. partial trisomy.
c. inversion.
d. Down syndrome.
ANS: B
Partial trisomy is a condition in which only an extra portion of a chromosome is present in each
cell. A reciprocal translocation occurs when breaks take place in two different chromosomes and
the material is exchanged. An inversion occurs when two breaks take place
on a chromosome, followed b
N
y t h
Re rI
einsG
ertio
B
n
.
oC
f t hM
e missing fragment at its original site, but in
inverted order. Down syndrome is an aneuploidy of the twenty-first chromosome.
REF: p. 46
10. After a geneticist talks to a patient about being a chromosomal mosaic, the patient asks the
nurse what that means. How should the nurse respond? You may genetic disease(s).
a. only be a carrier of the
b. have a mild form of the
c. have two
d. be sterile as a result of the
ANS: B
A chromosomal mosaic means the body has two or more different cell lines, each of which has
a different karyotype; thus, the person has a mild form of the disease. Mosaics are not only
carriers; they have the disease; they have two different lines but not two different diseases; and
they are not necessarily sterile.
REF: p. 46
11. What is the most common cause of Down syndrome?
a. Paternal nondisjunction
b. Maternal translocations
c. Maternal nondisjunction
NURSINGTB.COM
TO GET ALL CHAPTERS EMAIL ME AT>>>>>
donc8246@gmail.com
d. Paternal translocations
ANS: C
The most common cause of Down syndrome is maternal, not paternal, nondisjunction.
Translocation is not a cause of this syndrome.
REF: p. 46, Table 2-1
12. A patient wants to know the risk factors for Down syndrome. What is the nurse’s best
response?
a. Fetal exposure to mutagens in the uterus.
b. Increased paternal age.
c. Family history of Down syndrome.
d. Pregnancy in women over age 35.
ANS: D
The primary risk for Down syndrome is pregnancy in women over 35. Down syndrome is a
trisomy and not due to fetal exposure or paternal age. Down syndrome is a chromosomal
abnormality and is not related to family history.
REF: pp. 46-47
13. A 13-year-old girl has a karyotype that reveals an absent homologous X chromosome with
only a single X chromosome present. What medical diagnosis will the nurse observe on the
chart?
a. Down syndrome
b. Cri du chat syndrome
c. Turner syndrome
d. Fragile X syndrome
ANS: C
NURSINGTB.COM
A condition with the presence of a single X chromosome and no homologous X or Y
chromosome, so the individual has a total of 45 chromosomes, is known as Turner syndrome.
Down syndrome is a change in one arm of a chromosome. Cri du chat syndrome is due to a
chromosome deletion. Fragile X syndrome is due to a break or a gap in a chromosome.
REF: p. 47
14. What genetic disorder is the result if an individual possesses an XXY chromosome
configuration?
a. Turner
b. Klinefelter
c. Down
d. Fragile X
ANS: B
Individuals with at least two X chromosomes and one Y chromosome in each cell (47 XXY
karyotype) have a disorder known as Klinefelter syndrome. A condition with the presence of a
single X chromosome and no homologous X or Y chromosome, so the individual has a total of
45 chromosomes, is known as Turner syndrome. Down syndrome is a trisomy. Fragile X
syndrome is due to a break or a gap in a chromosome, not an extra chromosome.
NURSINGTB.COM
TO GET ALL CHAPTERS EMAIL ME AT>>>>>
donc8246@gmail.com
REF: p. 47
15. A patient demonstrates severe mental retardation caused by a deletion of part of chromosome
5. What genetic disorder will the nurse see documented in the chart?
a. Prader-Willi syndrome
b. Down syndrome
c. Cri du chat syndrome
d. Trisomy X
ANS: C
Cri du chat syndrome means “cry of the cat” and describes the characteristic cry of the affected
child. Another symptom of the disorder is mental retardation. The disease is caused by a
deletion of part of the short arm of chromosome 5. Prader-Willi syndrome is characterized by
short stature, obesity, and hypogonadism. Down syndrome does cause mental retardation but is
due to chromosome 21, not chromosome 5. Trisomy X can result in mental retardation but is
due to an extra X chromosome.
REF: p. 48
16. An aide asks the nurse why people who have neurofibromatosis will show varying degrees of
the disease. Which genetic principle should the nurse explain to the aide?
a. Penetrance
b. Expressivity
c. Dominance
d. Recessiveness
ANS: B
Expressivity is the extent of v
N
ariR
aU
tion
I
SinN
G
phT
eB
no
.tC
ypeM
associated with a particular genotype. For
neurofibromatosis, a variety of manifestations occur among individuals. The penetrance of a
trait is the percentage of individuals with a specific genotype who also exhibit the expected
phenotype. Dominance refers to observable traits and risk of transmission. Recessiveness refers
to silent strains with reduced risk of occurrence.
REF: p. 51
17. Cystic fibrosis is caused by what gene abnormality?
a. X-linked dominant
b. X-linked recessive
c. Autosomal dominant
d. Autosomal recessive
ANS: D
Cystic fibrosis is an autosomal recessive disorder. It is not a result of X links or dominant
pathology.
REF: p. 52
18. A 15-year-old female is diagnosed with Prader-Willi syndrome. This condition is an example of:
a. genomic imprinting.
b. an autosomal recessive trait.
NURSINGTB.COM
TO GET ALL CHAPTERS EMAIL ME AT>>>>>
donc8246@gmail.com
c. an autosomal dominant trait.
d. a sex-linked trait.
ANS: A
Prader-Willi, an example of gene imprinting, is not associated with any autosomal sex-linked
abnormality.
REF: p. 52
19. A patient, age 9, is admitted to a pediatric unit with Duchenne muscular dystrophy. When
planning care the nurse recalls the patient inherited this condition through a trait that is:
a. X-linked dominant.
b. X-influenced.
c. X-limited.
d. X-linked recessive.
ANS: D
Duchenne muscular dystrophy is a relatively common X-linked recessive, not dominant,
disorder. While it is sex linked, it is not X-limited or X-influenced.
REF: p. 55
20. A child is diagnosed with cystic fibrosis. History reveals that the child’s parents are siblings.
Cystic fibrosis was most likely the result of:
a. X-inactivation.
b. genomic imprinting.
c. consanguinity.
d. obligate carriers.
ANS: C
NURSINGTB.COM
Consanguinity refers to the mating of two related individuals, and the offspring of such
matings are said to be inbred. Consanguineous matings produce a significant increase in
recessive disorders and are seen most often in pedigrees for rare recessive disorders.
X-inactivation occurs when one X chromosome in the somatic cells of females is permanently
inactivated. Genomic imprinting is related to methylation and other changes. Obligate carriers
are those who have an affected parent and affected children and, therefore, must themselves
carry the mutation.
REF: p. 54
21. A 12-year-old male is diagnosed with Klinefelter syndrome. His karyotype would reveal
which of the following?
a. XY
b. XX
c. XYY
d. XXY
ANS: D
A person with Klinefelter syndrome has an XXY karyotype. An XY is a normal male. An XX is
a normal female. An XYY is an aneuploid karyotype.
REF: p. 47
NURSINGTB.COM
TO GET ALL CHAPTERS EMAIL ME AT>>>>>
donc8246@gmail.com
22. To express a polygenic trait:
a. genes must interact with the environment.
b. several genes must act together.
c. multiple mutations must occur in the same family.
d. penetrance must occur.
ANS: B
Polygenic traits are those that result from several genes acting together. When environmental
factors influence the expression of the trait, the term multifactorial inheritance is used. When
multiple mutations occur in the same family, the mechanism most likely responsible is termed
germline mosaicism. Penetrance of a trait is the percentage of individuals with a specific
genotype who also exhibit the expected phenotype.
REF: pp. 57-58
23. What is the diagnosis of a 13-year-old female who has a karyotype that reveals an absent
homologous X chromosome with only a single X chromosome present? Her features include a
short stature, widely spaced nipples, and a reduced carrying angle at the elbow.
a. Down syndrome
b. Cri du chat syndrome
c. Turner syndrome
d. Klinefelter syndrome
ANS: C
Turner syndrome is characterized by short stature, female genitalia, webbed neck, shield-like
chest with underdeveloped breasts and widely spaced nipples, and imperfectly developed
ovaries. Down syndrome is cN
ha
U
ra
R
cS
teI
riz
N
eG
d T
by
Bd
.
is
C
tiO
nc
M
tive characteristics: low nasal bridge,
epicanthal folds, protruding tongue, and low-set ears. Cri du chat syndrome is characterized by
low birth weight, severe mental retardation, microcephaly (smaller than normal head size),
and heart defects. Klinefelter syndrome is characterized by small testes, some development of
the breasts, sparse body hair, and long limbs.
REF: p. 46, Table 2-1
24. The gradual increase in height among the human population over the past 100 years is an
example of:
a. a polygenic trait.
b. a multifactorial trait.
c. crossing over.
d. recombination.
ANS: B
The gradual increase in height is an example of multifactorial traits influenced by genes and also
by environment. Polygenic traits result from several genes acting together. Crossing over is an
abnormal chromosome structure. Recombination results from new arrangements of alleles.
REF: p. 58
25. When discussing DNA replication, which enzyme is most important?
NURSINGTB.COM
TO GET ALL CHAPTERS EMAIL ME AT>>>>>
donc8246@gmail.com
a. RNA polymerase
b. Transfer RNA
c. Messenger RNA
d. DNA polymerase
ANS: D
DNA polymerase, not RNA polymerase, is the primary enzyme involved in replication. It adds
bases to the new DNA strand and performs “proofreading” functions. Neither messenger RNA
nor transfer RNA is as important to DNA replication.
REF: p. 39
26. The regions of the heterogeneous nuclear RNA that must be spliced out to form functional
RNA are called:
a. promoter sites.
b. introns.
c. exons.
d. anticodon.
ANS: B
When the mRNA is first transcribed from the DNA template, it reflects exactly the base
sequence of the DNA. In eukaryotes, many RNA sequences are removed by nuclear enzymes,
and the remaining sequences are spliced together to form the functional mRNA that migrates to
the cytoplasm. The excised sequences are called introns (intervening sequences), and the
sequences that are left to code for proteins are called exons. In translation, RNA directs the
synthesis of a polypeptide, a cloverleaf-shaped strand of about 80 nucleotides. The tRNA
molecule has a site where an amino acid attaches. The three-nucleotide sequence at the opposite
side ofthe cloverleaN
f U
isR
ca
S
ll
I
ed
NtG
he
Ta
B
n.
tic
C
oO
do
M
n.
REF: p. 41
27. A 50-year-old male was recently diagnosed with Huntington disease. Transmission of this
disease is associated with:
a. penetrance of a trait.
b. recurrence risk.
c. expressivity.
d. delayed age of onset.
ANS: D
A key feature of Huntington disease is its delayed age of onset such that symptoms are not seen
until 40 years of age or later. The penetrance of a trait is the percentage of individuals with a
specific genotype who also exhibit the expected phenotype. Recurrence risk is the percentage of
family members who will inherit the disease. Expressivity is the extent of variation in phenotype
associated with a particular genotype.
REF: p. 51
28. What type of mutation does not change the amino acid sequence and thus has no observable
consequence?
a. Frameshift
b. Spontaneous
NURSINGTB.COM
TO GET ALL CHAPTERS EMAIL ME AT>>>>>
donc8246@gmail.com
c. Silent
d. Missense
ANS: C
Silent mutations do not change the amino acid sequence and therefore have no consequences.
Frameshift mutations involve the insertion or deletion of one or more base pairs of the DNA
molecule. They alter the amino acid sequence. Spontaneous mutations occur in the absence of
exposure to a mutagen and produce changes in the amino acid sequence. Missense mutations, a
form of base pair substitution, alter amino acids, which produce a change (i.e., the “sense”) in a
single amino acid.
REF: p. 39
29. A nurse is reviewing the pedigree chart. When checking for a proband, what is the nurse
looking for?
a. The person who is first diagnosed with a genetic disease.
b. The individual who has a disease gene but is phenotypically normal.
c. The phenotype of genetic material.
d. The codominance.
ANS: A
The pedigree chart summarizes family relationships and shows which members of a family are
affected by a genetic disease. The pedigree begins with the proband. The person who has a
disease gene but is phenotypically normal is a carrier. The phenotype is the result of both
genotype and environment; it is not a proband. Codominance is not represented by a proband,
but it occurs when the heterozygote is distinguishable from both homozygotes.
REF: p. 50 NURSINGTB.COM
30. Which of the following disorders is manifested primarily in males?
a. Cystic fibrosis
b. Neurofibromatosis
c. Muscular dystrophy
d. Klinefelter syndrome
ANS: C
Muscular dystrophy is manifested primarily in males. Cystic fibrosis, neurofibromatosis, and
Klinefelter syndrome are manifested in both males and females.
REF: p. 55
MULTIPLE RESPONSE
1. When the nurse is teaching the staff about X-linked recessive disorders, which information
should the nurse include? (select all that apply)
a. The trait is seen much more often in females than in males.
b. The trait is never transmitted from father to son.
c. The gene can be transmitted through a series of carrier females.
d. The gene is passed from an affected father to all his daughters.
e. The trait never skips generations.
NURSINGTB.COM
TO GET ALL CHAPTERS EMAIL ME AT>>>>>
donc8246@gmail.com
ANS: B, C, D
The principles of X-linked recessive inheritance include: the trait is seen much more often in
males than in females; the trait is never transmitted from father to son; the gene can be
transmitted through a series of carrier females; the gene is passed from an affected father to all his
daughters, who, as phenotypically normal carriers, transmit it to approximately half their sons,
who are affected. X-linked recessive disorders can skip generations since it is a one in four
chance.
REF: p. 55
NURSINGTB.COM
NURSINGTB.COM
TO GET ALL CHAPTERS EMAIL ME AT>>>>>
donc8246@gmail.com
N R
I
Chapter 03: Epigenetics and Disease
MULTIPLE CHOICE
1. When considering abnormal epigenetic modifications, what factor is currently being viewed as
strongly associated with the development of some cancers?
a. Family genetics
b. Lifestyle choices
c. Environmental stressors
d. Emotional coping skills
ANS: C
Environmental stressors can markedly increase the risk of aberrant epigenetic modification and
are strongly associated with some cancers. While genetics, lifestyle choices, and coping skills can
affect the development and management of cancer, they are not currently considered as being
the primary factors in the epigenetic modification that occurs.
REF: p. 62
2. Housekeeping genes are vital to the function and maintenance of all the body’s cells. What
characteristic is associated with these genes?
a. They lack encoding histones.
b. They are transcriptionally active.
c. Ribosomal RNA genes are absent.
d. Epigenetic silencing has occurred. GB.CM
U S N T O
ANS: B
A small percentage of genes, termed housekeeping genes, are necessary for the function and
maintenance of all cells. These genes escape epigenetic silencing and remain transcriptionally
active in all or nearly all cells. Housekeeping genes include encoding histones, DNA and RNA
polymerases, and ribosomal RNA genes.
REF: p. 64
3. What characteristic of Prader-Willi syndrome is not a characteristic of Angelman syndrome?
a. It is inherited from the father.
b. Mental retardation is observable.
c. Imprinting of an abnormal chromosome 15.
d. Seizure disorder is present.
ANS: A
A well-known disease example of imprinting is associated with a deletion of about 4 million
base (Mb) pairs of the long arm of chromosome 15. When this deletion is inherited from the
father, the child manifests Prader-Willi syndrome, whose features include short stature,
hypotonia, small hands and feet, obesity, mild to moderate mental retardation, and
hypogonadism. The same 4 Mb deletion, when inherited from the mother, causes Angelman
syndrome, which is characterized by severe mental retardation, seizures, and an ataxic gait.
REF: p. 65
NURSINGTB.COM
TO GET ALL CHAPTERS EMAIL ME AT>>>>>
donc8246@gmail.com
4. Research has demonstrated that neural stem cells have an impaired ability to differentiate into
functional neurons when subjected to:
a. ethanol.
b. marijuana.
c. insufficient nutrients.
d. poor oxygenation.
ANS: A
It has been found that treating cultured neural stem cells with ethanol impairs their ability to
differentiate to functional neurons; this impairment seems to be correlated with aberrant, dense
methylation at loci that are active in normal neuronal tissue. The research does not support the
effects of marijuana, insufficient nutrition, or poor oxygenation on the stem cell’s ability to
differentiate appropriately.
REF: p. 67
5. What is the role of inactive MLH1 in the development of some forms of inherited colon
cancer?
a. Deletion of a nucleotide repeat in the DUX4 gene.
b. DNA damage is left unrepaired.
c. Expansion in the number of cytosine-guanine (CG) dinucleotide.
d. Abnormalities of chromosome 11p15.5 that lead to downregulation of IGF2.
ANS: B
A major cause of one form of inherited colon cancer (hereditary nonpolyposis colorectal cancer
[HNPCC]) is the methylation of the promoter region of a gene, MLH1, whose protein product
repairs damaged DNN
AU
. W
RS
he
I
nN
M
G
LT
HB
1 .
be
C
co
O
m
M
es inactive, DNA damage accumulates,
eventually resulting in colon tumors. Facioscapulohumeral muscular dystrophy (FSHMD) is
associated with the DUX4 gene. Fragile X is associated with the cytosine-guanine (CG)
dinucleotide. Downregulation of IGF2 is associated with Russell-Silver syndrome.
REF: p. 69
6. Mutations in the encoding of histone-modifying proteins have been shown to influence the
development of what congenial condition?
a. Cleft palate
b. Acephalous
c. Heart disease
d. Webbed digits
ANS: C
Mutations in genes that encode histone-modifying proteins have been implicated in congenital
heart disease. Research has yet to provide a relationship between abnormal histone-modifying
proteins and cleft palate, acephalous, and webbed digits.
REF: p. 64
7. Which embryonic stem cell characteristic is referred to as totipotent?
a. Ability to differentiate into any type of somatic cell.
b. Ability to repair its own damaged DNA.
NURSINGTB.COM
TO GET ALL CHAPTERS EMAIL ME AT>>>>>
donc8246@gmail.com
c. Ability to determine which parental chromosome copy it will imprint.
d. Ability to minimize the impact of poor in utero nutrition.
ANS: A
Each of the cells in the very early embryo has the potential to give rise to a somatic cell of any
type. These embryonic stem cells are therefore said to be totipotent (“possessing all powers”).
The term totipotent does not infer the ability to repair damaged DNA, select specific
chromosome copies, or adjust for poor in utero nutrition.
REF: p. 64
8. 5-Azacytidine has demonstrated promise in the treatment of which form of cancer?
a. Liver
b. Colon
c. Gallbladder
d. Pancreatic
ANS: D
Though associated with various side effects, including digestive disturbance, 5-azacytidine has
shown promise in the treatment of pancreatic cancer. There is no support for its use in the
treatment of liver, colon, or gallbladder cancers.
REF: p. 70
9. During which stage of human development does the role of epigenetics have the greatest
impact on the development of epigenetic abnormalities?
a. Infancy
b. Puberty
c. In utero
d. Middle age
ANS: C
NURSINGTB.COM
Conditions encountered in utero, during childhood, and even during adolescence or later can have
long-term impacts on epigenetic states, which sometimes can be transmitted across generations.
The impact is not supported for the periods of infancy, puberty, and middle age.
REF: p. 66
10. What comorbid condition does an individual diagnosed with Beckwith-Wiedemann syndrome
have an increased risk of developing?
a. Cancer
b. Diabetes
c. Depression
d. Food allergies
ANS: A
Beckwith-Wiedemann syndrome is accompanied by an increased predisposition to cancer.
There is no current correlation between Beckwith-Wiedemann syndrome and diabetes,
depression, or food allergies.
REF: p. 65
NURSINGTB.COM
TO GET ALL CHAPTERS EMAIL ME AT>>>>>
donc8246@gmail.com
MULTIPLE RESPONSE
1. Screening tools that are epigenetically based have shown promise in diagnosing which types of
cancer? (select all that apply)
a. Colon
b. Breast
c. Skin
d. Bladder
e. Prostate
ANS: A, B, D, E
Monitoring for misregulation of miRNAs has shown promise as a tool for early diagnosis of
cancers of the colon, breast, and prostate. Other epigenetics-based screening approaches have
shown promise for detection of cancers of the bladder, lung, and prostate. Such screenings have
not yet been developed for skin cancers.
REF: p. 69
2. Research has provided support for the theory that epigenetic modifications can result from
deficient in utero nutrition causing which chronic disease? (select all that apply)
a. Obesity
b. Asthma
c. Cardiovascular disease
d. Diabetes
e. Crohn’s disease
ANS: A, C, D
NURSINGTB.COM
When researchers sought to investigate how exposure to famine in utero had impacted
individuals born in a historically prosperous country whose nutritional intake was dramatically
impacted by WWII, they found that individuals who suffered nutritional deprivation in utero
were more likely to suffer from obesity and diabetes as adults than individuals who had not
experienced nutritional deprivation during gestation. Other data sets reveal elevated risk of
cardiovascular and metabolic disease for offspring of individuals exposed during early
development to fluctuations in agricultural yields. The research does support the development
of asthma or Crohn’s disease.
REF: p. 66
NURSINGTB.COM
TO GET ALL CHAPTERS EMAIL ME AT>>>>>
donc8246@gmail.com
Chapter 04: Altered Cellular and Tissue Biology
MULTIPLE CHOICE
1. A report comes back indicating that muscular atrophy has occurred. A nurse recalls that
muscular atrophy involves a decrease in muscle cell:
a. number.
b. size.
c. vacuoles.
d. lipofuscin.
ANS: B
Atrophy is a decrease or shrinkage in cellular size. Hyperplasia is an increase in the number of
cells. Vacuoles are membrane-bound vesicles within the cell that contain cellular debris and
hydrolytic enzymes. Lipofuscin is the yellow-brown age pigment.
REF: p. 74
2. During childhood, the thymus decreases in size, and this is referred to as what type of
atrophy?
a. Physiologic
b. Pathologic
c. Disuse
d. Neurogenic
ANS: A NURSINGTB.COM
A normal decrease in cell size is physiologic atrophy. Atrophy can result from disease
(pathologic), disuse, or nerve injury (neurogenic).
REF: p. 74
3. When planning care for a cardiac patient, the nurse knows that in response to an increased
workload, cardiac myocardial cells will experience hypertrophy which is an:
a. increase in size.
b. decrease in length.
c. increase in excitability.
d. decrease in number.
ANS: A
Hypertrophy is a compensatory increase in the size of cells in response to mechanical stimuli
(also called mechanical load or stress, such as from stretching, repetitive, chronic, pressure, or
volume overload) and consequently increases the size of the affected organ. The cells of the
heart and kidneys are particularly prone to enlargement. A decrease in length is not associated
with hypertrophy. A deficiency of electrolytes or minerals could lead to an increase in excitability;
it is not due to increased workload or related to hypertrophy. A decrease in cell numbers is
referred to as hypoplasia.
REF: p. 75
NURSINGTB.COM
TO GET ALL CHAPTERS EMAIL ME AT>>>>>
donc8246@gmail.com
4. A 55-year-old male with a 30-year history of smoking is examined for respiratory disturbance.
Examination of his airway (bronchial) reveals that stratified squamous epithelial cells have
replaced the normal columnar ciliated cells. This type of cellular adaptation is called:
a. anaplasia.
b. hyperplasia.
c. metaplasia.
d. dysplasia.
ANS: C
Metaplasia is the reversible replacement of one mature cell type by another, sometimes a less
differentiated cell type. Anaplasia is loss of cellular differentiation. Hyperplasia is an increase in
the number of cells resulting from an increased rate of cellular division. Dysplasia refers to
abnormal changes in the size, shape, and organization of mature cells.
REF: p. 77
5. When planning care for the pregnant patient, the nurse will recall that the mammary glands
enlarge as a consequence of:
a. compensatory hyperplasia.
b. hormonal hyperplasia.
c. hormonal anaplasia.
d. compensatory anaplasia.
ANS: B
An increase in the mammary glands during pregnancy is a result of hormonal changes. The
number of mammary cells increases in response to increased hormone levels, not as a
compensatory mechanism. Anaplasia is a reversal to less mature cells.
NURSINGTB.COM
REF: p. 76
6. A 24-year-old female presents with excessive menstrual bleeding. The physician identified
endometrial changes that are due to hormonal imbalances. These cellular changes would be
referred to as:
a. dysplasia.
b. pathologic dysplasia.
c. hyperplasia.
d. pathologic hyperplasia.
ANS: D
Because the changes are due to an imbalance, they would be considered pathologic hyperplasia, a
term more descriptive than simple hyperplasia. The endometrial changes were not abnormal in
size and shape; thus, it is not dysplasia regardless of cause.
REF: pp. 76-77
7. A 55-year-old male is diagnosed with hepatocellular cancer secondary to hepatitis C. If the
cancerous region of the liver is removed, the remaining cells would undergo:
a. pathologic hyperplasia.
b. pathologic metaplasia.
c. compensatory hyperplasia.
d. compensatory aplasia.
NURSINGTB.COM
TO GET ALL CHAPTERS EMAIL ME AT>>>>>
donc8246@gmail.com
ANS: C
Compensatory hyperplasia is an adaptive, not pathologic, mechanism that enables certain
organs to regenerate. Metaplasia is the reversible replacement of one mature cell type by
another, sometimes less differentiated, cell type. Aplasia is not a compensatory mechanism.
REF: p. 76
8. A 40-year-old female’s Pap smear indicates abnormal changes in the shape and organization of
cervical cells. Which term would be used to identify this type of change?
a. Metaplasia
b. Atrophy
c. Hypertrophy
d. Dysplasia
ANS: D
When cervical cells undergo dysplasia, there is a change in their size, shape, and organization.
Metaplasia is the reversible replacement of one mature cell type. The cells have not decreased in
size; therefore, atrophy is incorrect. The cells have not increased in size in response to stimuli;
therefore, they have not hypertrophied.
REF: p. 77
9. A 75-year-old male presents with chest pain on exertion. The chest pain is most likely due to
hypoxic injury secondary to:
a. malnutrition.
b. free radicals.
c. ischemia.
d. chemical toxicity.
ANS: C
NURSINGTB.COM
The cardiac cells are deprived of oxygen, leading to ischemia, a reduction in blood supply to
tissues. The cells are deprived of oxygen; they are not malnourished. Free radicals are electrically
uncharged atoms or groups of atoms that have an unpaired electron. Chemical toxicity is not a
factor in the chest pain.
REF: pp. 78-79
10. A patient has a heart attack that leads to progressive cell injury that causes cell death with
severe cell swelling and breakdown of organelles. What term would the nurse use to define
this process?
a. Adaptation
b. Calcification
c. Apoptosis
d. Necrosis
ANS: D
Necrosis is the sum of cellular changes after local cell death. Cellular adaptation is a reversible,
structural, or functional response to both normal or physiologic conditions and adverse or
pathologic conditions. Calcification is an accumulation of calcium salts. Apoptosis is an active
process of cellular self-destruction.
REF: p. 78, Table 4-1
NURSINGTB.COM
TO GET ALL CHAPTERS EMAIL ME AT>>>>>
donc8246@gmail.com
11. Sodium and water accumulation in an injured cell are a direct result of:
a. decreased ATP production.
b. karyorrhexis.
c. ribosome detachment.
d. dehydration.
ANS: A
A reduction in ATP levels causes the plasma membrane’s sodium-potassium (Na+
-K+
) pump
and sodium-calcium exchange to fail. Sodium and water can then enter the cell freely.
Karyorrhexis means fragmentation of the nucleus into smaller particles or “nuclear dust.”
Ribosome detachment reduces protein synthesis. Dehydration leads to loss of sodium and
water.
REF: p. 80
12. The early dilation (swelling) of the cell’s endoplasmic reticulum results in:
a. increased aerobic metabolism.
b. failure of DNA.
c. reduced protein synthesis.
d. increased Na+
-K+
pump function.
ANS: C
Early dilation of the endoplasmic reticulum causes the ribosomes to detach from the rough
endoplasmic reticulum, reducing protein synthesis. Aerobic metabolism is a normal process and
would not lead to swelling. Cellular swelling will not alter cellular DNA. A reduction in the Na+
-
K+
pump leads to an intracellular accumulation of sodium and calcium and diffusion
of potassium out of the cell. S
N
odU
R
iumS
I
and
N
Gwa
T
B
te.
r c
Ca n M
then enter the cell freely, and cellular
swelling results.
REF: p. 80
13. A 52-year-old male suffered a myocardial infarction secondary to atherosclerosis and
ischemia. Once blood flow is returned to the damaged heart, reperfusion injury occurs as a
result of:
a. oxidation stress.
b. vacuolation.
c. decreased intracellular calcium.
d. lipid acceptor proteins.
ANS: A
Reperfusion injury can result from oxidative stress, increased intracellular calcium, inflammation,
or complement activation. Oxidative stress causes the formation of radicals that cause further
membrane damage and mitochondrial calcium overload. Vacuolation leads to cellular swelling
but is not associated with reperfusion. An increase of intracellular calcium is a cause of
reperfusion injury. Lipid acceptor proteins bind with triglycerides to create fatty liver, and they
do not affect the myocardium.
REF: p. 81
NURSINGTB.COM
TO GET ALL CHAPTERS EMAIL ME AT>>>>>
donc8246@gmail.com
14. A family presents to their primary care provider reporting headache, nausea, weakness, tinnitus,
and vomiting. Which of the following would be the most likely explanation for these symptoms?
a. Lead exposure
b. Carbon monoxide poisoning
c. Ethanol exposure
d. Mercury poisoning
ANS: B
Symptoms related to carbon monoxide poisoning include headache, giddiness, tinnitus (ringing
in the ears), nausea, weakness, and vomiting. Although nausea and vomiting can occur with lead
exposure, lead toxicity is primarily manifested by convulsions and delirium and, with peripheral
nerve involvement, wrist, finger, and sometimes foot paralysis. Ethanol exposure has CNS
effects and would not affect the whole family. Mercury poisoning is manifested by CNS effects
and would not lead to nausea and vomiting.
REF: p. 90
15. A common pathway of irreversible cell injury involves increased intracellular:
a. sodium.
b. potassium.
c. magnesium.
d. calcium.
ANS: D
Increased intracellular calcium levels activate cell enzymes (caspases) that promote cell death by
apoptosis. Persistent ischemia is associated with irreversible injury and necrosis.
Irreversible injury is a ssoc iatN
edUsR
trS
ucI
tuN
raG
llT
y B
w.
ith
Cs
O
eM
vere swelling ofthe mitochondria, severe
damage to plasma membranes, and swelling of lysosomes. Cellular injury is not associated
with sodium, potassium, or magnesium levels.
REF: p. 81
16. A 50-year-old male sustained a closed head injury as a result of a motor vehicle accident. CT
scan revealed a collection of blood between the inner surface of the dura mater and the surface
of the brain. Which type of injury will the nurse be caring for?
a. Subdural hematoma
b. Epidural hematoma
c. Contusion
d. Abrasion
ANS: A
A subdural hematoma occurs when blood is between the inner surface of the dura mater and the
surface of the brain; it can result from blows, falls, or sudden acceleration/deceleration of the
head. An epidural hematoma is a collection of blood between the inner surface of the skull and
the dura; it is most often associated with a skull fracture. A contusion is bleeding into the skin or
underlying tissues. An abrasion (scrape) results from removal of the superficial layers of the skin
caused by friction between the skin and injuring object.
REF: p. 94, Table 4-8
NURSINGTB.COM
TO GET ALL CHAPTERS EMAIL ME AT>>>>>
donc8246@gmail.com
17. A 20-year-old male presents to the emergency department with a jagged sharp-force injury
that is longer than it is deep. Which type of wound will the nurse be caring for?
a. Stab wound
b. Incised wound
c. Puncture wound
d. Chopping wound
ANS: B
An incised wound is a cut that is longer than it is deep. A stab wound is a penetrating
sharp-force injury that is deeper than it is long. A puncture wound is without sharp edges and is
made with an instrument like a nail. Heavy, edged instruments (axes, hatchets, propeller blades)
produce wounds with a combination of sharp and blunt force characteristics.
REF: p. 94, Table 4-8
18. A 30-year-old female presents with a gunshot wound to the head. The wound has seared edges
and a deep penetration of smoke and gunpowder fragments. This wound would be documented
as a(n):
a. exit.
b. intermediate range entrance.
c. contact range entrance.
d. indeterminate range entrance.
ANS: C
A contact range entrance wound is a distinctive type of wound that happens when a gun is held
so the muzzle rests on or presses into the skin surface; in addition to the hole, there is searing of
the edges of the wound from flame and soot or smoke on the edges of the wound. It
is unlikely it is an exit wound
Ns
U
in
R
cS
e tI
he
N
re
Gi
T
s B
on
.
ly
Co
O
nM
e wound and it has seared edges and
gunpowder fragments. An intermediate range entrance wound is surrounded by gunpowder
tattooing or stippling. An indeterminate range entrance wound occurs when flame, soot, or
gunpowder does not reach the skin surface but the bullet does.
REF: p. 95, Table 4-8
19. A 15-year-old female presents to the ER following a physical assault. She has internal damage to
the neck with deep bruising. X-ray reveals fractures of the hyoid bone and tracheal and cricoid
cartilage. Which of the following most likely caused her injuries?
a. Chemical asphyxiation
b. Choking asphyxiation
c. Ligature strangulation
d. Manual strangulation
ANS: D
Squeezing of the neck as with strangulation would fraction the hyoid bone. Chemical
asphyxiation would lead to breathing problems but would not result in fracture. Choking
asphyxiation would lead to swelling of tissues but would not result in fracture. In ligature
strangulation, the mark on the neck is horizontal without the inverted V pattern seen in
hangings. It would not lead to fracture.
REF: p. 96
NURSINGTB.COM
TO GET ALL CHAPTERS EMAIL ME AT>>>>>
donc8246@gmail.com
20. A 55-year-old male has swelling of the feet. Which of the following aided in the development of
swelling?
a. Increased ATP
b. Chloride movement out of the cell
c. Na+
movement into the cell
d. Decreased oncotic pressure
ANS: C
When sodium and water enter the cell freely, cellular swelling, as well as early dilation of the
endoplasmic reticulum, results. Decreased ATP would lead to swelling. Chloride movement out
of the cell would affect muscle contraction but does not lead to swelling. Increased oncotic
pressure would not affect swelling.
REF: p. 97
21. A 35-year-old female is diagnosed with multiple myeloma. Biopsy of the tumor reveals Russell
bodies, and laboratory testing reveals kidney dysfunction. Which substance should the nurse
monitor as it is accumulating in the patient’s body?
a. Glycogen
b. Protein
c. Pigment
d. Melanin
ANS: B
Russell bodies occur due to excess aggregates of protein. Excess glycogen would affect blood
glucose. Increased pigment would not lead to kidney dysfunction. Melanin accumulates in
epithelial cells (keratinocytes) of the skin and retina.
NURSINGTB.COM
REF: p. 99
22. A newborn male is diagnosed with albinism based on skin, eye, and hair appearance. Which
finding will support this diagnosis?
a. Increased melanin
b. Increased hemoproteins
c. Inability to convert tyrosine to DOPA (3,4-dihydroxyphenylalanine)
d. Inability to convert bile to bilirubin
ANS: C
The person with albinism is unable to convert tyrosine to DOPA, an intermediate in melanin
biosynthesis. An increase in melanin would cause skin to be darker. Hemoprotein accumulations
in cells are caused by excessive storage of iron, which is transferred to the cells from the
bloodstream. An inability to convert bile to bilirubin would not lead to albinism.
REF: p. 100
23. A 23-year-old male develops a black eye following a fight. When the aide asks the nurse why
this occurred, the nurse’s best response is that the bruising is due to an accumulation of:
a. transferrin.
b. bilirubin.
c. albumin.
d. hemosiderin.
NURSINGTB.COM
TO GET ALL CHAPTERS EMAIL ME AT>>>>>
donc8246@gmail.com
ANS: D
Hemosiderin is responsible for the color changes in a black eye. Transferrin is a transport
protein responsible for iron transport. Bilirubin is the normal, yellow-to-green pigment of bile
derived from the porphyrin structure of hemoglobin. Albumin is the protein in the serum,
responsible for cellular integrity.
REF: p. 100
24. Liquefactive necrosis occurs in the brain because:
a. debris is not digested by hydrolases.
b. of protein denaturation.
c. it is rich in hydrolytic enzymes and lipids.
d. ischemia results in chemical injury.
ANS: C
Liquefactive necrosis is due to enzymatic action and because cells of the brain are rich in
enzymes. Protein denaturation occurs primarily in the kidneys. Liquefactive necrosis is due to
enzymatic reaction and not to hypoxia or hydrolases.
REF: p. 103
25. A 2-year-old swallowed watch batteries. Following ingestion, kidney function was impaired, and
the heart began to fail. Which of the following was the most likely cause?
a. Karyorrhexis
b. Coagulative necrosis
c. Ammonia accumulation
d. Caseous necrosis
ANS: B NURSINGTB.COM
Coagulative necrosis occurs primarily in the kidneys, heart, and adrenal glands and commonly
results from hypoxia. Karyorrhexis means fragmentation of the nucleus into smaller particles or
“nuclear dust.” Ammonia accumulation is not associated with this toxicity. Caseous necrosis
results from tuberculosis pulmonary infection.
REF: pp. 102-103
26. A group of prison inmates developed tuberculosis following exposure to an infected inmate. On
examination, tissues were soft and granular (like clumped cheese). Which of the following is the
most likely cause?
a. Coagulative necrosis
b. Liquefactive necrosis
c. Caseous necrosis
d. Autonecrosis
ANS: C
Caseous necrosis results from tuberculosis pulmonary infection. Coagulative necrosis occurs
primarily in the kidneys, heart, and adrenal glands, and commonly results from hypoxia.
Liquefactive necrosis results from ischemic injury to neurons and glial cells in the brain.
Autonecrosis is a process of cellular self-digestion and is not due to infection such as
tuberculosis.
REF: p. 103
NURSINGTB.COM
TO GET ALL CHAPTERS EMAIL ME AT>>>>>
donc8246@gmail.com
27. A 50-year-old female became infected with Clostridium bacteria and died a week later.
Examination of her red blood cells revealed lysis of membranes. Which of the following was the
most likely cause of her death?
a. Fat necrosis
b. Wet gangrene
c. Gangrenous necrosis
d. Gas gangrene
ANS: D
Gas gangrene is a special type of gangrene caused by infection of injured tissue by one of many
species of Clostridium. Fat necrosis is cellular dissolution caused by powerful enzymes, called
lipases, that occur in the breast, pancreas, and other abdominal structures. Wet gangrene
develops when neutrophils invade the site, causing liquefactive necrosis.
Gangrenous necrosis is due to death of tissue and results from severe hypoxic injury.
REF: p. 104
28. While reading a textbook, a student reads the term apoptosis. The student recalls that
apoptosis is a condition in which cells program themselves to:
a. atrophy.
b. die.
c. regenerate.
d. age.
ANS: B
In apoptosis, cells are programmed to die. Apoptosis is not associated with cell atrophy,
regeneration, or aging. NURSINGTB.COM
REF: p. 104
29. A 50-year-old male intravenous drug user is diagnosed with hepatitis C. Examination of the
liver reveals cell death secondary to:
a. fat necrosis.
b. physiologic apoptosis.
c. infection-induced apoptosis.
d. pyknosis.
ANS: C
With hepatitis C, the liver will demonstrate apoptosis that is a result of the viral infection. Fat
necrosis occurs with enzymatic action due to lipases. Apoptosis is not a normal physiological
process. Pyknosis occurs when the nucleus shrinks and becomes a small, dense mass of genetic
material.
REF: p. 104
30. What principle should the nurse remember when trying to distinguish aging from diseases?
a. It is difficult to tell the difference because both processes are believed to result
from cell injury.
b. It is easy to tell normal processes from abnormal processes.
c. Disease, unlike aging, has a genetic component.
NURSINGTB.COM
TO GET ALL CHAPTERS EMAIL ME AT>>>>>
donc8246@gmail.com
U S N T O
d. Aging is defined as exceeding life expectancy, but not maximal life span.
ANS: A
It is difficult to differentiate between aging and disease because both occur secondary to cellular
aging. It is not easy to differentiate normal processes from abnormal because aging appears as a
normal process. Disease and aging have a possible genetic component. Aging is a time-dependent
loss of structure and function that proceeds slowly and in such small increments that it appears
to be the result of the accumulation of small, imperceptible injuries. It is not a time period
outside of life expectancy.
REF: p. 107
31. When a nurse observes muscle stiffening occurring within 6–14 hours after death, the nurse
should document this finding as the presence of:
a. livor mortis.
b. gangrene.
c. algor mortis.
d. rigor mortis.
ANS: D
Rigor mortis occurs within 6 hours after death and is evidenced by muscle stiffening. Livor
mortis is a purple discoloration. Gangrene refers to death of tissue and results from severe
hypoxic injury and does not lead to stiffening. Algor mortis is postmortem reduction of body
temperature.
REF: pp. 109-110
32. When a nurse is checking a uN
rinaR
lysiI
s, t h
Ge fi
B
nd
.
in
C
g tM
hat would alert the nurse to cellular injury is
the presence of:
a. slight glucose.
b. excessive protein.
c. blood.
d. urea.
ANS: B
The presence of protein in the urine in significant amounts indicates cellular injury and altered
cellular function. Neither glucose nor blood is normally present in the urine, but its presence is
not indicative of altered cellular function. Urea is an expected substance in the kidney.
REF: p. 99
33. An 86-year-old female patient has the wasting syndrome of aging, making her vulnerable to
falls, functional decline, disease, and death. The nurse knows this patient is experiencing:
a. frailty.
b. sarcopenia.
c. somatic death.
d. cellular aging.
ANS: A
NURSINGTB.COM
TO GET ALL CHAPTERS EMAIL ME AT>>>>>
donc8246@gmail.com
U S N
Frailty leaves the individual vulnerable to falls, disease, and death. Sarcopenia is loss of muscle
mass. It is associated with aging but is not as severe as frailty. Somatic death is death of the
whole person. Cellular aging occurs at the cellular level and is characteristic of aging, including
cell atrophy, decreased function, and loss of cells.
REF: p. 109
34. Confirmation of somatic death is based on:
a. presence of algor mortis.
b. presence of livor mortis.
c. complete cessation of respiration and circulation.
d. change in skin color to pale yellow.
ANS: C
The most notable manifestations are complete cessation of respiration and circulation. Algor
mortis is postmortem reduction of body temperature and is not confirmation of somatic death.
Livor mortis is muscle stiffening and is not confirmation of somatic death. Change in skin color
to pale yellow does occur, but complete cessation of respiration and circulation confirms somatic
death.
REF: p. 109
MULTIPLE RESPONSE
1. A patient has been reading on the Internet that light to moderate intake of alcohol is
cardioprotective. When the patient asks the nurse what this means, the nurse should respond
that the heart is protected byN
whiR
ch o
I
f thG
e foB
ll.
ow
Cing
Mmechanisms? (select all that apply)
a. Increased levels of high-density lipoprotein cholesterol
b. Prevention of clot formation
c. Reduction in platelet aggregation
d. Decrease in blood pressure
e. Increased collateral circulation
f. Decreased folate absorption
ANS: A, B, C, D
The suggested mechanisms for cardioprotection by low to moderate alcohol intake include
increased levels of high-density lipoprotein cholesterol (HDL-c), prevention of clot formation,
reduction in platelet aggregation, and decrease in blood pressure. Increased collateral circulation
is not a benefit of alcohol. Alcohol consumption can decrease folate absorption, which can lead
to a nutritional deficient.
REF: p. 91
2. After ingestion of lead, what organ systems should the nurse monitor because they are the
most sensitive to the effects of lead? (select all that apply)
a. Heart
b. Lungs
c. Liver
d. Kidneys
e. Brain
NURSINGTB.COM
TO GET ALL CHAPTERS EMAIL ME AT>>>>>
donc8246@gmail.com
f. Hematopoietic
ANS: D, E, F
The organ systems primarily affected by lead ingestion include the nervous system (brain), the
hematopoietic system (tissues that produce blood cells), and the kidneys. The heart, liver, and
lungs are not affected.
REF: p. 89
NURSINGTB.COM
NURSINGTB.COM
TO GET ALL CHAPTERS EMAIL ME AT>>>>>
donc8246@gmail.com
Chapter 05: Fluids and Electrolytes, Acids and Bases
MULTIPLE CHOICE
1. A nurse is reviewing lab reports. The nurse recalls blood plasma is located in which of the
following fluid compartments?
a. Intracellular fluid (ICF)
b. Extracellular fluid (ECF)
c. Interstitial fluid
d. Intravascular fluid
ANS: D
Blood plasma is the intravascular fluid. ICF is fluid in the cells. ECF is all the fluid outside the
cells. Interstitial fluid is fluid between the cells and outside the blood vessels.
REF: p. 114
2. A 35-year-old male weighs 70 kg. Approximately how much of this weight is considered the
total volume of body water?
a. 5 L
b. 10 L
c. 28 L
d. 42 L
ANS: D
The total volume of body waN
teU
r f
R
oS
r a
I
7
N
0G
-kT
gB
pe
.
rs
C
oO
n M
is about 42 L or two thirds of 70 kg.
REF: p. 114
3. While planning care for elderly individuals, the nurse remembers the elderly are at a higher risk
for developing dehydration because they have:
a. a higher total body water volume.
b. decreased muscle mass.
c. increased thirst.
d. an increased tendency toward developing edema.
ANS: B
The elderly are at higher risk for dehydration due to a decrease in muscle mass. The elderly have
a decrease in total body water and thirst. The increased tendency to develop edema is not related
to dehydration.
REF: p. 131, Geriatric Considerations
4. Which of the following patients should the nurse assess for decreased oncotic pressure in the
capillaries? A patient with:
a. a high-protein diet.
b. liver failure.
c. low blood pressure.
d. low blood glucose.
NURSINGTB.COM
TO GET ALL CHAPTERS EMAIL ME AT>>>>>
donc8246@gmail.com
ANS: B
Liver failure leads to lost or diminished plasma albumin production, and this contributes to
decreased plasma oncotic pressure. A high-protein diet would provide albumin for the
maintenance of oncotic pressure. Low blood pressure would lead to decreased hydrostatic
pressure. Decreased glucose does not affect oncotic pressure.
REF: p. 116
5. Water movement between the ICF and ECF compartments is determined by:
a. osmotic forces.
b. plasma oncotic pressure.
c. antidiuretic hormone.
d. buffer systems.
ANS: A
Osmotic forces determine water movement between the ECF and ICF compartments. Oncotic
pressure pulls water at the end of the capillary, which makes it move between intra and extra as
interstitial is considered extra. The antidiuretic hormone regulates water balance, which would
make water move between the intra- and extracellular spaces. Buffer systems help regulate acid
balance.
REF: p. 115
6. An experiment was designed to test the effects of the Starling forces on fluid movement.
Which of the following alterations would result in fluid moving into the interstitial space?
a. Increased capillary oncotic pressure.
b. Increased interstitial hydrostatic pressure.
c. Decreased capillary hydrN
osU
taR
tic
Sp
I
re
N
sG
suT
re
B
. .COM
d. Increased interstitial oncotic pressure.
ANS: D
Increased interstitial oncotic pressure would attract water from the capillary into the interstitial
space. Increased capillary oncotic pressure would attract water from the interstitial space back
into the capillary. Increased interstitial hydrostatic pressure would attract movement of water
from the interstitial spaces into the capillary. Decreased capillary hydrostatic pressure would
move water into the capillaries.
REF: p. 115
7. When planning care for a dehydrated patient, the nurse remembers that the principle of water
balance is closely related to the balance of:
a. potassium.
b. chloride.
c. bicarbonate.
d. sodium.
ANS: D
Because water follows the osmotic gradients established by changes in salt concentration, water
balance is tied to sodium balance, not that of potassium, chloride, or bicarbonate.
REF: pp. 116-117 | p. 118, Figure 5-5
NURSINGTB.COM
TO GET ALL CHAPTERS EMAIL ME AT>>>>>
donc8246@gmail.com
8. A 70-year-old male with chronic renal failure presents with edema. Which of the following is the
most likely cause of this condition?
a. Increased capillary oncotic pressure.
b. Decreased interstitial oncotic pressure.
c. Increased capillary hydrostatic pressure.
d. Increased interstitial hydrostatic pressure.
ANS: C
Increased capillary hydrostatic pressure would facilitate increased movement from the capillary
to the interstitial space, leading to edema. Increased capillary (plasma) oncotic pressure attracts
water from the interstitial space back into the capillary. Decreased interstitial oncotic pressure
would keep water in the capillary. Increased interstitial hydrostatic pressure would facilitate
increased water movement from the interstitial space into the capillary.
REF: pp. 115-116
9. A 10-year-old male is brought to the emergency room (ER) incoherent and semiconscious. CT
scan reveals that he is suffering from cerebral edema. This type of edema is referred to as:
a. localized edema.
b. generalized edema.
c. pitting edema.
d. lymphedema.
ANS: A
Cerebral edema is a form of localized edema. Generalized edema is manifested by a more
uniform distribution of fluid in interstitial spaces. When pressure is applied, pitting edema results
in an indention in the skin. Lymphedema is due to swelling in interstitial spaces, primarily in the
extremities. NURSINGTB.COM
REF: p. 116
10. A nurse is teaching the staff about antidiuretic hormone (ADH). Which information should
the nurse include? Secretion of ADH is stimulated by:
a. increased serum potassium.
b. increased plasma osmolality.
c. decreased renal blood flow.
d. generalized edema.
ANS: B
ADH is secreted when plasma osmolality increases or circulating blood volume decreases and
blood pressure drops. ADH is not secreted by an increase in potassium, a decrease in renal
blood flow, or the presence of generalized edema.
REF: p. 118
11. Which statement by the staff indicates teaching was successful concerning aldosterone?
Secretion of aldosterone results in:
a. decreased plasma osmolality.
b. increased serum potassium levels.
c. increased blood volume.
d. localized edema.
NURSINGTB.COM
TO GET ALL CHAPTERS EMAIL ME AT>>>>>
donc8246@gmail.com
ANS: C
Aldosterone promotes renal sodium and water reabsorption and excretion of potassium, thus
increasing blood volume. Aldosterone secretion would cause increased plasma osmolality but it
does not promote the development of localized edema; it affects blood volume.
REF: p. 117
12. A 25-year-old male is diagnosed with a hormone-secreting tumor of the adrenal cortex. Which
finding would the nurse expect to see in the lab results?
a. Decreased blood volume
b. Decreased blood K+
levels
c. Increased urine Na+
levels
d. Increased white blood cells
ANS: B
Aldosterone is secreted from the adrenal cortex. It promotes renal sodium and water
reabsorption and excretion of potassium, leading to decreased potassium levels. Blood volume
actually increases with aldosterone secretion. Aldosterone promotes sodium reabsorption,
leading to normal or decreased Na+
levels, and is not associated with white blood cells.
REF: p. 117
13. A patient has been searching on the Internet about natriuretic hormones. When the patient
asks the nurse what these hormones do, how should the nurse respond? Natriuretic hormones
affect the balance of:
a. calcium.
b. sodium.
c. magnesium.
d. potassium.
ANS: B
NURSINGTB.COM
Natriuretic hormones are sometimes called a “third factor” in sodium regulation but have no
influence on calcium, magnesium, or potassium balance.
REF: p. 117
14. A 5-year-old male presents to the ER with delirium and sunken eyes. After diagnosing him
with severe dehydration, the primary care provider orders fluid replacement. The nurse
administers a hypertonic intravenous solution. Which of the following would be expected?
a. Symptoms subside quickly
b. Increased ICF volume
c. Decreased ECF volume
d. Intracellular dehydration
ANS: D
A hypertonic solution would cause fluid to move into the extracellular space, leading to
intracellular dehydration. With this solution, his symptoms will not subside quickly because his
cells will lose fluid. His intracellular volume will decrease and his extracellular volume will
increase.
REF: p. 120
NURSINGTB.COM
TO GET ALL CHAPTERS EMAIL ME AT>>>>>
donc8246@gmail.com
15. Which of the following patients is the most at risk for developing hypernatremia? A patient
with:
a. vomiting.
b. diuretic use.
c. dehydration.
d. hypoaldosteronism.
ANS: C
Dehydration leads to hypernatremia because an increase in sodium occurs with a net loss in
water. Vomiting and diuretic use leads to hyponatremia. Hypoaldosteronism leads to
hyponatremia.
REF: p. 119
16. Which of the following conditions would cause the nurse to monitor for hyperkalemia?
a. Excess aldosterone
b. Acute acidosis
c. Insulin usage
d. Metabolic alkalosis
ANS: B
In acidosis, ECF hydrogen ions shift into the cells in exchange for ICF potassium and sodium;
hyperkalemia and acidosis therefore often occur together. Acidosis does not cause excess
aldosterone. Insulin would help treat hyperkalemia, not cause it.
Alkalosis does not lead to hyperkalemia.
REF: p. 124
NURSINGTB.COM
17. Which organ system should the nurse monitor when the patient has long-term potassium
deficits?
a. Central nervous system (CNS)
b. Lungs
c. Kidneys
d. Gastrointestinal tract
ANS: C
Long-term potassium deficits lasting more than 1 month may damage renal tissue, with
interstitial fibrosis and tubular atrophy. Long-term potassium deficits are not associated with
damage to the CNS, GI tract, or lungs.
REF: pp. 123-124
18. A 42-year-old female presents to her primary care provider reporting muscle weakness and
cardiac abnormalities. Laboratory tests indicate that she is hypokalemic. Which of the
following could be the cause of her condition?
a. Respiratory acidosis
b. Constipation
c. Hypoglycemia
d. Laxative abuse
ANS: D
NURSINGTB.COM
TO GET ALL CHAPTERS EMAIL ME AT>>>>>
donc8246@gmail.com
Losses of potassium from body stores are usually caused by gastrointestinal and renal disorders.
Diarrhea, intestinal drainage tubes or fistulae, and laxative abuse also result in hypokalemia.
Acidosis is related to hyperkalemia, not hypokalemia. Constipation can occur with hypokalemia
but does not cause it. Hypoglycemia is not related to muscle weakness.
REF: p. 123
19. A 19-year-old male presents to his primary care provider reporting restlessness, muscle
cramping, and diarrhea. Lab tests reveal that he is hyperkalemic. Which of the following
could have caused his condition?
a. Primary hyperaldosteronism
b. Acidosis
c. Insulin secretion
d. Diuretic use
ANS: B
During acute acidosis, hydrogen ions accumulate in the ICF and potassium shifts out of the cell
to the ECF, causing hyperkalemia. Primary hyperaldosteronism is associated with hypokalemia,
not hyperkalemia. Insulin secretion helps reduce potassium levels in the cell; it does not cause
hyperkalemia. Diuretics would cause hypokalemia, not hyperkalemia.
REF: p. 122
20. A 60-year-old female is diagnosed with hyperkalemia. Which assessment finding should the
nurse expect to observe?
a. Weak pulse
b. Excessive thirst
c. Oliguria
d. Constipation
ANS: C
NURSINGTB.COM
Hyperkalemia is manifested by oliguria. Hypokalemia is manifested by a weak pulse; it is not
caused by hyperkalemia. Hypokalemia is manifested by excessive thirst. Diarrhea, not
constipation, is a manifestation of hyperkalemia.
REF: p. 124, Table 5-6
21. Which of the following buffer pairs is considered the major plasma buffering system?
a. Protein/fat
b. Carbonic acid/bicarbonate
c. Sodium/potassium
d. Amylase/albumin
ANS: B
The carbonic acid/bicarbonate buffer pair operates in both the lung and the kidney and is a
major extracellular buffer. Protein and fat are nutrients not related to the buffering system.
Sodium and potassium are electrolytes for fluid and electrolyte balance, not the major plasma
buffering system for acid-base balance. Amylase is a carbohydrate enzyme, and albumin is a
protein; neither is a buffering system.
REF: p. 125
NURSINGTB.COM
TO GET ALL CHAPTERS EMAIL ME AT>>>>>
donc8246@gmail.com
22. A nurse recalls that regulation of acid-base balance through removal or retention of volatile
acids is accomplished by the:
a. buffer systems.
b. skin.
c. lungs.
d. liver.
ANS: C
The volatile acid is carbonic acid (H2CO3), which readily dissociates into carbon dioxide
(CO2) and water (H2O). The CO2 is then eliminated by the lungs. Buffer systems are
throughout the body and operate in the extracellular and intracellular systems. Neither the
liver nor the skin regulates acid-base balance.
REF: p. 125
23. Which patient is most prone to metabolic alkalosis? A patient with:
a. retention of metabolic acids.
b. hypoaldosteronism.
c. excessive loss of chloride (Cl).
d. hyperventilation.
ANS: C
When acid loss is caused by vomiting, renal compensation is not very effective because loss of
Cl stimulates renal retention of bicarbonate, leading to alkalosis. Retention of metabolic acids
would lead to acidosis, not alkalosis. Hypoaldosteronism leads to hyponatremia and does not
cause alkalosis. Hyperventilation leads to respiratory alkalosis, not metabolic alkalosis.
REF: p. 127 NURSINGTB.COM
24. Which patient should the nurse assess for both hyperkalemia and metabolic acidosis? A
patient diagnosed with:
a. diabetes insipidus.
b. pulmonary disorders.
c. Cushing syndrome.
d. renal failure.
ANS: D
Renal failure is associated with hyperkalemia and metabolic acidosis. Diabetes insipidus results
in hypernatremia. Pulmonary disorders are a cause of respiratory acidosis or alkalosis but do not
affect hyperkalemia. Cushing syndrome results in hypernatremia.
REF: p. 124 | p. 127
25. For a patient experiencing metabolic acidosis, the body will compensate by:
a. excreting H+
through the kidneys.
b. hyperventilating.
c. retaining CO2 in the lungs.
d. secreting aldosterone.
ANS: B
NURSINGTB.COM
TO GET ALL CHAPTERS EMAIL ME AT>>>>>
donc8246@gmail.com
It is the lungs hyperventilating that would compensate for metabolic acidosis by blowing off
CO2, not any function associated with the kidneys. CO2 retention would increase the acidotic
state. Aldosterone would conserve water but does not help compensate for acidosis.
REF: p. 127
26. Which finding would support the diagnosis of respiratory acidosis?
a. Vomiting
b. Hyperventilation
c. Pneumonia
d. An increase in noncarbonic acids
ANS: C
Respiratory acidosis occurs with hypoventilation, and pneumonia leads to hypoventilation.
Vomiting leads to loss of acids and then to alkalosis. Hyperventilation leads to respiratory
alkalosis. Metabolic acidosis is caused by an increase in noncarbonic acids.
REF: pp. 128-129
27. A 54-year-old male with a long history of smoking complains of excessive tiredness, shortness
of breath, and overall ill feelings. Lab results reveal decreased pH, increased CO2, and normal
bicarbonate ion. These findings help to confirm the diagnosis of:
a. respiratory alkalosis.
b. metabolic acidosis.
c. respiratory acidosis.
d. metabolic alkalosis.
ANS: C NURSINGTB.COM
A decreased pH indicates acidosis. With increased CO2, it is respiratory acidosis. The
bicarbonate is normal, so it cannot be metabolic acidosis.
REF: pp. 128-129
28. For a patient with respiratory acidosis, chronic compensation by the body will include:
a. kidney excretion of H+
.
b. kidney excretion of HCO3.
c. prolonged exhalations to blow off CO2.
d. protein buffering.
ANS: A
The kidneys excrete H+
to compensate for respiratory acidosis. The kidneys do not excrete
HCO3 to compensate; this would increase acidosis. Prolonged exhalations would not be
effective for compensation, especially in a chronic state. Protein buffering is intracellular and
would not be effective enough to compensate for respiratory acidosis.
REF: pp. 128-130
29. A 55-year-old female presents to her primary care provider and reports dizziness, confusion,
and tingling in the extremities. Blood tests reveal an elevated pH, decreased PCO2, and slightly
decreased HCO3. Which of the following is the most likely diagnosis?
a. Respiratory alkalosis with renal compensation
b. Respiratory acidosis with renal compensation
NURSINGTB.COM
TO GET ALL CHAPTERS EMAIL ME AT>>>>>
donc8246@gmail.com
c. Metabolic alkalosis with respiratory compensation
d. Metabolic acidosis with respiratory compensation
ANS: A
With an elevated pH, the diagnosis must be alkalosis. Since the PCO2 is low, it is likely
respiratory, with a slight decrease in HCO3 indicating renal compensation.
REF: p. 130
MULTIPLE RESPONSE
1. A 60-year-old male with a 30-year history of smoking is diagnosed with a hormone-secreting
lung tumor. Further testing indicates that the tumor secretes ADH. Which of the following
assessment findings should the nurse expect? (select all that apply)
a. Confusion
b. Weakness
c. Nausea
d. Muscle twitching
e. Increased reflexes
ANS: A, B, C, D
Secretion of ADH leads to water intoxication with symptoms of cerebral edema, including
confusion, convulsions, weakness, nausea, and muscle twitching. Depressed reflexes are
associated with water intoxication.
REF: p. 119 | p. 121
NURSINGTB.COM
NURSINGTB.COM
TO GET ALL CHAPTERS EMAIL ME AT>>>>>
donc8246@gmail.com
Chapter 06: Innate Immunity: Inflammation and Wound Healing
MULTIPLE CHOICE
1. A public health nurse is teaching the community about health promotion. Which information
should the nurse include for innate immunity? Innate immunity is gained:
a. following an illness.
b. at birth.
c. via injection of specific antibodies.
d. in adulthood.
ANS: B
Innate immunity is present at birth. It is not dependent on illness or injection.
REF: p. 134
2. Which statement indicates teaching was successful regarding collectins? Collectins are
produced by the:
a. kidneys.
b. bowel.
c. lungs.
d. integument.
ANS: C
Collectins are produced by the lungs. Collectins are not associated with the kidneys, bowel, or
integumentary system. NURSINGTB.COM
REF: p. 135
3. A 20-year-old male received a knife wound to the arm during an altercation. Which of the
following types of immunity was compromised?
a. Innate immunity
b. Inflammatory response
c. Adaptive immunity
d. Specific immunity
ANS: A
The epithelial cells of the skin are a part of innate immunity. The inflammatory response is
not a type of immunity. Adaptive immunity is represented by the normal flora of the bowel.
Specific immunity is a type of adaptive immunity and is not associated with a break in skin
integrity.
REF: pp. 134-135
4. Biochemical secretions that trap and kill microorganisms include:
a. hormones.
b. neurotransmitters.
c. earwax.
d. gastric acid.
NURSINGTB.COM
TO GET ALL CHAPTERS EMAIL ME AT>>>>>
donc8246@gmail.com
ANS: C
Epithelial cells secrete several substances that protect against infection, including earwax.
Hormones do not contain biochemical secretions that trap and kill microorganisms.
Neurotransmitters carry important messages, and gastric acid helps break down food into its
component parts, but neither contains biochemical secretions.
REF: p. 135
5. A 25-year-old female presents to her primary care provider reporting vaginal discharge of a
white, viscous, and foul-smelling substance. She reports that she has been taking antibiotics for
the past 6 months. Which finding will the nurse most likely see on the microorganism report?
a. Clostridium difficile overgrowth
b. Decreased Lactobacillus
c. Streptococcus overgrowth
d. Decreased Candida albicans
ANS: B
Diminished colonization with Lactobacillus that occurs as a result of prolonged antibiotic
treatment increases the risk for vaginal infections, such as vaginosis. Clostridium difficile and
Candida albicans occur in the colon, not the vagina. Streptococcus overgrowth will occur in the
mouth.
REF: p. 136
6. When an aide asks the nurse about the purpose of the inflammatory process, how should the
nurse respond? N R I G B.C M
a. To provide specific respons
U
es tS
owa
N
rd a
T
ntigens
O
.
b. To lyse cell membranes of microorganisms.
c. To prevent infection of the injured tissue.
d. To create immunity against subsequent tissue injury.
ANS: C
One purpose of the inflammatory process is to prevent infection and further damage by
contaminating microorganisms. Specific response toward antigens is a part of the complement
system that assists in the inflammatory response, but not its purpose. Lysis of cell membranes is
part of the process of phagocytosis, which removes foreign material. Immunity cannot be
achieved against future tissue injury.
REF: p. 137
7. A child fell off a swing and scraped his right knee. The injured area becomes painful. What
else will the nurse observe upon assessment?
a. Vasoconstriction at injured site
b. Decreased RBC concentration at injured site
c. Pale skin at injured site
d. Edema at injured site
ANS: D
Increased vascular permeability and leakage of fluid out of the vessel cause edema at the site of
injury. Vasodilation occurs, bringing increased RBCs to the site and causing redness.
NURSINGTB.COM
TO GET ALL CHAPTERS EMAIL ME AT>>>>>
donc8246@gmail.com
REF: p. 137
8. A nurse recalls the mast cell, a major activator of inflammation, initiates the inflammatory
response through the process of:
a. chemotaxis.
b. endocytosis.
c. degranulation.
d. opsonization.
ANS: C
Degranulation of mast cells is a major cellular component of inflammation. Chemotaxis is the
process of white cell migration. Endocytosis and opsonization are parts of phagocytosis but are
not factors in mast cell response.
REF: pp. 144-145
9. Which of the following individuals would be at greatest risk for an opportunistic infection?
a. 18-year-old with diabetes
b. 70-year-old with congestive heart failure
c. 24-year-old who is immunocompromised
d. 30-year-old with pneumonia
ANS: C
Opportunistic microorganisms can cause disease if the individual’s defenses are compromised.
Diabetes, congestive heart failure, and pneumonia are not associated with immunocompromised
disorders.
NURSINGTB.COM
REF: p. 137
10. The directional migration of leukocytes along a chemical gradient is termed:
a. chemotaxis.
b. endocytosis.
c. margination.
d. diapedesis.
ANS: A
Chemotaxis is the process by which leukocytes undergo directed migration. Endocytosis is a
form of engulfment and a part of phagocytosis. Margination occurs when leukocytes adhere to
endothelial cells in the walls of vessels. Diapedesis is the emigration of the cells through cell
junctions that have loosened in response to inflammatory mediators.
REF: p. 147
11. A 20-year-old male shoots his hand with a nail gun while replacing roofing shingles. Which of
the following cell types would be the first to aid in killing bacteria to prevent infection in his
hand?
a. Eosinophils
b. Neutrophils
c. Leukotrienes
d. Monocytes

More Related Content

Similar to TEST BANK For Huether and McCance's Understanding Pathophysiology, Canadian 2nd Edition.pdf

Biology first grading review
Biology first grading reviewBiology first grading review
Biology first grading review
gailma13
 
TEST BANK For Robbins & Kumar Basic Pathology, 11th Edition by Vinay Kumar, A...
TEST BANK For Robbins & Kumar Basic Pathology, 11th Edition by Vinay Kumar, A...TEST BANK For Robbins & Kumar Basic Pathology, 11th Edition by Vinay Kumar, A...
TEST BANK For Robbins & Kumar Basic Pathology, 11th Edition by Vinay Kumar, A...
rightmanforbloodline
 
TEST BANK FOR PATHOPHYSIOLOGY 9TH EDITION by MCCANCE 2023.pdf
TEST BANK FOR PATHOPHYSIOLOGY 9TH EDITION by MCCANCE 2023.pdfTEST BANK FOR PATHOPHYSIOLOGY 9TH EDITION by MCCANCE 2023.pdf
TEST BANK FOR PATHOPHYSIOLOGY 9TH EDITION by MCCANCE 2023.pdf
nursing premium
 
Test Bank for Applied Pathophysiology A Conceptual Approach to theMechanisms ...
Test Bank for Applied Pathophysiology A Conceptual Approach to theMechanisms ...Test Bank for Applied Pathophysiology A Conceptual Approach to theMechanisms ...
Test Bank for Applied Pathophysiology A Conceptual Approach to theMechanisms ...
nursing premium
 
Cell Physiology Spring 2016 Name Page 1 This test is .docx
Cell Physiology Spring 2016 Name Page 1 This test is .docxCell Physiology Spring 2016 Name Page 1 This test is .docx
Cell Physiology Spring 2016 Name Page 1 This test is .docx
tidwellveronique
 
Ganong-Physiology-Compilation.pdf
Ganong-Physiology-Compilation.pdfGanong-Physiology-Compilation.pdf
Ganong-Physiology-Compilation.pdf
RochelleJoyceArado
 
Deciphering Cellular Respiration: Stages, Mechanisms, and Implications | The ...
Deciphering Cellular Respiration: Stages, Mechanisms, and Implications | The ...Deciphering Cellular Respiration: Stages, Mechanisms, and Implications | The ...
Deciphering Cellular Respiration: Stages, Mechanisms, and Implications | The ...
The Lifesciences Magazine
 
Cell structure and function re.pdf
Cell structure and function re.pdfCell structure and function re.pdf
Cell structure and function re.pdf
SoheirRizk
 
Biology 204 Principles of Biology I Assignment 1C
Biology 204 Principles of Biology I Assignment 1CBiology 204 Principles of Biology I Assignment 1C
Biology 204 Principles of Biology I Assignment 1C
Michael Taylor
 
SELF-TEST Amswi each af the follimins suesions by reiecting the ene an.pdf
SELF-TEST Amswi each af the follimins suesions by reiecting the ene an.pdfSELF-TEST Amswi each af the follimins suesions by reiecting the ene an.pdf
SELF-TEST Amswi each af the follimins suesions by reiecting the ene an.pdf
12angeldesignworld1
 
LF & PPG- FINAL EXAM, TOS, & SUMMATIVE TEST.docx
LF & PPG- FINAL EXAM, TOS, & SUMMATIVE TEST.docxLF & PPG- FINAL EXAM, TOS, & SUMMATIVE TEST.docx
LF & PPG- FINAL EXAM, TOS, & SUMMATIVE TEST.docx
ThraiaGabriellaMerca
 
Cell Physiology Basics
Cell Physiology BasicsCell Physiology Basics
Cell Physiology Basics
Timothy Zagada
 
Ib ch 1 question-answer
Ib ch 1 question-answerIb ch 1 question-answer
Ib ch 1 question-answer
Ayushikaushik15
 
ECOSYSTEM-Life-Energy.pptx
ECOSYSTEM-Life-Energy.pptxECOSYSTEM-Life-Energy.pptx
ECOSYSTEM-Life-Energy.pptx
PT2JJ
 
test bank Lehninger Principles of Biochemistry, 8e David Nelson, Michael Cox ...
test bank Lehninger Principles of Biochemistry, 8e David Nelson, Michael Cox ...test bank Lehninger Principles of Biochemistry, 8e David Nelson, Michael Cox ...
test bank Lehninger Principles of Biochemistry, 8e David Nelson, Michael Cox ...
NailBasko
 
Tutorial mitosis
Tutorial mitosisTutorial mitosis
Tutorial mitosis
Osama Barayan
 
Assignment On Bioenergetics
Assignment On BioenergeticsAssignment On Bioenergetics
Assignment On BioenergeticsMugdha Padhye
 
9.1 Cellular Respiration_ An Overview.pdf
9.1 Cellular Respiration_ An Overview.pdf9.1 Cellular Respiration_ An Overview.pdf
9.1 Cellular Respiration_ An Overview.pdf
MervatMarji2
 
Cellular Respiration Essay
Cellular Respiration EssayCellular Respiration Essay
Cellular Respiration Essay
Paper Writer Services
 
2. Absorption & Secretion Of Materials
2. Absorption & Secretion Of Materials2. Absorption & Secretion Of Materials
2. Absorption & Secretion Of Materialsrossbiology
 

Similar to TEST BANK For Huether and McCance's Understanding Pathophysiology, Canadian 2nd Edition.pdf (20)

Biology first grading review
Biology first grading reviewBiology first grading review
Biology first grading review
 
TEST BANK For Robbins & Kumar Basic Pathology, 11th Edition by Vinay Kumar, A...
TEST BANK For Robbins & Kumar Basic Pathology, 11th Edition by Vinay Kumar, A...TEST BANK For Robbins & Kumar Basic Pathology, 11th Edition by Vinay Kumar, A...
TEST BANK For Robbins & Kumar Basic Pathology, 11th Edition by Vinay Kumar, A...
 
TEST BANK FOR PATHOPHYSIOLOGY 9TH EDITION by MCCANCE 2023.pdf
TEST BANK FOR PATHOPHYSIOLOGY 9TH EDITION by MCCANCE 2023.pdfTEST BANK FOR PATHOPHYSIOLOGY 9TH EDITION by MCCANCE 2023.pdf
TEST BANK FOR PATHOPHYSIOLOGY 9TH EDITION by MCCANCE 2023.pdf
 
Test Bank for Applied Pathophysiology A Conceptual Approach to theMechanisms ...
Test Bank for Applied Pathophysiology A Conceptual Approach to theMechanisms ...Test Bank for Applied Pathophysiology A Conceptual Approach to theMechanisms ...
Test Bank for Applied Pathophysiology A Conceptual Approach to theMechanisms ...
 
Cell Physiology Spring 2016 Name Page 1 This test is .docx
Cell Physiology Spring 2016 Name Page 1 This test is .docxCell Physiology Spring 2016 Name Page 1 This test is .docx
Cell Physiology Spring 2016 Name Page 1 This test is .docx
 
Ganong-Physiology-Compilation.pdf
Ganong-Physiology-Compilation.pdfGanong-Physiology-Compilation.pdf
Ganong-Physiology-Compilation.pdf
 
Deciphering Cellular Respiration: Stages, Mechanisms, and Implications | The ...
Deciphering Cellular Respiration: Stages, Mechanisms, and Implications | The ...Deciphering Cellular Respiration: Stages, Mechanisms, and Implications | The ...
Deciphering Cellular Respiration: Stages, Mechanisms, and Implications | The ...
 
Cell structure and function re.pdf
Cell structure and function re.pdfCell structure and function re.pdf
Cell structure and function re.pdf
 
Biology 204 Principles of Biology I Assignment 1C
Biology 204 Principles of Biology I Assignment 1CBiology 204 Principles of Biology I Assignment 1C
Biology 204 Principles of Biology I Assignment 1C
 
SELF-TEST Amswi each af the follimins suesions by reiecting the ene an.pdf
SELF-TEST Amswi each af the follimins suesions by reiecting the ene an.pdfSELF-TEST Amswi each af the follimins suesions by reiecting the ene an.pdf
SELF-TEST Amswi each af the follimins suesions by reiecting the ene an.pdf
 
LF & PPG- FINAL EXAM, TOS, & SUMMATIVE TEST.docx
LF & PPG- FINAL EXAM, TOS, & SUMMATIVE TEST.docxLF & PPG- FINAL EXAM, TOS, & SUMMATIVE TEST.docx
LF & PPG- FINAL EXAM, TOS, & SUMMATIVE TEST.docx
 
Cell Physiology Basics
Cell Physiology BasicsCell Physiology Basics
Cell Physiology Basics
 
Ib ch 1 question-answer
Ib ch 1 question-answerIb ch 1 question-answer
Ib ch 1 question-answer
 
ECOSYSTEM-Life-Energy.pptx
ECOSYSTEM-Life-Energy.pptxECOSYSTEM-Life-Energy.pptx
ECOSYSTEM-Life-Energy.pptx
 
test bank Lehninger Principles of Biochemistry, 8e David Nelson, Michael Cox ...
test bank Lehninger Principles of Biochemistry, 8e David Nelson, Michael Cox ...test bank Lehninger Principles of Biochemistry, 8e David Nelson, Michael Cox ...
test bank Lehninger Principles of Biochemistry, 8e David Nelson, Michael Cox ...
 
Tutorial mitosis
Tutorial mitosisTutorial mitosis
Tutorial mitosis
 
Assignment On Bioenergetics
Assignment On BioenergeticsAssignment On Bioenergetics
Assignment On Bioenergetics
 
9.1 Cellular Respiration_ An Overview.pdf
9.1 Cellular Respiration_ An Overview.pdf9.1 Cellular Respiration_ An Overview.pdf
9.1 Cellular Respiration_ An Overview.pdf
 
Cellular Respiration Essay
Cellular Respiration EssayCellular Respiration Essay
Cellular Respiration Essay
 
2. Absorption & Secretion Of Materials
2. Absorption & Secretion Of Materials2. Absorption & Secretion Of Materials
2. Absorption & Secretion Of Materials
 

More from Donc Test

Libby/Libby/Hodge Financial Accounting 11th Edition TEST BANK & SOLUTION MANU...
Libby/Libby/Hodge Financial Accounting 11th Edition TEST BANK & SOLUTION MANU...Libby/Libby/Hodge Financial Accounting 11th Edition TEST BANK & SOLUTION MANU...
Libby/Libby/Hodge Financial Accounting 11th Edition TEST BANK & SOLUTION MANU...
Donc Test
 
Solution Manual For Financial Accounting, 8th Canadian Edition 2024, by Libby...
Solution Manual For Financial Accounting, 8th Canadian Edition 2024, by Libby...Solution Manual For Financial Accounting, 8th Canadian Edition 2024, by Libby...
Solution Manual For Financial Accounting, 8th Canadian Edition 2024, by Libby...
Donc Test
 
TEST BANK For Community and Public Health Nursing: Evidence for Practice, 4th...
TEST BANK For Community and Public Health Nursing: Evidence for Practice, 4th...TEST BANK For Community and Public Health Nursing: Evidence for Practice, 4th...
TEST BANK For Community and Public Health Nursing: Evidence for Practice, 4th...
Donc Test
 
TEST BANK For Community and Public Health Nursing: Evidence for Practice, 3rd...
TEST BANK For Community and Public Health Nursing: Evidence for Practice, 3rd...TEST BANK For Community and Public Health Nursing: Evidence for Practice, 3rd...
TEST BANK For Community and Public Health Nursing: Evidence for Practice, 3rd...
Donc Test
 
TEST BANK For Community Health Nursing A Canadian Perspective, 5th Edition by...
TEST BANK For Community Health Nursing A Canadian Perspective, 5th Edition by...TEST BANK For Community Health Nursing A Canadian Perspective, 5th Edition by...
TEST BANK For Community Health Nursing A Canadian Perspective, 5th Edition by...
Donc Test
 
TEST BANK For CURRENT Diagnosis & Treatment Pediatrics, 26th Edition by Maya ...
TEST BANK For CURRENT Diagnosis & Treatment Pediatrics, 26th Edition by Maya ...TEST BANK For CURRENT Diagnosis & Treatment Pediatrics, 26th Edition by Maya ...
TEST BANK For CURRENT Diagnosis & Treatment Pediatrics, 26th Edition by Maya ...
Donc Test
 
TEST BANK For Current Medical Diagnosis And Treatment 2024, 63rd Edition By M...
TEST BANK For Current Medical Diagnosis And Treatment 2024, 63rd Edition By M...TEST BANK For Current Medical Diagnosis And Treatment 2024, 63rd Edition By M...
TEST BANK For Current Medical Diagnosis And Treatment 2024, 63rd Edition By M...
Donc Test
 
TEST BANK For Davis Advantage for Pathophysiology Introductory Concepts and C...
TEST BANK For Davis Advantage for Pathophysiology Introductory Concepts and C...TEST BANK For Davis Advantage for Pathophysiology Introductory Concepts and C...
TEST BANK For Davis Advantage for Pathophysiology Introductory Concepts and C...
Donc Test
 
TEST BANK For Discovering the Life Span, 5th Edition Robert S. Feldman, Verif...
TEST BANK For Discovering the Life Span, 5th Edition Robert S. Feldman, Verif...TEST BANK For Discovering the Life Span, 5th Edition Robert S. Feldman, Verif...
TEST BANK For Discovering the Life Span, 5th Edition Robert S. Feldman, Verif...
Donc Test
 
Test Bank - Karch Focus on Nursing Pharmacology 9th Edition by Rebecca Tucker...
Test Bank - Karch Focus on Nursing Pharmacology 9th Edition by Rebecca Tucker...Test Bank - Karch Focus on Nursing Pharmacology 9th Edition by Rebecca Tucker...
Test Bank - Karch Focus on Nursing Pharmacology 9th Edition by Rebecca Tucker...
Donc Test
 
Test Bank For Clinical Nursing Skills and Techniques 10th Edition (1).pdf
Test Bank For Clinical Nursing Skills and Techniques 10th Edition (1).pdfTest Bank For Clinical Nursing Skills and Techniques 10th Edition (1).pdf
Test Bank For Clinical Nursing Skills and Techniques 10th Edition (1).pdf
Donc Test
 
Test Bank for Anatomy of Oriented Structure 8th edition.pdf
Test Bank for Anatomy of Oriented Structure 8th edition.pdfTest Bank for Anatomy of Oriented Structure 8th edition.pdf
Test Bank for Anatomy of Oriented Structure 8th edition.pdf
Donc Test
 
TEST BANK Essentials of dental radiography 9th edition by Evelyn Thomson, Orl...
TEST BANK Essentials of dental radiography 9th edition by Evelyn Thomson, Orl...TEST BANK Essentials of dental radiography 9th edition by Evelyn Thomson, Orl...
TEST BANK Essentials of dental radiography 9th edition by Evelyn Thomson, Orl...
Donc Test
 
Test bank clinical nursing skills a concept based approach 4e pearson educati...
Test bank clinical nursing skills a concept based approach 4e pearson educati...Test bank clinical nursing skills a concept based approach 4e pearson educati...
Test bank clinical nursing skills a concept based approach 4e pearson educati...
Donc Test
 
TEST BANK For, Information Technology Project Management 9th Edition Kathy Sc...
TEST BANK For, Information Technology Project Management 9th Edition Kathy Sc...TEST BANK For, Information Technology Project Management 9th Edition Kathy Sc...
TEST BANK For, Information Technology Project Management 9th Edition Kathy Sc...
Donc Test
 
TEST BANK For Wong's Nursing Care of Infants and Children, 11th Edition by Ma...
TEST BANK For Wong's Nursing Care of Infants and Children, 11th Edition by Ma...TEST BANK For Wong's Nursing Care of Infants and Children, 11th Edition by Ma...
TEST BANK For Wong's Nursing Care of Infants and Children, 11th Edition by Ma...
Donc Test
 
TEST BANK for Wilkins’ Clinical Assessment in Respiratory Care, 9th Edition b...
TEST BANK for Wilkins’ Clinical Assessment in Respiratory Care, 9th Edition b...TEST BANK for Wilkins’ Clinical Assessment in Respiratory Care, 9th Edition b...
TEST BANK for Wilkins’ Clinical Assessment in Respiratory Care, 9th Edition b...
Donc Test
 
TEST BANK For Understanding the Essentials of Critical Care Nursing, 3rd Edit...
TEST BANK For Understanding the Essentials of Critical Care Nursing, 3rd Edit...TEST BANK For Understanding the Essentials of Critical Care Nursing, 3rd Edit...
TEST BANK For Understanding the Essentials of Critical Care Nursing, 3rd Edit...
Donc Test
 
TEST BANK For Timby's Introductory Medical-Surgical Nursing, 13th Edition by ...
TEST BANK For Timby's Introductory Medical-Surgical Nursing, 13th Edition by ...TEST BANK For Timby's Introductory Medical-Surgical Nursing, 13th Edition by ...
TEST BANK For Timby's Introductory Medical-Surgical Nursing, 13th Edition by ...
Donc Test
 
TEST BANK for The Nursing Assistant Acute, Subacute, and Long-Term Care, 6th ...
TEST BANK for The Nursing Assistant Acute, Subacute, and Long-Term Care, 6th ...TEST BANK for The Nursing Assistant Acute, Subacute, and Long-Term Care, 6th ...
TEST BANK for The Nursing Assistant Acute, Subacute, and Long-Term Care, 6th ...
Donc Test
 

More from Donc Test (20)

Libby/Libby/Hodge Financial Accounting 11th Edition TEST BANK & SOLUTION MANU...
Libby/Libby/Hodge Financial Accounting 11th Edition TEST BANK & SOLUTION MANU...Libby/Libby/Hodge Financial Accounting 11th Edition TEST BANK & SOLUTION MANU...
Libby/Libby/Hodge Financial Accounting 11th Edition TEST BANK & SOLUTION MANU...
 
Solution Manual For Financial Accounting, 8th Canadian Edition 2024, by Libby...
Solution Manual For Financial Accounting, 8th Canadian Edition 2024, by Libby...Solution Manual For Financial Accounting, 8th Canadian Edition 2024, by Libby...
Solution Manual For Financial Accounting, 8th Canadian Edition 2024, by Libby...
 
TEST BANK For Community and Public Health Nursing: Evidence for Practice, 4th...
TEST BANK For Community and Public Health Nursing: Evidence for Practice, 4th...TEST BANK For Community and Public Health Nursing: Evidence for Practice, 4th...
TEST BANK For Community and Public Health Nursing: Evidence for Practice, 4th...
 
TEST BANK For Community and Public Health Nursing: Evidence for Practice, 3rd...
TEST BANK For Community and Public Health Nursing: Evidence for Practice, 3rd...TEST BANK For Community and Public Health Nursing: Evidence for Practice, 3rd...
TEST BANK For Community and Public Health Nursing: Evidence for Practice, 3rd...
 
TEST BANK For Community Health Nursing A Canadian Perspective, 5th Edition by...
TEST BANK For Community Health Nursing A Canadian Perspective, 5th Edition by...TEST BANK For Community Health Nursing A Canadian Perspective, 5th Edition by...
TEST BANK For Community Health Nursing A Canadian Perspective, 5th Edition by...
 
TEST BANK For CURRENT Diagnosis & Treatment Pediatrics, 26th Edition by Maya ...
TEST BANK For CURRENT Diagnosis & Treatment Pediatrics, 26th Edition by Maya ...TEST BANK For CURRENT Diagnosis & Treatment Pediatrics, 26th Edition by Maya ...
TEST BANK For CURRENT Diagnosis & Treatment Pediatrics, 26th Edition by Maya ...
 
TEST BANK For Current Medical Diagnosis And Treatment 2024, 63rd Edition By M...
TEST BANK For Current Medical Diagnosis And Treatment 2024, 63rd Edition By M...TEST BANK For Current Medical Diagnosis And Treatment 2024, 63rd Edition By M...
TEST BANK For Current Medical Diagnosis And Treatment 2024, 63rd Edition By M...
 
TEST BANK For Davis Advantage for Pathophysiology Introductory Concepts and C...
TEST BANK For Davis Advantage for Pathophysiology Introductory Concepts and C...TEST BANK For Davis Advantage for Pathophysiology Introductory Concepts and C...
TEST BANK For Davis Advantage for Pathophysiology Introductory Concepts and C...
 
TEST BANK For Discovering the Life Span, 5th Edition Robert S. Feldman, Verif...
TEST BANK For Discovering the Life Span, 5th Edition Robert S. Feldman, Verif...TEST BANK For Discovering the Life Span, 5th Edition Robert S. Feldman, Verif...
TEST BANK For Discovering the Life Span, 5th Edition Robert S. Feldman, Verif...
 
Test Bank - Karch Focus on Nursing Pharmacology 9th Edition by Rebecca Tucker...
Test Bank - Karch Focus on Nursing Pharmacology 9th Edition by Rebecca Tucker...Test Bank - Karch Focus on Nursing Pharmacology 9th Edition by Rebecca Tucker...
Test Bank - Karch Focus on Nursing Pharmacology 9th Edition by Rebecca Tucker...
 
Test Bank For Clinical Nursing Skills and Techniques 10th Edition (1).pdf
Test Bank For Clinical Nursing Skills and Techniques 10th Edition (1).pdfTest Bank For Clinical Nursing Skills and Techniques 10th Edition (1).pdf
Test Bank For Clinical Nursing Skills and Techniques 10th Edition (1).pdf
 
Test Bank for Anatomy of Oriented Structure 8th edition.pdf
Test Bank for Anatomy of Oriented Structure 8th edition.pdfTest Bank for Anatomy of Oriented Structure 8th edition.pdf
Test Bank for Anatomy of Oriented Structure 8th edition.pdf
 
TEST BANK Essentials of dental radiography 9th edition by Evelyn Thomson, Orl...
TEST BANK Essentials of dental radiography 9th edition by Evelyn Thomson, Orl...TEST BANK Essentials of dental radiography 9th edition by Evelyn Thomson, Orl...
TEST BANK Essentials of dental radiography 9th edition by Evelyn Thomson, Orl...
 
Test bank clinical nursing skills a concept based approach 4e pearson educati...
Test bank clinical nursing skills a concept based approach 4e pearson educati...Test bank clinical nursing skills a concept based approach 4e pearson educati...
Test bank clinical nursing skills a concept based approach 4e pearson educati...
 
TEST BANK For, Information Technology Project Management 9th Edition Kathy Sc...
TEST BANK For, Information Technology Project Management 9th Edition Kathy Sc...TEST BANK For, Information Technology Project Management 9th Edition Kathy Sc...
TEST BANK For, Information Technology Project Management 9th Edition Kathy Sc...
 
TEST BANK For Wong's Nursing Care of Infants and Children, 11th Edition by Ma...
TEST BANK For Wong's Nursing Care of Infants and Children, 11th Edition by Ma...TEST BANK For Wong's Nursing Care of Infants and Children, 11th Edition by Ma...
TEST BANK For Wong's Nursing Care of Infants and Children, 11th Edition by Ma...
 
TEST BANK for Wilkins’ Clinical Assessment in Respiratory Care, 9th Edition b...
TEST BANK for Wilkins’ Clinical Assessment in Respiratory Care, 9th Edition b...TEST BANK for Wilkins’ Clinical Assessment in Respiratory Care, 9th Edition b...
TEST BANK for Wilkins’ Clinical Assessment in Respiratory Care, 9th Edition b...
 
TEST BANK For Understanding the Essentials of Critical Care Nursing, 3rd Edit...
TEST BANK For Understanding the Essentials of Critical Care Nursing, 3rd Edit...TEST BANK For Understanding the Essentials of Critical Care Nursing, 3rd Edit...
TEST BANK For Understanding the Essentials of Critical Care Nursing, 3rd Edit...
 
TEST BANK For Timby's Introductory Medical-Surgical Nursing, 13th Edition by ...
TEST BANK For Timby's Introductory Medical-Surgical Nursing, 13th Edition by ...TEST BANK For Timby's Introductory Medical-Surgical Nursing, 13th Edition by ...
TEST BANK For Timby's Introductory Medical-Surgical Nursing, 13th Edition by ...
 
TEST BANK for The Nursing Assistant Acute, Subacute, and Long-Term Care, 6th ...
TEST BANK for The Nursing Assistant Acute, Subacute, and Long-Term Care, 6th ...TEST BANK for The Nursing Assistant Acute, Subacute, and Long-Term Care, 6th ...
TEST BANK for The Nursing Assistant Acute, Subacute, and Long-Term Care, 6th ...
 

Recently uploaded

A Classical Text Review on Basavarajeeyam
A Classical Text Review on BasavarajeeyamA Classical Text Review on Basavarajeeyam
A Classical Text Review on Basavarajeeyam
Dr. Jyothirmai Paindla
 
The Electrocardiogram - Physiologic Principles
The Electrocardiogram - Physiologic PrinciplesThe Electrocardiogram - Physiologic Principles
The Electrocardiogram - Physiologic Principles
MedicoseAcademics
 
Pictures of Superficial & Deep Fascia.ppt.pdf
Pictures of Superficial & Deep Fascia.ppt.pdfPictures of Superficial & Deep Fascia.ppt.pdf
Pictures of Superficial & Deep Fascia.ppt.pdf
Dr. Rabia Inam Gandapore
 
How STIs Influence the Development of Pelvic Inflammatory Disease.pptx
How STIs Influence the Development of Pelvic Inflammatory Disease.pptxHow STIs Influence the Development of Pelvic Inflammatory Disease.pptx
How STIs Influence the Development of Pelvic Inflammatory Disease.pptx
FFragrant
 
Adv. biopharm. APPLICATION OF PHARMACOKINETICS : TARGETED DRUG DELIVERY SYSTEMS
Adv. biopharm. APPLICATION OF PHARMACOKINETICS : TARGETED DRUG DELIVERY SYSTEMSAdv. biopharm. APPLICATION OF PHARMACOKINETICS : TARGETED DRUG DELIVERY SYSTEMS
Adv. biopharm. APPLICATION OF PHARMACOKINETICS : TARGETED DRUG DELIVERY SYSTEMS
AkankshaAshtankar
 
Superficial & Deep Fascia of the NECK.pptx
Superficial & Deep Fascia of the NECK.pptxSuperficial & Deep Fascia of the NECK.pptx
Superficial & Deep Fascia of the NECK.pptx
Dr. Rabia Inam Gandapore
 
SURGICAL ANATOMY OF THE RETROPERITONEUM, ADRENALS, KIDNEYS AND URETERS.pptx
SURGICAL ANATOMY OF THE RETROPERITONEUM, ADRENALS, KIDNEYS AND URETERS.pptxSURGICAL ANATOMY OF THE RETROPERITONEUM, ADRENALS, KIDNEYS AND URETERS.pptx
SURGICAL ANATOMY OF THE RETROPERITONEUM, ADRENALS, KIDNEYS AND URETERS.pptx
Bright Chipili
 
Basavarajeeyam - Ayurvedic heritage book of Andhra pradesh
Basavarajeeyam - Ayurvedic heritage book of Andhra pradeshBasavarajeeyam - Ayurvedic heritage book of Andhra pradesh
Basavarajeeyam - Ayurvedic heritage book of Andhra pradesh
Dr. Madduru Muni Haritha
 
KDIGO 2024 guidelines for diabetologists
KDIGO 2024 guidelines for diabetologistsKDIGO 2024 guidelines for diabetologists
KDIGO 2024 guidelines for diabetologists
د.محمود نجيب
 
Ophthalmology Clinical Tests for OSCE exam
Ophthalmology Clinical Tests for OSCE examOphthalmology Clinical Tests for OSCE exam
Ophthalmology Clinical Tests for OSCE exam
KafrELShiekh University
 
Knee anatomy and clinical tests 2024.pdf
Knee anatomy and clinical tests 2024.pdfKnee anatomy and clinical tests 2024.pdf
Knee anatomy and clinical tests 2024.pdf
vimalpl1234
 
Top-Vitamin-Supplement-Brands-in-India.pptx
Top-Vitamin-Supplement-Brands-in-India.pptxTop-Vitamin-Supplement-Brands-in-India.pptx
Top-Vitamin-Supplement-Brands-in-India.pptx
SwisschemDerma
 
CDSCO and Phamacovigilance {Regulatory body in India}
CDSCO and Phamacovigilance {Regulatory body in India}CDSCO and Phamacovigilance {Regulatory body in India}
CDSCO and Phamacovigilance {Regulatory body in India}
NEHA GUPTA
 
New Drug Discovery and Development .....
New Drug Discovery and Development .....New Drug Discovery and Development .....
New Drug Discovery and Development .....
NEHA GUPTA
 
Novas diretrizes da OMS para os cuidados perinatais de mais qualidade
Novas diretrizes da OMS para os cuidados perinatais de mais qualidadeNovas diretrizes da OMS para os cuidados perinatais de mais qualidade
Novas diretrizes da OMS para os cuidados perinatais de mais qualidade
Prof. Marcus Renato de Carvalho
 
Vision-1.pptx, Eye structure, basics of optics
Vision-1.pptx, Eye structure, basics of opticsVision-1.pptx, Eye structure, basics of optics
Vision-1.pptx, Eye structure, basics of optics
Sai Sailesh Kumar Goothy
 
Ocular injury ppt Upendra pal optometrist upums saifai etawah
Ocular injury  ppt  Upendra pal  optometrist upums saifai etawahOcular injury  ppt  Upendra pal  optometrist upums saifai etawah
Ocular injury ppt Upendra pal optometrist upums saifai etawah
pal078100
 
NVBDCP.pptx Nation vector borne disease control program
NVBDCP.pptx Nation vector borne disease control programNVBDCP.pptx Nation vector borne disease control program
NVBDCP.pptx Nation vector borne disease control program
Sapna Thakur
 
Hemodialysis: Chapter 3, Dialysis Water Unit - Dr.Gawad
Hemodialysis: Chapter 3, Dialysis Water Unit - Dr.GawadHemodialysis: Chapter 3, Dialysis Water Unit - Dr.Gawad
Hemodialysis: Chapter 3, Dialysis Water Unit - Dr.Gawad
NephroTube - Dr.Gawad
 
Flu Vaccine Alert in Bangalore Karnataka
Flu Vaccine Alert in Bangalore KarnatakaFlu Vaccine Alert in Bangalore Karnataka
Flu Vaccine Alert in Bangalore Karnataka
addon Scans
 

Recently uploaded (20)

A Classical Text Review on Basavarajeeyam
A Classical Text Review on BasavarajeeyamA Classical Text Review on Basavarajeeyam
A Classical Text Review on Basavarajeeyam
 
The Electrocardiogram - Physiologic Principles
The Electrocardiogram - Physiologic PrinciplesThe Electrocardiogram - Physiologic Principles
The Electrocardiogram - Physiologic Principles
 
Pictures of Superficial & Deep Fascia.ppt.pdf
Pictures of Superficial & Deep Fascia.ppt.pdfPictures of Superficial & Deep Fascia.ppt.pdf
Pictures of Superficial & Deep Fascia.ppt.pdf
 
How STIs Influence the Development of Pelvic Inflammatory Disease.pptx
How STIs Influence the Development of Pelvic Inflammatory Disease.pptxHow STIs Influence the Development of Pelvic Inflammatory Disease.pptx
How STIs Influence the Development of Pelvic Inflammatory Disease.pptx
 
Adv. biopharm. APPLICATION OF PHARMACOKINETICS : TARGETED DRUG DELIVERY SYSTEMS
Adv. biopharm. APPLICATION OF PHARMACOKINETICS : TARGETED DRUG DELIVERY SYSTEMSAdv. biopharm. APPLICATION OF PHARMACOKINETICS : TARGETED DRUG DELIVERY SYSTEMS
Adv. biopharm. APPLICATION OF PHARMACOKINETICS : TARGETED DRUG DELIVERY SYSTEMS
 
Superficial & Deep Fascia of the NECK.pptx
Superficial & Deep Fascia of the NECK.pptxSuperficial & Deep Fascia of the NECK.pptx
Superficial & Deep Fascia of the NECK.pptx
 
SURGICAL ANATOMY OF THE RETROPERITONEUM, ADRENALS, KIDNEYS AND URETERS.pptx
SURGICAL ANATOMY OF THE RETROPERITONEUM, ADRENALS, KIDNEYS AND URETERS.pptxSURGICAL ANATOMY OF THE RETROPERITONEUM, ADRENALS, KIDNEYS AND URETERS.pptx
SURGICAL ANATOMY OF THE RETROPERITONEUM, ADRENALS, KIDNEYS AND URETERS.pptx
 
Basavarajeeyam - Ayurvedic heritage book of Andhra pradesh
Basavarajeeyam - Ayurvedic heritage book of Andhra pradeshBasavarajeeyam - Ayurvedic heritage book of Andhra pradesh
Basavarajeeyam - Ayurvedic heritage book of Andhra pradesh
 
KDIGO 2024 guidelines for diabetologists
KDIGO 2024 guidelines for diabetologistsKDIGO 2024 guidelines for diabetologists
KDIGO 2024 guidelines for diabetologists
 
Ophthalmology Clinical Tests for OSCE exam
Ophthalmology Clinical Tests for OSCE examOphthalmology Clinical Tests for OSCE exam
Ophthalmology Clinical Tests for OSCE exam
 
Knee anatomy and clinical tests 2024.pdf
Knee anatomy and clinical tests 2024.pdfKnee anatomy and clinical tests 2024.pdf
Knee anatomy and clinical tests 2024.pdf
 
Top-Vitamin-Supplement-Brands-in-India.pptx
Top-Vitamin-Supplement-Brands-in-India.pptxTop-Vitamin-Supplement-Brands-in-India.pptx
Top-Vitamin-Supplement-Brands-in-India.pptx
 
CDSCO and Phamacovigilance {Regulatory body in India}
CDSCO and Phamacovigilance {Regulatory body in India}CDSCO and Phamacovigilance {Regulatory body in India}
CDSCO and Phamacovigilance {Regulatory body in India}
 
New Drug Discovery and Development .....
New Drug Discovery and Development .....New Drug Discovery and Development .....
New Drug Discovery and Development .....
 
Novas diretrizes da OMS para os cuidados perinatais de mais qualidade
Novas diretrizes da OMS para os cuidados perinatais de mais qualidadeNovas diretrizes da OMS para os cuidados perinatais de mais qualidade
Novas diretrizes da OMS para os cuidados perinatais de mais qualidade
 
Vision-1.pptx, Eye structure, basics of optics
Vision-1.pptx, Eye structure, basics of opticsVision-1.pptx, Eye structure, basics of optics
Vision-1.pptx, Eye structure, basics of optics
 
Ocular injury ppt Upendra pal optometrist upums saifai etawah
Ocular injury  ppt  Upendra pal  optometrist upums saifai etawahOcular injury  ppt  Upendra pal  optometrist upums saifai etawah
Ocular injury ppt Upendra pal optometrist upums saifai etawah
 
NVBDCP.pptx Nation vector borne disease control program
NVBDCP.pptx Nation vector borne disease control programNVBDCP.pptx Nation vector borne disease control program
NVBDCP.pptx Nation vector borne disease control program
 
Hemodialysis: Chapter 3, Dialysis Water Unit - Dr.Gawad
Hemodialysis: Chapter 3, Dialysis Water Unit - Dr.GawadHemodialysis: Chapter 3, Dialysis Water Unit - Dr.Gawad
Hemodialysis: Chapter 3, Dialysis Water Unit - Dr.Gawad
 
Flu Vaccine Alert in Bangalore Karnataka
Flu Vaccine Alert in Bangalore KarnatakaFlu Vaccine Alert in Bangalore Karnataka
Flu Vaccine Alert in Bangalore Karnataka
 

TEST BANK For Huether and McCance's Understanding Pathophysiology, Canadian 2nd Edition.pdf

  • 1. TO GET ALL CHAPTERS EMAIL ME AT>>>>> donc8246@gmail.com Test Bank for Huether and McCance's Understanding Pathophysiology, Canadian Edition, 2nd Edition by Kelly Power-Kean, Chapter 1 - 42
  • 2. TO GET ALL CHAPTERS EMAIL ME AT>>>>> donc8246@gmail.com Table of Contents PART ONE: BASIC CONCEPTS OF PATHOPHYSIOLOGY Unit 1: The Cell 1. Cellular Biology 2. Genes and Genetic Diseases 3. Epigenetics and Disease 4. Altered Cellular and Tissue Biology 5. Fluids and Electrolytes, Acids and Bases Unit 2: Mechanisms of Self-Defense 6. Innate Immunity: Inflammation and Wound Healing 7. Adaptive Immunity 8. Infection and Defects in Mechanisms of Defense 9. Stress and Disease Unit 3: Cellular Proliferation: Cancer 10. Biology of Cancer 11. Cancer Epidemiology 12. Cancer in Children and Adolescents PART TWO: BODY SYSTEMS AND DISEASES Unit 4: The Neurologic System 13. Structure and Function of the Neurologic System 14. Pain, Temperature, Sleep, and Sensory Function 15. Alterations in Cognitive Systems, Cerebral Hemodynamics and Motor Function 16. Disorders of the Central and Peripheral Nervous Systems and Neuromuscular Junction 17. Alterations of Neurologic Function in Children Unit 5: The Endocrine System 18. Mechanisms of Hormonal Regulation 19. Alterations of Hormonal Regulation Unit 6: The Hematologic System 20. Structure and Function of the Hematologic System 21. Alterations in Hematologic Function 22. Alterations of Hematologic Function in Children Unit 7: The Cardiovascular and Lymphatic Systems 23. Structure and Function of the Cardiovascular and Lymphatic Systems 24. Alterations of Cardiovascular Function 25. Alterations of Cardiovascular Function in Children Unit 8: The Pulmonary System 26. Structure and Function of the Pulmonary System 27. Alterations of Pulmonary Function 28. Alterations of Pulmonary Function in Children Unit 9: The Renal and Urologic Systems 29. Structure and Function of the Renal and Urologic Systems 30. Alterations of Renal and Urinary Tract Function 31. Alterations of Renal and Urinary Tract Function in Children
  • 3. TO GET ALL CHAPTERS EMAIL ME AT>>>>> donc8246@gmail.com Unit 10: The Reproductive Systems 32. Structure and Function of the Reproductive Systems 33. Alterations of the Female Reproductive System 34. Alterations of the Male Reproductive System Unit 11: The Digestive System 35. Structure and Function of the Digestive System 36. Alterations of Digestive Function 37. Alterations in Digestive Function in Children Unit 12: The Musculoskeletal and Integumentary Systems 38. Structure and Function of the Musculoskeletal System 39. Alterations of Musculoskeletal Function 40. Alterations of Musculoskeletal Function in Children 41. Structure, Function, and Disorders of the Integument 42. Alterations of the Integument in Children
  • 4. NURSINGTB.COM TO GET ALL CHAPTERS EMAIL ME AT>>>>> donc8246@gmail.com Chapter 01: Cellular Biology MULTIPLE CHOICE 1. A student is observing a cell under the microscope. It is observed to have supercoiled DNA with histones. Which of the following would also be observed by the student? a. A single circular chromosome b. A nucleus c. Free-floating nuclear material d. No organelles ANS: B The cell described is a eukaryotic cell, so it has histones and a supercoiled DNA within its nucleus; thus, the nucleus should be observed. A single circular chromosome called a prokaryote contains free-floating nuclear material but has no organelles. REF: p. 2 2. A nurse is instructing the staff about cellular functions. Which cellular function is the nurse describing when an isolated cell absorbs oxygen and uses it to transform nutrients to energy? a. Metabolic absorption b. Communication c. Secretion d. Respiration ANS: D NURSINGTB.COM The cell’s ability to absorb oxygen is referred to as respiration while its communication ability involves maintenance of a steady dynamic state, metabolic absorption provides nutrition, and secretion allows for the synthesizing of new substances. REF: p. 2 3. A eukaryotic cell is undergoing DNA replication. In which region of the cell would most of the genetic information be contained? a. Mitochondria b. Ribosome c. Nucleolus d. Nucleus Cytoplasm ANS: C The region of the cell that contains genetic material, including a large amount of ribonucleic acid, most of the DNA, and DNA-binding proteins, is the nucleolus, which is located within the cell’s nucleus. Mitochondria is associated with cellular respiration, while ribosomes are involved with protein manufacturing. Cytoplasm is a fluid filling that is a component of the cell. REF: p. 2
  • 5. NURSINGTB.COM TO GET ALL CHAPTERS EMAIL ME AT>>>>> donc8246@gmail.com U S N 4. Which of the following can remove proteins attached to the cell’s bilayer by dissolving the layer itself? a. Peripheral membrane proteins b. Integral membrane proteins c. Glycoproteins d. Cell adhesion molecules ANS: B Proteins directly attached to the membrane bilayer can be removed by the action of integral membrane proteins that dissolve the bilayer. Peripheral membrane proteins reside at the surface while cell adhesion molecules are on the outside of the membrane. Glycoprotein marks cells and does not float. REF: p. 7 5. Which of the following can bind to plasma membrane receptors? a. Oxygen b. Ribosomes c. Amphipathic lipids d. Ligands ANS: D Ligands are the only specific molecules that can bind with receptors on the cell membrane. REF: p. 9 6. A nurse is reviewing a report from a patient with metastatic cancer. What alternation in the extracellular matrix would suN ppoR rt t h Ie dG iagn B o. siC s ofM metastatic cancer? a. Decreased fibronectin b. Increased collagen c. Decreased elastin d. Increased glycoproteins ANS: A Only a reduced amount of fibronectin is found in some types of cancerous cells, allowing them to travel or metastasize. REF: p. 10 7. Which form of cell communication is used to relate to other cells in direct physical contact? a. Cell junction b. Gap junction c. Desmosome d. Tight junction ANS: A Cell junctions hold cells together and permit molecules to pass from cell to cell. Gap junctions allow for cellular communication between cells. Neither desmosomes nor tight junctions are associated with cellular communication. REF: p. 11
  • 6. NURSINGTB.COM TO GET ALL CHAPTERS EMAIL ME AT>>>>> donc8246@gmail.com 8. Pancreatic beta cells secrete insulin, which inhibits secretion of glucagon from neighboring alpha cells. This action is an example of which of the following signaling types? a. Paracrine b. Autocrine c. Neurohormonal d. Hormonal ANS: A Paracrine signaling involves the release of local chemical mediators that are quickly taken up, destroyed, or immobilized, as in the case of insulin and the inhibition of the secretion of glucagon. None of the other options involve signaling that is associated with a local chemical mediator like insulin. REF: p. 12 9. In cellular metabolism, each enzyme has a high affinity for a: a. solute. b. substrate. c. receptor. d. ribosome. ANS: B Each enzyme has a high affinity for a substrate, a specific substance converted to a product of the reaction. Cellular metabolism is not dependent on an attraction between an enzyme and any of the remaining options. REF: p. 16 NURSINGTB.COM 10. An athlete runs a marathon, after which his muscles feel fatigued and unable to contract. The athlete asks the nurse why this happened. The nurse’s response is based on the knowledge that the problem is result of a deficiency of: a. GTP b. AMP c. ATP d. GMP ANS: C When ATP is deficient, impaired muscle contraction results. None of the other options are involved in muscle contraction. REF: p. 16 11. Which phase of catabolism produces the most ATP? a. Digestion b. Glycolysis c. Oxidation d. Citric acid cycle ANS: D While some ATP is produced during the oxidation and glycolysis phases, most of the ATP is generated during the citric acid cycle. Digestion does not produce any ATP.
  • 7. NURSINGTB.COM TO GET ALL CHAPTERS EMAIL ME AT>>>>> donc8246@gmail.com REF: p. 16 12. A nurse is teaching the staff about the phases of cellular catabolism. Which phases should the nurse include? a. Digestion, glycolysis, oxidation, and the citric acid cycle b. Diffusion, osmosis, and mediated transport c. S phase, G phase, and M phase d. Metabolic absorption, respiration, and excretion ANS: A Only digestion, glycolysis, oxidation, and the citric acid cycle are the phases of cellular catabolism. REF: p. 16 13. A runner has depleted all the oxygen available for muscle energy. Which of the following will facilitate his continued muscle performance? a. Electron-transport chain b. Aerobic glycolysis c. Anaerobic glycolysis d. Oxidative phosphorylation ANS: C When no oxygen is available, anaerobic glycolysis occurs. The electron-transport chain is part of the citric acid cycle. Aerobic glycolysis involves the presence of oxygen. Oxidative phosphorylation is the mechanism by which the energy produced from carbohydrates, fats, and proteins is transferred to ATP. It is not part of muscle performance. REF: p. 16 NURSINGTB.COM 14. A faculty member asks a student to identify the appropriate term for the movement of a solute from an area of greater to lesser concentration. Which answer indicates the nursing student understood the teaching? a. Osmosis b. Diffusion c. Hydrostatic pressure d. Active transport ANS: B Diffusion is the movement of a solute molecule from an area of greater solute concentration to an area of lesser solute concentration through a permeable membrane. Osmosis is the movement of water across a semipermeable membrane from a region of higher water concentration to one of lower concentration. Hydrostatic pressure is the force of fluid against a cell membrane. In active transport, molecules move up a concentration gradient. REF: p. 19 15. Which description accurately describes electrolytes? a. Small lipid-soluble molecules b. Large protein molecules c. Micronutrients used to produce ATP
  • 8. NURSINGTB.COM TO GET ALL CHAPTERS EMAIL ME AT>>>>> donc8246@gmail.com d. Electrically charged molecules ANS: D Electrolytes are electrically charged molecules. They are not lipid soluble, they are not made up of protein, and they do not play a role in ATP production. REF: p. 18 16. A nurse is reading a chart and sees the term oncotic pressure. The nurse recalls that oncotic pressure (colloid osmotic pressure) is determined by: a. the concentration of sodium. b. plasma proteins. c. hydrostatic pressure. d. the availability of membrane transporter proteins. ANS: B Oncotic pressure is determined by the effect of colloids or plasma proteins. The concentration of sodium plays a role in tonicity. Hydrostatic pressure is the force within a vessel. Membrane transporter proteins are involved in active transport within a concentration gradient. REF: p. 20 17. A patient has a body fluid of 300 mOsm/kg. This lab result is measuring: a. osmolality. b. osmolarity. c. osmotic pressure. d. oncotic pressure. ANS: A NURSINGTB.COM Osmolality measures the number of milliosmoles per kilogram of water, or the concentration of molecules per weight of water, while osmolarity measures the number of milliosmoles per liter of solution, or the concentration of molecules per volume of solution. Osmotic pressure is the amount of hydrostatic pressure required to oppose the osmotic movement of water. Oncotic pressure is from plasma proteins, not body fluids. REF: p. 19 18. A nurse is discussing the movement of fluid across the arterial end of capillary membranes into the interstitial fluid surrounding the capillary. Which process of fluid movement is the nurse describing? a. Hydrostatic pressure b. Osmosis c. Diffusion d. Active transport ANS: A Blood reaching the capillary bed has a hydrostatic pressure of 25–30 mm Hg, which is sufficient force to push water across the thin capillary membranes into the interstitial space. Osmosis involves the movement of fluid from an area of higher concentration to an area of lower concentration. It does not involve pressure or force. Diffusion is the passive movement of a solute from an area of higher solute concentration to an area of lower solute concentration. Active transport involves movement up a concentration gradient.
  • 9. NURSINGTB.COM TO GET ALL CHAPTERS EMAIL ME AT>>>>> donc8246@gmail.com REF: p. 19 19. How are potassium and sodium transported across plasma membranes? a. By passive electrolyte channels b. By coupled channels c. By adenosine triphosphate enzyme (ATPase) d. By diffusion ANS: C The transporter protein ATPase is directly related to sodium and potassium transport via active transport. Electrolyte movements require energy and do not move passively, nor are they transported by diffusion. Enzymes, not electrolytes, are passed via coupled channels. REF: p. 21 20. The ion transporter that moves Na+ and Ca2+ simultaneously in the same direction is an example of which of the following types of transport? a. Biport b. Uniport c. Antiport d. Symport ANS: D When ions are transported in one direction, it is termed symport. There is no such term as biport. Uniport refers to the movement of a single molecule. Antiport refers to the movement of molecules in the opposite direction. NURSINGTB.COM REF: p. 19, Figure 1-22 21. During which process are bacteria engulfed for ingestion? a. Endocytosis b. Pinocytosis c. Phagocytosis d. Exocytosis ANS: C Phagocytosis (cell eating) involves the ingestion of large particles, such as bacteria, through the formation of large vesicles. Endocytosis involves the formation of vesicles to facilitate movement into the cell. Pinocytosis is a type of endocytosis in which fluids and solute molecules are ingested through the formation of small vesicles. Exocytosis occurs when coated pits invaginate and internalize ligand-receptor complexes in coated vesicles. REF: p. 22 22. Some cancer drugs work during the cell cycle phase where nuclear and cytoplasmic divisions occur. What is this cell cycle phase called? a. G1 b. S c. M d. G2
  • 10. NURSINGTB.COM TO GET ALL CHAPTERS EMAIL ME AT>>>>> donc8246@gmail.com ANS: C The M phase includes both nuclear and cytoplasmic divisions. The G1 phase includes the period between the M phase and the start of DNA synthesis. The S phase includes synthesis of DNA in the cell nucleus. The G2 phase includes RNA and protein synthesis. REF: pp. 25-26 23. Which causes the rapid change in the resting membrane potential that initiates an action potential? a. Potassium gates open, and potassium rushes into the cell, changing the membrane potential from negative to positive. b. Sodium gates open, and sodium rushes into the cell, changing the membrane potential from negative to positive. c. Sodium gates close, allowing potassium into the cell to change the membrane potential from positive to negative. d. Potassium gates close, allowing sodium into the cell to change the membrane potential from positive to negative. ANS: B When the threshold is reached, the cell will continue to depolarize with no further stimulation. The sodium gates open, and sodium rushes into the cell, causing the membrane potential to reduce to zero and then become positive (depolarization). Sodium is involved in creating the action potential, not potassium. The sodium gate and channel must be open, not closed. The action potential is not affected by a change in the potassium gate. REF: pp. 24-25 24. A cell is isolated, and electroN ph U yR siS olI og N yG st T uB di. esC re O vM eal that the resting membrane potential is 70 mV. The predominant intracellular ion is Na+ , and the predominant extracellular ion is K+ . With voltage change, which of the following would result in an action potential? a. K+ rushing into the cell b. Na+ rushing into the cell c. Na+ rushing out of the cell d. K+ rushing out of the cell ANS: A With voltage change, potassium rushes into, not out of, the cell. Sodium movement is not related to this process. REF: pp. 24-25 25. A nurse teaching the staff about platelet-derived growth factor includes information that platelet-derived growth factor (PDGF) stimulates the production of: a. platelets. b. epidermal cells. c. connective tissue cells. d. fibroblast cells. ANS: C Different types of cells require different growth factors; for example, PDGF stimulates the production of connective tissue cells, but not platelets, epidermal cells, or fibroblast cells.
  • 11. NURSINGTB.COM TO GET ALL CHAPTERS EMAIL ME AT>>>>> donc8246@gmail.com REF: p. 27 26. The phase of the cell cycle during which the centromeres split and the sister chromatids are pulled apart is referred to as: a. anaphase. b. telophase. c. prophase. d. metaphase. ANS: A Anaphase begins when the centromeres split and the sister chromatids are pulled apart. During telophase, a new nuclear membrane is formed around each group of 46 chromosomes, the spindle fibers disappear, and the chromosomes begin to uncoil. During prophase, the first appearance of chromosomes occurs. Metaphase occurs when two centrioles located at opposite poles of the cell pull the chromosomes to opposite sides of the cell. REF: p. 26 27. What is the role of cytokines in cell reproduction? a. Provide growth factor for tissue growth and development. b. Block progress of cell reproduction through the cell cycle. c. Restrain cell growth and development. d. Provide nutrients for cell growth and development. ANS: A Cytokines play a major role in the regulation of tissue growth and development but do not oR meI intrG aceB llu. laC r br M aking mechanisms that restrain cell restrain it. Cytokines help ovN erc U S N growth and promote cell growth, but they do not provide nutrients. REF: p. 26 28. A biopsy of the lung bronchi revealed ciliated epithelial cells that are capable of secretion and absorption. These cells are called columnar epithelium. a. simple b. ciliated simple c. stratified d. pseudostratified ciliated ANS: B Ciliated simple columnar epithelium is found in the lungs. Simple columnar epithelium is found from the stomach to the anus. Stratified columnar epithelium is found in the lining of the epiglottis, part of the pharynx, the anus, and the male urethra. Pseudostratified ciliate columnar epithelium is found in the lining of the large ducts of some glands (parotid, salivary), male urethra, respiratory passages, and Eustachian tubes of the ears. REF: p. 30, Table 1-6 29. A student is reviewing functions of the cell. The student would be correct in identifying a chief function of the nerve cell as: a. sensory interpretation. b. conductivity.
  • 12. NURSINGTB.COM TO GET ALL CHAPTERS EMAIL ME AT>>>>> donc8246@gmail.com c. maintenance of homeostasis. d. communication. ANS: B Conductivity, not sensory interpretation, homeostasis, or communication, is one of the eight chief functions of nerve cells. REF: p. 2 MULTIPLE RESPONSE 1. A nurse recalls that the basic types of tissues are: (select all that apply) a. nerve. b. epithelial. c. mucosal. d. connective. e. skeletal. f. muscle. ANS: A, B, D, F The basic tissue types include nerve, epithelial, connective, and muscle. Mucosal is a type of epithelial cell, while skeletal is a type of connective tissue. REF: p. 27 2. Characteristics of prokaryotes include which of the following? (select all that apply) a. They contain no organelles. b. Their nuclear material isN no U tR en S cI asN ed G b T yB a. nu C cO leM ar membrane. c. They contain a distinct nucleus. d. They contain histones. e. They contain a cellular membrane. ANS: A, B The prokaryotes lack a cellular membrane that encases nuclear material, thus they have no distinct nucleus; organelles and histones are also missing. REF: p. 1
  • 13. NURSINGTB.COM TO GET ALL CHAPTERS EMAIL ME AT>>>>> donc8246@gmail.com Chapter 02: Genes and Genetic Diseases MULTIPLE CHOICE 1. A nurse recalls the basic components of DNA are: a. pentose sugars and four phosphate bases. b. a phosphate molecule, deoxyribose, and four nitrogenous bases. c. adenine, guanine, and purine. d. codons, oxygen, and cytosine. ANS: B The three basic components of DNA are deoxyribose; a phosphate molecule; and four types of nitrogenous, not phosphate, bases. DNA does not contain condone. REF: p. 38 2. Which of the following mutations have the most significant effect on protein synthesis? a. Base pair substitutions b. Silent mutations c. Intron mutations d. Frameshift mutations ANS: D The frameshift mutation involves the insertion or deletion of one or more base pairs of the DNA molecule. This greatly alters the amino acid sequence, which affects protein synthesis. The base pair substitution is N a tU yp R eS oI f N mG utT atB io. n C inO w M hich one base pair replaces another. Silent mutations do not change amino acids or protein synthesis. Intron mutations are part of RNA sequencing. REF: p. 39 3. The base components of DNA are: a. A, G, C, and U. b. P, G, C, and T. c. A, G, C, and T. d. X, XX, XY, and YY. ANS: C The four base components of DNA are cytosine, thymine, adenine, and guanine, and are commonly represented by their first letters (A, C, T, and G) and not components identified as P or U. X, XX, XY, and YY are components of human chromosomes. REF: p. 38 4. A DNA strand has a region with the sequence ATCGGAT. Which of the following would be a complementary strand? a. CGATACGT b. TAGCCTAG c. TUGCCTUG
  • 14. NURSINGTB.COM TO GET ALL CHAPTERS EMAIL ME AT>>>>> donc8246@gmail.com d. UAGCCUAG ANS: B The consistent pairing of adenine with thymine and of guanine with cytosine is known as complementary base pairing; thus, A complements to T and C to G and vice versa throughout the strand. A complements to T; thus, the first letter must be a T. U does not represent a complement in the sequence. REF: p. 39 5. A biologist is explaining how RNA directs the synthesis of protein. Which process is the biologist describing? a. Termination b. Transcription c. Translocation d. Translation ANS: D In translation, RNA directs the synthesis of a polypeptide, interacting with transfer RNA (tRNA), a cloverleaf-shaped strand of about 80 nucleotides. Termination does not involve synthesis of protein. Transcription is the process by which DNA specifies a sequence of messenger RNA (mRNA). Translocation is the interchange of genetic material between nonhomologous chromosomes. REF: p. 41 6. What is the result of homologous chromosomes failing to separate during meiosis? a. Neurofibromatosis b. Nondisjunction c. Polyploidy d. Conjoined twins ANS: B NURSINGTB.COM Nondisjunction is an error in which homologous chromosomes or sister chromatids fail to separate normally during meiosis or mitosis. Neurofibromatosis is not due to chromosome failure during meiosis. Polyploidy occurs when a euploid cell has more than the diploid number of chromosomes. Conjoined twins are not due to chromosome failure during meiosis. REF: p. 45 7. A cell that does not contain a multiple of 23 chromosomes is called a cell. a. diploid b. euploid c. polyploid d. haploid ANS: C A polyploid cell is one in which a euploid cell has more than 23 pairs of chromosomes. A diploid cell is when the somatic cell nucleus has 46 chromosomes in 23 pairs. A euploid cell is a cell with multiples of the normal number of chromosomes. A haploid cell has only one member of each chromosome pair, for a total of 23 chromosomes.
  • 15. NURSINGTB.COM TO GET ALL CHAPTERS EMAIL ME AT>>>>> donc8246@gmail.com U S N REF: p. 42 8. A 20-year-old pregnant female gives birth to a stillborn child. Autopsy reveals that the fetus has 92 chromosomes. What term may be on the autopsy report to describe this condition? a. Biploidy b. Triploidy c. Tetraploidy d. Aneuploidy ANS: C Tetraploidy is a condition in which euploid cells have 92 chromosomes. Biploidy is a euploid cell with two times more chromosomes, or 46. Triploidy is a zygote that has three copies of each chromosome, rather than the usual two. Aneuploidy is when an aneuploid cell does not contain a multiple of 23 chromosomes. REF: p. 42 9. The condition in which an extra portion of a chromosome is present in each cell is called: a. reciprocal translocation. b. partial trisomy. c. inversion. d. Down syndrome. ANS: B Partial trisomy is a condition in which only an extra portion of a chromosome is present in each cell. A reciprocal translocation occurs when breaks take place in two different chromosomes and the material is exchanged. An inversion occurs when two breaks take place on a chromosome, followed b N y t h Re rI einsG ertio B n . oC f t hM e missing fragment at its original site, but in inverted order. Down syndrome is an aneuploidy of the twenty-first chromosome. REF: p. 46 10. After a geneticist talks to a patient about being a chromosomal mosaic, the patient asks the nurse what that means. How should the nurse respond? You may genetic disease(s). a. only be a carrier of the b. have a mild form of the c. have two d. be sterile as a result of the ANS: B A chromosomal mosaic means the body has two or more different cell lines, each of which has a different karyotype; thus, the person has a mild form of the disease. Mosaics are not only carriers; they have the disease; they have two different lines but not two different diseases; and they are not necessarily sterile. REF: p. 46 11. What is the most common cause of Down syndrome? a. Paternal nondisjunction b. Maternal translocations c. Maternal nondisjunction
  • 16. NURSINGTB.COM TO GET ALL CHAPTERS EMAIL ME AT>>>>> donc8246@gmail.com d. Paternal translocations ANS: C The most common cause of Down syndrome is maternal, not paternal, nondisjunction. Translocation is not a cause of this syndrome. REF: p. 46, Table 2-1 12. A patient wants to know the risk factors for Down syndrome. What is the nurse’s best response? a. Fetal exposure to mutagens in the uterus. b. Increased paternal age. c. Family history of Down syndrome. d. Pregnancy in women over age 35. ANS: D The primary risk for Down syndrome is pregnancy in women over 35. Down syndrome is a trisomy and not due to fetal exposure or paternal age. Down syndrome is a chromosomal abnormality and is not related to family history. REF: pp. 46-47 13. A 13-year-old girl has a karyotype that reveals an absent homologous X chromosome with only a single X chromosome present. What medical diagnosis will the nurse observe on the chart? a. Down syndrome b. Cri du chat syndrome c. Turner syndrome d. Fragile X syndrome ANS: C NURSINGTB.COM A condition with the presence of a single X chromosome and no homologous X or Y chromosome, so the individual has a total of 45 chromosomes, is known as Turner syndrome. Down syndrome is a change in one arm of a chromosome. Cri du chat syndrome is due to a chromosome deletion. Fragile X syndrome is due to a break or a gap in a chromosome. REF: p. 47 14. What genetic disorder is the result if an individual possesses an XXY chromosome configuration? a. Turner b. Klinefelter c. Down d. Fragile X ANS: B Individuals with at least two X chromosomes and one Y chromosome in each cell (47 XXY karyotype) have a disorder known as Klinefelter syndrome. A condition with the presence of a single X chromosome and no homologous X or Y chromosome, so the individual has a total of 45 chromosomes, is known as Turner syndrome. Down syndrome is a trisomy. Fragile X syndrome is due to a break or a gap in a chromosome, not an extra chromosome.
  • 17. NURSINGTB.COM TO GET ALL CHAPTERS EMAIL ME AT>>>>> donc8246@gmail.com REF: p. 47 15. A patient demonstrates severe mental retardation caused by a deletion of part of chromosome 5. What genetic disorder will the nurse see documented in the chart? a. Prader-Willi syndrome b. Down syndrome c. Cri du chat syndrome d. Trisomy X ANS: C Cri du chat syndrome means “cry of the cat” and describes the characteristic cry of the affected child. Another symptom of the disorder is mental retardation. The disease is caused by a deletion of part of the short arm of chromosome 5. Prader-Willi syndrome is characterized by short stature, obesity, and hypogonadism. Down syndrome does cause mental retardation but is due to chromosome 21, not chromosome 5. Trisomy X can result in mental retardation but is due to an extra X chromosome. REF: p. 48 16. An aide asks the nurse why people who have neurofibromatosis will show varying degrees of the disease. Which genetic principle should the nurse explain to the aide? a. Penetrance b. Expressivity c. Dominance d. Recessiveness ANS: B Expressivity is the extent of v N ariR aU tion I SinN G phT eB no .tC ypeM associated with a particular genotype. For neurofibromatosis, a variety of manifestations occur among individuals. The penetrance of a trait is the percentage of individuals with a specific genotype who also exhibit the expected phenotype. Dominance refers to observable traits and risk of transmission. Recessiveness refers to silent strains with reduced risk of occurrence. REF: p. 51 17. Cystic fibrosis is caused by what gene abnormality? a. X-linked dominant b. X-linked recessive c. Autosomal dominant d. Autosomal recessive ANS: D Cystic fibrosis is an autosomal recessive disorder. It is not a result of X links or dominant pathology. REF: p. 52 18. A 15-year-old female is diagnosed with Prader-Willi syndrome. This condition is an example of: a. genomic imprinting. b. an autosomal recessive trait.
  • 18. NURSINGTB.COM TO GET ALL CHAPTERS EMAIL ME AT>>>>> donc8246@gmail.com c. an autosomal dominant trait. d. a sex-linked trait. ANS: A Prader-Willi, an example of gene imprinting, is not associated with any autosomal sex-linked abnormality. REF: p. 52 19. A patient, age 9, is admitted to a pediatric unit with Duchenne muscular dystrophy. When planning care the nurse recalls the patient inherited this condition through a trait that is: a. X-linked dominant. b. X-influenced. c. X-limited. d. X-linked recessive. ANS: D Duchenne muscular dystrophy is a relatively common X-linked recessive, not dominant, disorder. While it is sex linked, it is not X-limited or X-influenced. REF: p. 55 20. A child is diagnosed with cystic fibrosis. History reveals that the child’s parents are siblings. Cystic fibrosis was most likely the result of: a. X-inactivation. b. genomic imprinting. c. consanguinity. d. obligate carriers. ANS: C NURSINGTB.COM Consanguinity refers to the mating of two related individuals, and the offspring of such matings are said to be inbred. Consanguineous matings produce a significant increase in recessive disorders and are seen most often in pedigrees for rare recessive disorders. X-inactivation occurs when one X chromosome in the somatic cells of females is permanently inactivated. Genomic imprinting is related to methylation and other changes. Obligate carriers are those who have an affected parent and affected children and, therefore, must themselves carry the mutation. REF: p. 54 21. A 12-year-old male is diagnosed with Klinefelter syndrome. His karyotype would reveal which of the following? a. XY b. XX c. XYY d. XXY ANS: D A person with Klinefelter syndrome has an XXY karyotype. An XY is a normal male. An XX is a normal female. An XYY is an aneuploid karyotype. REF: p. 47
  • 19. NURSINGTB.COM TO GET ALL CHAPTERS EMAIL ME AT>>>>> donc8246@gmail.com 22. To express a polygenic trait: a. genes must interact with the environment. b. several genes must act together. c. multiple mutations must occur in the same family. d. penetrance must occur. ANS: B Polygenic traits are those that result from several genes acting together. When environmental factors influence the expression of the trait, the term multifactorial inheritance is used. When multiple mutations occur in the same family, the mechanism most likely responsible is termed germline mosaicism. Penetrance of a trait is the percentage of individuals with a specific genotype who also exhibit the expected phenotype. REF: pp. 57-58 23. What is the diagnosis of a 13-year-old female who has a karyotype that reveals an absent homologous X chromosome with only a single X chromosome present? Her features include a short stature, widely spaced nipples, and a reduced carrying angle at the elbow. a. Down syndrome b. Cri du chat syndrome c. Turner syndrome d. Klinefelter syndrome ANS: C Turner syndrome is characterized by short stature, female genitalia, webbed neck, shield-like chest with underdeveloped breasts and widely spaced nipples, and imperfectly developed ovaries. Down syndrome is cN ha U ra R cS teI riz N eG d T by Bd . is C tiO nc M tive characteristics: low nasal bridge, epicanthal folds, protruding tongue, and low-set ears. Cri du chat syndrome is characterized by low birth weight, severe mental retardation, microcephaly (smaller than normal head size), and heart defects. Klinefelter syndrome is characterized by small testes, some development of the breasts, sparse body hair, and long limbs. REF: p. 46, Table 2-1 24. The gradual increase in height among the human population over the past 100 years is an example of: a. a polygenic trait. b. a multifactorial trait. c. crossing over. d. recombination. ANS: B The gradual increase in height is an example of multifactorial traits influenced by genes and also by environment. Polygenic traits result from several genes acting together. Crossing over is an abnormal chromosome structure. Recombination results from new arrangements of alleles. REF: p. 58 25. When discussing DNA replication, which enzyme is most important?
  • 20. NURSINGTB.COM TO GET ALL CHAPTERS EMAIL ME AT>>>>> donc8246@gmail.com a. RNA polymerase b. Transfer RNA c. Messenger RNA d. DNA polymerase ANS: D DNA polymerase, not RNA polymerase, is the primary enzyme involved in replication. It adds bases to the new DNA strand and performs “proofreading” functions. Neither messenger RNA nor transfer RNA is as important to DNA replication. REF: p. 39 26. The regions of the heterogeneous nuclear RNA that must be spliced out to form functional RNA are called: a. promoter sites. b. introns. c. exons. d. anticodon. ANS: B When the mRNA is first transcribed from the DNA template, it reflects exactly the base sequence of the DNA. In eukaryotes, many RNA sequences are removed by nuclear enzymes, and the remaining sequences are spliced together to form the functional mRNA that migrates to the cytoplasm. The excised sequences are called introns (intervening sequences), and the sequences that are left to code for proteins are called exons. In translation, RNA directs the synthesis of a polypeptide, a cloverleaf-shaped strand of about 80 nucleotides. The tRNA molecule has a site where an amino acid attaches. The three-nucleotide sequence at the opposite side ofthe cloverleaN f U isR ca S ll I ed NtG he Ta B n. tic C oO do M n. REF: p. 41 27. A 50-year-old male was recently diagnosed with Huntington disease. Transmission of this disease is associated with: a. penetrance of a trait. b. recurrence risk. c. expressivity. d. delayed age of onset. ANS: D A key feature of Huntington disease is its delayed age of onset such that symptoms are not seen until 40 years of age or later. The penetrance of a trait is the percentage of individuals with a specific genotype who also exhibit the expected phenotype. Recurrence risk is the percentage of family members who will inherit the disease. Expressivity is the extent of variation in phenotype associated with a particular genotype. REF: p. 51 28. What type of mutation does not change the amino acid sequence and thus has no observable consequence? a. Frameshift b. Spontaneous
  • 21. NURSINGTB.COM TO GET ALL CHAPTERS EMAIL ME AT>>>>> donc8246@gmail.com c. Silent d. Missense ANS: C Silent mutations do not change the amino acid sequence and therefore have no consequences. Frameshift mutations involve the insertion or deletion of one or more base pairs of the DNA molecule. They alter the amino acid sequence. Spontaneous mutations occur in the absence of exposure to a mutagen and produce changes in the amino acid sequence. Missense mutations, a form of base pair substitution, alter amino acids, which produce a change (i.e., the “sense”) in a single amino acid. REF: p. 39 29. A nurse is reviewing the pedigree chart. When checking for a proband, what is the nurse looking for? a. The person who is first diagnosed with a genetic disease. b. The individual who has a disease gene but is phenotypically normal. c. The phenotype of genetic material. d. The codominance. ANS: A The pedigree chart summarizes family relationships and shows which members of a family are affected by a genetic disease. The pedigree begins with the proband. The person who has a disease gene but is phenotypically normal is a carrier. The phenotype is the result of both genotype and environment; it is not a proband. Codominance is not represented by a proband, but it occurs when the heterozygote is distinguishable from both homozygotes. REF: p. 50 NURSINGTB.COM 30. Which of the following disorders is manifested primarily in males? a. Cystic fibrosis b. Neurofibromatosis c. Muscular dystrophy d. Klinefelter syndrome ANS: C Muscular dystrophy is manifested primarily in males. Cystic fibrosis, neurofibromatosis, and Klinefelter syndrome are manifested in both males and females. REF: p. 55 MULTIPLE RESPONSE 1. When the nurse is teaching the staff about X-linked recessive disorders, which information should the nurse include? (select all that apply) a. The trait is seen much more often in females than in males. b. The trait is never transmitted from father to son. c. The gene can be transmitted through a series of carrier females. d. The gene is passed from an affected father to all his daughters. e. The trait never skips generations.
  • 22. NURSINGTB.COM TO GET ALL CHAPTERS EMAIL ME AT>>>>> donc8246@gmail.com ANS: B, C, D The principles of X-linked recessive inheritance include: the trait is seen much more often in males than in females; the trait is never transmitted from father to son; the gene can be transmitted through a series of carrier females; the gene is passed from an affected father to all his daughters, who, as phenotypically normal carriers, transmit it to approximately half their sons, who are affected. X-linked recessive disorders can skip generations since it is a one in four chance. REF: p. 55 NURSINGTB.COM
  • 23. NURSINGTB.COM TO GET ALL CHAPTERS EMAIL ME AT>>>>> donc8246@gmail.com N R I Chapter 03: Epigenetics and Disease MULTIPLE CHOICE 1. When considering abnormal epigenetic modifications, what factor is currently being viewed as strongly associated with the development of some cancers? a. Family genetics b. Lifestyle choices c. Environmental stressors d. Emotional coping skills ANS: C Environmental stressors can markedly increase the risk of aberrant epigenetic modification and are strongly associated with some cancers. While genetics, lifestyle choices, and coping skills can affect the development and management of cancer, they are not currently considered as being the primary factors in the epigenetic modification that occurs. REF: p. 62 2. Housekeeping genes are vital to the function and maintenance of all the body’s cells. What characteristic is associated with these genes? a. They lack encoding histones. b. They are transcriptionally active. c. Ribosomal RNA genes are absent. d. Epigenetic silencing has occurred. GB.CM U S N T O ANS: B A small percentage of genes, termed housekeeping genes, are necessary for the function and maintenance of all cells. These genes escape epigenetic silencing and remain transcriptionally active in all or nearly all cells. Housekeeping genes include encoding histones, DNA and RNA polymerases, and ribosomal RNA genes. REF: p. 64 3. What characteristic of Prader-Willi syndrome is not a characteristic of Angelman syndrome? a. It is inherited from the father. b. Mental retardation is observable. c. Imprinting of an abnormal chromosome 15. d. Seizure disorder is present. ANS: A A well-known disease example of imprinting is associated with a deletion of about 4 million base (Mb) pairs of the long arm of chromosome 15. When this deletion is inherited from the father, the child manifests Prader-Willi syndrome, whose features include short stature, hypotonia, small hands and feet, obesity, mild to moderate mental retardation, and hypogonadism. The same 4 Mb deletion, when inherited from the mother, causes Angelman syndrome, which is characterized by severe mental retardation, seizures, and an ataxic gait. REF: p. 65
  • 24. NURSINGTB.COM TO GET ALL CHAPTERS EMAIL ME AT>>>>> donc8246@gmail.com 4. Research has demonstrated that neural stem cells have an impaired ability to differentiate into functional neurons when subjected to: a. ethanol. b. marijuana. c. insufficient nutrients. d. poor oxygenation. ANS: A It has been found that treating cultured neural stem cells with ethanol impairs their ability to differentiate to functional neurons; this impairment seems to be correlated with aberrant, dense methylation at loci that are active in normal neuronal tissue. The research does not support the effects of marijuana, insufficient nutrition, or poor oxygenation on the stem cell’s ability to differentiate appropriately. REF: p. 67 5. What is the role of inactive MLH1 in the development of some forms of inherited colon cancer? a. Deletion of a nucleotide repeat in the DUX4 gene. b. DNA damage is left unrepaired. c. Expansion in the number of cytosine-guanine (CG) dinucleotide. d. Abnormalities of chromosome 11p15.5 that lead to downregulation of IGF2. ANS: B A major cause of one form of inherited colon cancer (hereditary nonpolyposis colorectal cancer [HNPCC]) is the methylation of the promoter region of a gene, MLH1, whose protein product repairs damaged DNN AU . W RS he I nN M G LT HB 1 . be C co O m M es inactive, DNA damage accumulates, eventually resulting in colon tumors. Facioscapulohumeral muscular dystrophy (FSHMD) is associated with the DUX4 gene. Fragile X is associated with the cytosine-guanine (CG) dinucleotide. Downregulation of IGF2 is associated with Russell-Silver syndrome. REF: p. 69 6. Mutations in the encoding of histone-modifying proteins have been shown to influence the development of what congenial condition? a. Cleft palate b. Acephalous c. Heart disease d. Webbed digits ANS: C Mutations in genes that encode histone-modifying proteins have been implicated in congenital heart disease. Research has yet to provide a relationship between abnormal histone-modifying proteins and cleft palate, acephalous, and webbed digits. REF: p. 64 7. Which embryonic stem cell characteristic is referred to as totipotent? a. Ability to differentiate into any type of somatic cell. b. Ability to repair its own damaged DNA.
  • 25. NURSINGTB.COM TO GET ALL CHAPTERS EMAIL ME AT>>>>> donc8246@gmail.com c. Ability to determine which parental chromosome copy it will imprint. d. Ability to minimize the impact of poor in utero nutrition. ANS: A Each of the cells in the very early embryo has the potential to give rise to a somatic cell of any type. These embryonic stem cells are therefore said to be totipotent (“possessing all powers”). The term totipotent does not infer the ability to repair damaged DNA, select specific chromosome copies, or adjust for poor in utero nutrition. REF: p. 64 8. 5-Azacytidine has demonstrated promise in the treatment of which form of cancer? a. Liver b. Colon c. Gallbladder d. Pancreatic ANS: D Though associated with various side effects, including digestive disturbance, 5-azacytidine has shown promise in the treatment of pancreatic cancer. There is no support for its use in the treatment of liver, colon, or gallbladder cancers. REF: p. 70 9. During which stage of human development does the role of epigenetics have the greatest impact on the development of epigenetic abnormalities? a. Infancy b. Puberty c. In utero d. Middle age ANS: C NURSINGTB.COM Conditions encountered in utero, during childhood, and even during adolescence or later can have long-term impacts on epigenetic states, which sometimes can be transmitted across generations. The impact is not supported for the periods of infancy, puberty, and middle age. REF: p. 66 10. What comorbid condition does an individual diagnosed with Beckwith-Wiedemann syndrome have an increased risk of developing? a. Cancer b. Diabetes c. Depression d. Food allergies ANS: A Beckwith-Wiedemann syndrome is accompanied by an increased predisposition to cancer. There is no current correlation between Beckwith-Wiedemann syndrome and diabetes, depression, or food allergies. REF: p. 65
  • 26. NURSINGTB.COM TO GET ALL CHAPTERS EMAIL ME AT>>>>> donc8246@gmail.com MULTIPLE RESPONSE 1. Screening tools that are epigenetically based have shown promise in diagnosing which types of cancer? (select all that apply) a. Colon b. Breast c. Skin d. Bladder e. Prostate ANS: A, B, D, E Monitoring for misregulation of miRNAs has shown promise as a tool for early diagnosis of cancers of the colon, breast, and prostate. Other epigenetics-based screening approaches have shown promise for detection of cancers of the bladder, lung, and prostate. Such screenings have not yet been developed for skin cancers. REF: p. 69 2. Research has provided support for the theory that epigenetic modifications can result from deficient in utero nutrition causing which chronic disease? (select all that apply) a. Obesity b. Asthma c. Cardiovascular disease d. Diabetes e. Crohn’s disease ANS: A, C, D NURSINGTB.COM When researchers sought to investigate how exposure to famine in utero had impacted individuals born in a historically prosperous country whose nutritional intake was dramatically impacted by WWII, they found that individuals who suffered nutritional deprivation in utero were more likely to suffer from obesity and diabetes as adults than individuals who had not experienced nutritional deprivation during gestation. Other data sets reveal elevated risk of cardiovascular and metabolic disease for offspring of individuals exposed during early development to fluctuations in agricultural yields. The research does support the development of asthma or Crohn’s disease. REF: p. 66
  • 27. NURSINGTB.COM TO GET ALL CHAPTERS EMAIL ME AT>>>>> donc8246@gmail.com Chapter 04: Altered Cellular and Tissue Biology MULTIPLE CHOICE 1. A report comes back indicating that muscular atrophy has occurred. A nurse recalls that muscular atrophy involves a decrease in muscle cell: a. number. b. size. c. vacuoles. d. lipofuscin. ANS: B Atrophy is a decrease or shrinkage in cellular size. Hyperplasia is an increase in the number of cells. Vacuoles are membrane-bound vesicles within the cell that contain cellular debris and hydrolytic enzymes. Lipofuscin is the yellow-brown age pigment. REF: p. 74 2. During childhood, the thymus decreases in size, and this is referred to as what type of atrophy? a. Physiologic b. Pathologic c. Disuse d. Neurogenic ANS: A NURSINGTB.COM A normal decrease in cell size is physiologic atrophy. Atrophy can result from disease (pathologic), disuse, or nerve injury (neurogenic). REF: p. 74 3. When planning care for a cardiac patient, the nurse knows that in response to an increased workload, cardiac myocardial cells will experience hypertrophy which is an: a. increase in size. b. decrease in length. c. increase in excitability. d. decrease in number. ANS: A Hypertrophy is a compensatory increase in the size of cells in response to mechanical stimuli (also called mechanical load or stress, such as from stretching, repetitive, chronic, pressure, or volume overload) and consequently increases the size of the affected organ. The cells of the heart and kidneys are particularly prone to enlargement. A decrease in length is not associated with hypertrophy. A deficiency of electrolytes or minerals could lead to an increase in excitability; it is not due to increased workload or related to hypertrophy. A decrease in cell numbers is referred to as hypoplasia. REF: p. 75
  • 28. NURSINGTB.COM TO GET ALL CHAPTERS EMAIL ME AT>>>>> donc8246@gmail.com 4. A 55-year-old male with a 30-year history of smoking is examined for respiratory disturbance. Examination of his airway (bronchial) reveals that stratified squamous epithelial cells have replaced the normal columnar ciliated cells. This type of cellular adaptation is called: a. anaplasia. b. hyperplasia. c. metaplasia. d. dysplasia. ANS: C Metaplasia is the reversible replacement of one mature cell type by another, sometimes a less differentiated cell type. Anaplasia is loss of cellular differentiation. Hyperplasia is an increase in the number of cells resulting from an increased rate of cellular division. Dysplasia refers to abnormal changes in the size, shape, and organization of mature cells. REF: p. 77 5. When planning care for the pregnant patient, the nurse will recall that the mammary glands enlarge as a consequence of: a. compensatory hyperplasia. b. hormonal hyperplasia. c. hormonal anaplasia. d. compensatory anaplasia. ANS: B An increase in the mammary glands during pregnancy is a result of hormonal changes. The number of mammary cells increases in response to increased hormone levels, not as a compensatory mechanism. Anaplasia is a reversal to less mature cells. NURSINGTB.COM REF: p. 76 6. A 24-year-old female presents with excessive menstrual bleeding. The physician identified endometrial changes that are due to hormonal imbalances. These cellular changes would be referred to as: a. dysplasia. b. pathologic dysplasia. c. hyperplasia. d. pathologic hyperplasia. ANS: D Because the changes are due to an imbalance, they would be considered pathologic hyperplasia, a term more descriptive than simple hyperplasia. The endometrial changes were not abnormal in size and shape; thus, it is not dysplasia regardless of cause. REF: pp. 76-77 7. A 55-year-old male is diagnosed with hepatocellular cancer secondary to hepatitis C. If the cancerous region of the liver is removed, the remaining cells would undergo: a. pathologic hyperplasia. b. pathologic metaplasia. c. compensatory hyperplasia. d. compensatory aplasia.
  • 29. NURSINGTB.COM TO GET ALL CHAPTERS EMAIL ME AT>>>>> donc8246@gmail.com ANS: C Compensatory hyperplasia is an adaptive, not pathologic, mechanism that enables certain organs to regenerate. Metaplasia is the reversible replacement of one mature cell type by another, sometimes less differentiated, cell type. Aplasia is not a compensatory mechanism. REF: p. 76 8. A 40-year-old female’s Pap smear indicates abnormal changes in the shape and organization of cervical cells. Which term would be used to identify this type of change? a. Metaplasia b. Atrophy c. Hypertrophy d. Dysplasia ANS: D When cervical cells undergo dysplasia, there is a change in their size, shape, and organization. Metaplasia is the reversible replacement of one mature cell type. The cells have not decreased in size; therefore, atrophy is incorrect. The cells have not increased in size in response to stimuli; therefore, they have not hypertrophied. REF: p. 77 9. A 75-year-old male presents with chest pain on exertion. The chest pain is most likely due to hypoxic injury secondary to: a. malnutrition. b. free radicals. c. ischemia. d. chemical toxicity. ANS: C NURSINGTB.COM The cardiac cells are deprived of oxygen, leading to ischemia, a reduction in blood supply to tissues. The cells are deprived of oxygen; they are not malnourished. Free radicals are electrically uncharged atoms or groups of atoms that have an unpaired electron. Chemical toxicity is not a factor in the chest pain. REF: pp. 78-79 10. A patient has a heart attack that leads to progressive cell injury that causes cell death with severe cell swelling and breakdown of organelles. What term would the nurse use to define this process? a. Adaptation b. Calcification c. Apoptosis d. Necrosis ANS: D Necrosis is the sum of cellular changes after local cell death. Cellular adaptation is a reversible, structural, or functional response to both normal or physiologic conditions and adverse or pathologic conditions. Calcification is an accumulation of calcium salts. Apoptosis is an active process of cellular self-destruction. REF: p. 78, Table 4-1
  • 30. NURSINGTB.COM TO GET ALL CHAPTERS EMAIL ME AT>>>>> donc8246@gmail.com 11. Sodium and water accumulation in an injured cell are a direct result of: a. decreased ATP production. b. karyorrhexis. c. ribosome detachment. d. dehydration. ANS: A A reduction in ATP levels causes the plasma membrane’s sodium-potassium (Na+ -K+ ) pump and sodium-calcium exchange to fail. Sodium and water can then enter the cell freely. Karyorrhexis means fragmentation of the nucleus into smaller particles or “nuclear dust.” Ribosome detachment reduces protein synthesis. Dehydration leads to loss of sodium and water. REF: p. 80 12. The early dilation (swelling) of the cell’s endoplasmic reticulum results in: a. increased aerobic metabolism. b. failure of DNA. c. reduced protein synthesis. d. increased Na+ -K+ pump function. ANS: C Early dilation of the endoplasmic reticulum causes the ribosomes to detach from the rough endoplasmic reticulum, reducing protein synthesis. Aerobic metabolism is a normal process and would not lead to swelling. Cellular swelling will not alter cellular DNA. A reduction in the Na+ - K+ pump leads to an intracellular accumulation of sodium and calcium and diffusion of potassium out of the cell. S N odU R iumS I and N Gwa T B te. r c Ca n M then enter the cell freely, and cellular swelling results. REF: p. 80 13. A 52-year-old male suffered a myocardial infarction secondary to atherosclerosis and ischemia. Once blood flow is returned to the damaged heart, reperfusion injury occurs as a result of: a. oxidation stress. b. vacuolation. c. decreased intracellular calcium. d. lipid acceptor proteins. ANS: A Reperfusion injury can result from oxidative stress, increased intracellular calcium, inflammation, or complement activation. Oxidative stress causes the formation of radicals that cause further membrane damage and mitochondrial calcium overload. Vacuolation leads to cellular swelling but is not associated with reperfusion. An increase of intracellular calcium is a cause of reperfusion injury. Lipid acceptor proteins bind with triglycerides to create fatty liver, and they do not affect the myocardium. REF: p. 81
  • 31. NURSINGTB.COM TO GET ALL CHAPTERS EMAIL ME AT>>>>> donc8246@gmail.com 14. A family presents to their primary care provider reporting headache, nausea, weakness, tinnitus, and vomiting. Which of the following would be the most likely explanation for these symptoms? a. Lead exposure b. Carbon monoxide poisoning c. Ethanol exposure d. Mercury poisoning ANS: B Symptoms related to carbon monoxide poisoning include headache, giddiness, tinnitus (ringing in the ears), nausea, weakness, and vomiting. Although nausea and vomiting can occur with lead exposure, lead toxicity is primarily manifested by convulsions and delirium and, with peripheral nerve involvement, wrist, finger, and sometimes foot paralysis. Ethanol exposure has CNS effects and would not affect the whole family. Mercury poisoning is manifested by CNS effects and would not lead to nausea and vomiting. REF: p. 90 15. A common pathway of irreversible cell injury involves increased intracellular: a. sodium. b. potassium. c. magnesium. d. calcium. ANS: D Increased intracellular calcium levels activate cell enzymes (caspases) that promote cell death by apoptosis. Persistent ischemia is associated with irreversible injury and necrosis. Irreversible injury is a ssoc iatN edUsR trS ucI tuN raG llT y B w. ith Cs O eM vere swelling ofthe mitochondria, severe damage to plasma membranes, and swelling of lysosomes. Cellular injury is not associated with sodium, potassium, or magnesium levels. REF: p. 81 16. A 50-year-old male sustained a closed head injury as a result of a motor vehicle accident. CT scan revealed a collection of blood between the inner surface of the dura mater and the surface of the brain. Which type of injury will the nurse be caring for? a. Subdural hematoma b. Epidural hematoma c. Contusion d. Abrasion ANS: A A subdural hematoma occurs when blood is between the inner surface of the dura mater and the surface of the brain; it can result from blows, falls, or sudden acceleration/deceleration of the head. An epidural hematoma is a collection of blood between the inner surface of the skull and the dura; it is most often associated with a skull fracture. A contusion is bleeding into the skin or underlying tissues. An abrasion (scrape) results from removal of the superficial layers of the skin caused by friction between the skin and injuring object. REF: p. 94, Table 4-8
  • 32. NURSINGTB.COM TO GET ALL CHAPTERS EMAIL ME AT>>>>> donc8246@gmail.com 17. A 20-year-old male presents to the emergency department with a jagged sharp-force injury that is longer than it is deep. Which type of wound will the nurse be caring for? a. Stab wound b. Incised wound c. Puncture wound d. Chopping wound ANS: B An incised wound is a cut that is longer than it is deep. A stab wound is a penetrating sharp-force injury that is deeper than it is long. A puncture wound is without sharp edges and is made with an instrument like a nail. Heavy, edged instruments (axes, hatchets, propeller blades) produce wounds with a combination of sharp and blunt force characteristics. REF: p. 94, Table 4-8 18. A 30-year-old female presents with a gunshot wound to the head. The wound has seared edges and a deep penetration of smoke and gunpowder fragments. This wound would be documented as a(n): a. exit. b. intermediate range entrance. c. contact range entrance. d. indeterminate range entrance. ANS: C A contact range entrance wound is a distinctive type of wound that happens when a gun is held so the muzzle rests on or presses into the skin surface; in addition to the hole, there is searing of the edges of the wound from flame and soot or smoke on the edges of the wound. It is unlikely it is an exit wound Ns U in R cS e tI he N re Gi T s B on . ly Co O nM e wound and it has seared edges and gunpowder fragments. An intermediate range entrance wound is surrounded by gunpowder tattooing or stippling. An indeterminate range entrance wound occurs when flame, soot, or gunpowder does not reach the skin surface but the bullet does. REF: p. 95, Table 4-8 19. A 15-year-old female presents to the ER following a physical assault. She has internal damage to the neck with deep bruising. X-ray reveals fractures of the hyoid bone and tracheal and cricoid cartilage. Which of the following most likely caused her injuries? a. Chemical asphyxiation b. Choking asphyxiation c. Ligature strangulation d. Manual strangulation ANS: D Squeezing of the neck as with strangulation would fraction the hyoid bone. Chemical asphyxiation would lead to breathing problems but would not result in fracture. Choking asphyxiation would lead to swelling of tissues but would not result in fracture. In ligature strangulation, the mark on the neck is horizontal without the inverted V pattern seen in hangings. It would not lead to fracture. REF: p. 96
  • 33. NURSINGTB.COM TO GET ALL CHAPTERS EMAIL ME AT>>>>> donc8246@gmail.com 20. A 55-year-old male has swelling of the feet. Which of the following aided in the development of swelling? a. Increased ATP b. Chloride movement out of the cell c. Na+ movement into the cell d. Decreased oncotic pressure ANS: C When sodium and water enter the cell freely, cellular swelling, as well as early dilation of the endoplasmic reticulum, results. Decreased ATP would lead to swelling. Chloride movement out of the cell would affect muscle contraction but does not lead to swelling. Increased oncotic pressure would not affect swelling. REF: p. 97 21. A 35-year-old female is diagnosed with multiple myeloma. Biopsy of the tumor reveals Russell bodies, and laboratory testing reveals kidney dysfunction. Which substance should the nurse monitor as it is accumulating in the patient’s body? a. Glycogen b. Protein c. Pigment d. Melanin ANS: B Russell bodies occur due to excess aggregates of protein. Excess glycogen would affect blood glucose. Increased pigment would not lead to kidney dysfunction. Melanin accumulates in epithelial cells (keratinocytes) of the skin and retina. NURSINGTB.COM REF: p. 99 22. A newborn male is diagnosed with albinism based on skin, eye, and hair appearance. Which finding will support this diagnosis? a. Increased melanin b. Increased hemoproteins c. Inability to convert tyrosine to DOPA (3,4-dihydroxyphenylalanine) d. Inability to convert bile to bilirubin ANS: C The person with albinism is unable to convert tyrosine to DOPA, an intermediate in melanin biosynthesis. An increase in melanin would cause skin to be darker. Hemoprotein accumulations in cells are caused by excessive storage of iron, which is transferred to the cells from the bloodstream. An inability to convert bile to bilirubin would not lead to albinism. REF: p. 100 23. A 23-year-old male develops a black eye following a fight. When the aide asks the nurse why this occurred, the nurse’s best response is that the bruising is due to an accumulation of: a. transferrin. b. bilirubin. c. albumin. d. hemosiderin.
  • 34. NURSINGTB.COM TO GET ALL CHAPTERS EMAIL ME AT>>>>> donc8246@gmail.com ANS: D Hemosiderin is responsible for the color changes in a black eye. Transferrin is a transport protein responsible for iron transport. Bilirubin is the normal, yellow-to-green pigment of bile derived from the porphyrin structure of hemoglobin. Albumin is the protein in the serum, responsible for cellular integrity. REF: p. 100 24. Liquefactive necrosis occurs in the brain because: a. debris is not digested by hydrolases. b. of protein denaturation. c. it is rich in hydrolytic enzymes and lipids. d. ischemia results in chemical injury. ANS: C Liquefactive necrosis is due to enzymatic action and because cells of the brain are rich in enzymes. Protein denaturation occurs primarily in the kidneys. Liquefactive necrosis is due to enzymatic reaction and not to hypoxia or hydrolases. REF: p. 103 25. A 2-year-old swallowed watch batteries. Following ingestion, kidney function was impaired, and the heart began to fail. Which of the following was the most likely cause? a. Karyorrhexis b. Coagulative necrosis c. Ammonia accumulation d. Caseous necrosis ANS: B NURSINGTB.COM Coagulative necrosis occurs primarily in the kidneys, heart, and adrenal glands and commonly results from hypoxia. Karyorrhexis means fragmentation of the nucleus into smaller particles or “nuclear dust.” Ammonia accumulation is not associated with this toxicity. Caseous necrosis results from tuberculosis pulmonary infection. REF: pp. 102-103 26. A group of prison inmates developed tuberculosis following exposure to an infected inmate. On examination, tissues were soft and granular (like clumped cheese). Which of the following is the most likely cause? a. Coagulative necrosis b. Liquefactive necrosis c. Caseous necrosis d. Autonecrosis ANS: C Caseous necrosis results from tuberculosis pulmonary infection. Coagulative necrosis occurs primarily in the kidneys, heart, and adrenal glands, and commonly results from hypoxia. Liquefactive necrosis results from ischemic injury to neurons and glial cells in the brain. Autonecrosis is a process of cellular self-digestion and is not due to infection such as tuberculosis. REF: p. 103
  • 35. NURSINGTB.COM TO GET ALL CHAPTERS EMAIL ME AT>>>>> donc8246@gmail.com 27. A 50-year-old female became infected with Clostridium bacteria and died a week later. Examination of her red blood cells revealed lysis of membranes. Which of the following was the most likely cause of her death? a. Fat necrosis b. Wet gangrene c. Gangrenous necrosis d. Gas gangrene ANS: D Gas gangrene is a special type of gangrene caused by infection of injured tissue by one of many species of Clostridium. Fat necrosis is cellular dissolution caused by powerful enzymes, called lipases, that occur in the breast, pancreas, and other abdominal structures. Wet gangrene develops when neutrophils invade the site, causing liquefactive necrosis. Gangrenous necrosis is due to death of tissue and results from severe hypoxic injury. REF: p. 104 28. While reading a textbook, a student reads the term apoptosis. The student recalls that apoptosis is a condition in which cells program themselves to: a. atrophy. b. die. c. regenerate. d. age. ANS: B In apoptosis, cells are programmed to die. Apoptosis is not associated with cell atrophy, regeneration, or aging. NURSINGTB.COM REF: p. 104 29. A 50-year-old male intravenous drug user is diagnosed with hepatitis C. Examination of the liver reveals cell death secondary to: a. fat necrosis. b. physiologic apoptosis. c. infection-induced apoptosis. d. pyknosis. ANS: C With hepatitis C, the liver will demonstrate apoptosis that is a result of the viral infection. Fat necrosis occurs with enzymatic action due to lipases. Apoptosis is not a normal physiological process. Pyknosis occurs when the nucleus shrinks and becomes a small, dense mass of genetic material. REF: p. 104 30. What principle should the nurse remember when trying to distinguish aging from diseases? a. It is difficult to tell the difference because both processes are believed to result from cell injury. b. It is easy to tell normal processes from abnormal processes. c. Disease, unlike aging, has a genetic component.
  • 36. NURSINGTB.COM TO GET ALL CHAPTERS EMAIL ME AT>>>>> donc8246@gmail.com U S N T O d. Aging is defined as exceeding life expectancy, but not maximal life span. ANS: A It is difficult to differentiate between aging and disease because both occur secondary to cellular aging. It is not easy to differentiate normal processes from abnormal because aging appears as a normal process. Disease and aging have a possible genetic component. Aging is a time-dependent loss of structure and function that proceeds slowly and in such small increments that it appears to be the result of the accumulation of small, imperceptible injuries. It is not a time period outside of life expectancy. REF: p. 107 31. When a nurse observes muscle stiffening occurring within 6–14 hours after death, the nurse should document this finding as the presence of: a. livor mortis. b. gangrene. c. algor mortis. d. rigor mortis. ANS: D Rigor mortis occurs within 6 hours after death and is evidenced by muscle stiffening. Livor mortis is a purple discoloration. Gangrene refers to death of tissue and results from severe hypoxic injury and does not lead to stiffening. Algor mortis is postmortem reduction of body temperature. REF: pp. 109-110 32. When a nurse is checking a uN rinaR lysiI s, t h Ge fi B nd . in C g tM hat would alert the nurse to cellular injury is the presence of: a. slight glucose. b. excessive protein. c. blood. d. urea. ANS: B The presence of protein in the urine in significant amounts indicates cellular injury and altered cellular function. Neither glucose nor blood is normally present in the urine, but its presence is not indicative of altered cellular function. Urea is an expected substance in the kidney. REF: p. 99 33. An 86-year-old female patient has the wasting syndrome of aging, making her vulnerable to falls, functional decline, disease, and death. The nurse knows this patient is experiencing: a. frailty. b. sarcopenia. c. somatic death. d. cellular aging. ANS: A
  • 37. NURSINGTB.COM TO GET ALL CHAPTERS EMAIL ME AT>>>>> donc8246@gmail.com U S N Frailty leaves the individual vulnerable to falls, disease, and death. Sarcopenia is loss of muscle mass. It is associated with aging but is not as severe as frailty. Somatic death is death of the whole person. Cellular aging occurs at the cellular level and is characteristic of aging, including cell atrophy, decreased function, and loss of cells. REF: p. 109 34. Confirmation of somatic death is based on: a. presence of algor mortis. b. presence of livor mortis. c. complete cessation of respiration and circulation. d. change in skin color to pale yellow. ANS: C The most notable manifestations are complete cessation of respiration and circulation. Algor mortis is postmortem reduction of body temperature and is not confirmation of somatic death. Livor mortis is muscle stiffening and is not confirmation of somatic death. Change in skin color to pale yellow does occur, but complete cessation of respiration and circulation confirms somatic death. REF: p. 109 MULTIPLE RESPONSE 1. A patient has been reading on the Internet that light to moderate intake of alcohol is cardioprotective. When the patient asks the nurse what this means, the nurse should respond that the heart is protected byN whiR ch o I f thG e foB ll. ow Cing Mmechanisms? (select all that apply) a. Increased levels of high-density lipoprotein cholesterol b. Prevention of clot formation c. Reduction in platelet aggregation d. Decrease in blood pressure e. Increased collateral circulation f. Decreased folate absorption ANS: A, B, C, D The suggested mechanisms for cardioprotection by low to moderate alcohol intake include increased levels of high-density lipoprotein cholesterol (HDL-c), prevention of clot formation, reduction in platelet aggregation, and decrease in blood pressure. Increased collateral circulation is not a benefit of alcohol. Alcohol consumption can decrease folate absorption, which can lead to a nutritional deficient. REF: p. 91 2. After ingestion of lead, what organ systems should the nurse monitor because they are the most sensitive to the effects of lead? (select all that apply) a. Heart b. Lungs c. Liver d. Kidneys e. Brain
  • 38. NURSINGTB.COM TO GET ALL CHAPTERS EMAIL ME AT>>>>> donc8246@gmail.com f. Hematopoietic ANS: D, E, F The organ systems primarily affected by lead ingestion include the nervous system (brain), the hematopoietic system (tissues that produce blood cells), and the kidneys. The heart, liver, and lungs are not affected. REF: p. 89 NURSINGTB.COM
  • 39. NURSINGTB.COM TO GET ALL CHAPTERS EMAIL ME AT>>>>> donc8246@gmail.com Chapter 05: Fluids and Electrolytes, Acids and Bases MULTIPLE CHOICE 1. A nurse is reviewing lab reports. The nurse recalls blood plasma is located in which of the following fluid compartments? a. Intracellular fluid (ICF) b. Extracellular fluid (ECF) c. Interstitial fluid d. Intravascular fluid ANS: D Blood plasma is the intravascular fluid. ICF is fluid in the cells. ECF is all the fluid outside the cells. Interstitial fluid is fluid between the cells and outside the blood vessels. REF: p. 114 2. A 35-year-old male weighs 70 kg. Approximately how much of this weight is considered the total volume of body water? a. 5 L b. 10 L c. 28 L d. 42 L ANS: D The total volume of body waN teU r f R oS r a I 7 N 0G -kT gB pe . rs C oO n M is about 42 L or two thirds of 70 kg. REF: p. 114 3. While planning care for elderly individuals, the nurse remembers the elderly are at a higher risk for developing dehydration because they have: a. a higher total body water volume. b. decreased muscle mass. c. increased thirst. d. an increased tendency toward developing edema. ANS: B The elderly are at higher risk for dehydration due to a decrease in muscle mass. The elderly have a decrease in total body water and thirst. The increased tendency to develop edema is not related to dehydration. REF: p. 131, Geriatric Considerations 4. Which of the following patients should the nurse assess for decreased oncotic pressure in the capillaries? A patient with: a. a high-protein diet. b. liver failure. c. low blood pressure. d. low blood glucose.
  • 40. NURSINGTB.COM TO GET ALL CHAPTERS EMAIL ME AT>>>>> donc8246@gmail.com ANS: B Liver failure leads to lost or diminished plasma albumin production, and this contributes to decreased plasma oncotic pressure. A high-protein diet would provide albumin for the maintenance of oncotic pressure. Low blood pressure would lead to decreased hydrostatic pressure. Decreased glucose does not affect oncotic pressure. REF: p. 116 5. Water movement between the ICF and ECF compartments is determined by: a. osmotic forces. b. plasma oncotic pressure. c. antidiuretic hormone. d. buffer systems. ANS: A Osmotic forces determine water movement between the ECF and ICF compartments. Oncotic pressure pulls water at the end of the capillary, which makes it move between intra and extra as interstitial is considered extra. The antidiuretic hormone regulates water balance, which would make water move between the intra- and extracellular spaces. Buffer systems help regulate acid balance. REF: p. 115 6. An experiment was designed to test the effects of the Starling forces on fluid movement. Which of the following alterations would result in fluid moving into the interstitial space? a. Increased capillary oncotic pressure. b. Increased interstitial hydrostatic pressure. c. Decreased capillary hydrN osU taR tic Sp I re N sG suT re B . .COM d. Increased interstitial oncotic pressure. ANS: D Increased interstitial oncotic pressure would attract water from the capillary into the interstitial space. Increased capillary oncotic pressure would attract water from the interstitial space back into the capillary. Increased interstitial hydrostatic pressure would attract movement of water from the interstitial spaces into the capillary. Decreased capillary hydrostatic pressure would move water into the capillaries. REF: p. 115 7. When planning care for a dehydrated patient, the nurse remembers that the principle of water balance is closely related to the balance of: a. potassium. b. chloride. c. bicarbonate. d. sodium. ANS: D Because water follows the osmotic gradients established by changes in salt concentration, water balance is tied to sodium balance, not that of potassium, chloride, or bicarbonate. REF: pp. 116-117 | p. 118, Figure 5-5
  • 41. NURSINGTB.COM TO GET ALL CHAPTERS EMAIL ME AT>>>>> donc8246@gmail.com 8. A 70-year-old male with chronic renal failure presents with edema. Which of the following is the most likely cause of this condition? a. Increased capillary oncotic pressure. b. Decreased interstitial oncotic pressure. c. Increased capillary hydrostatic pressure. d. Increased interstitial hydrostatic pressure. ANS: C Increased capillary hydrostatic pressure would facilitate increased movement from the capillary to the interstitial space, leading to edema. Increased capillary (plasma) oncotic pressure attracts water from the interstitial space back into the capillary. Decreased interstitial oncotic pressure would keep water in the capillary. Increased interstitial hydrostatic pressure would facilitate increased water movement from the interstitial space into the capillary. REF: pp. 115-116 9. A 10-year-old male is brought to the emergency room (ER) incoherent and semiconscious. CT scan reveals that he is suffering from cerebral edema. This type of edema is referred to as: a. localized edema. b. generalized edema. c. pitting edema. d. lymphedema. ANS: A Cerebral edema is a form of localized edema. Generalized edema is manifested by a more uniform distribution of fluid in interstitial spaces. When pressure is applied, pitting edema results in an indention in the skin. Lymphedema is due to swelling in interstitial spaces, primarily in the extremities. NURSINGTB.COM REF: p. 116 10. A nurse is teaching the staff about antidiuretic hormone (ADH). Which information should the nurse include? Secretion of ADH is stimulated by: a. increased serum potassium. b. increased plasma osmolality. c. decreased renal blood flow. d. generalized edema. ANS: B ADH is secreted when plasma osmolality increases or circulating blood volume decreases and blood pressure drops. ADH is not secreted by an increase in potassium, a decrease in renal blood flow, or the presence of generalized edema. REF: p. 118 11. Which statement by the staff indicates teaching was successful concerning aldosterone? Secretion of aldosterone results in: a. decreased plasma osmolality. b. increased serum potassium levels. c. increased blood volume. d. localized edema.
  • 42. NURSINGTB.COM TO GET ALL CHAPTERS EMAIL ME AT>>>>> donc8246@gmail.com ANS: C Aldosterone promotes renal sodium and water reabsorption and excretion of potassium, thus increasing blood volume. Aldosterone secretion would cause increased plasma osmolality but it does not promote the development of localized edema; it affects blood volume. REF: p. 117 12. A 25-year-old male is diagnosed with a hormone-secreting tumor of the adrenal cortex. Which finding would the nurse expect to see in the lab results? a. Decreased blood volume b. Decreased blood K+ levels c. Increased urine Na+ levels d. Increased white blood cells ANS: B Aldosterone is secreted from the adrenal cortex. It promotes renal sodium and water reabsorption and excretion of potassium, leading to decreased potassium levels. Blood volume actually increases with aldosterone secretion. Aldosterone promotes sodium reabsorption, leading to normal or decreased Na+ levels, and is not associated with white blood cells. REF: p. 117 13. A patient has been searching on the Internet about natriuretic hormones. When the patient asks the nurse what these hormones do, how should the nurse respond? Natriuretic hormones affect the balance of: a. calcium. b. sodium. c. magnesium. d. potassium. ANS: B NURSINGTB.COM Natriuretic hormones are sometimes called a “third factor” in sodium regulation but have no influence on calcium, magnesium, or potassium balance. REF: p. 117 14. A 5-year-old male presents to the ER with delirium and sunken eyes. After diagnosing him with severe dehydration, the primary care provider orders fluid replacement. The nurse administers a hypertonic intravenous solution. Which of the following would be expected? a. Symptoms subside quickly b. Increased ICF volume c. Decreased ECF volume d. Intracellular dehydration ANS: D A hypertonic solution would cause fluid to move into the extracellular space, leading to intracellular dehydration. With this solution, his symptoms will not subside quickly because his cells will lose fluid. His intracellular volume will decrease and his extracellular volume will increase. REF: p. 120
  • 43. NURSINGTB.COM TO GET ALL CHAPTERS EMAIL ME AT>>>>> donc8246@gmail.com 15. Which of the following patients is the most at risk for developing hypernatremia? A patient with: a. vomiting. b. diuretic use. c. dehydration. d. hypoaldosteronism. ANS: C Dehydration leads to hypernatremia because an increase in sodium occurs with a net loss in water. Vomiting and diuretic use leads to hyponatremia. Hypoaldosteronism leads to hyponatremia. REF: p. 119 16. Which of the following conditions would cause the nurse to monitor for hyperkalemia? a. Excess aldosterone b. Acute acidosis c. Insulin usage d. Metabolic alkalosis ANS: B In acidosis, ECF hydrogen ions shift into the cells in exchange for ICF potassium and sodium; hyperkalemia and acidosis therefore often occur together. Acidosis does not cause excess aldosterone. Insulin would help treat hyperkalemia, not cause it. Alkalosis does not lead to hyperkalemia. REF: p. 124 NURSINGTB.COM 17. Which organ system should the nurse monitor when the patient has long-term potassium deficits? a. Central nervous system (CNS) b. Lungs c. Kidneys d. Gastrointestinal tract ANS: C Long-term potassium deficits lasting more than 1 month may damage renal tissue, with interstitial fibrosis and tubular atrophy. Long-term potassium deficits are not associated with damage to the CNS, GI tract, or lungs. REF: pp. 123-124 18. A 42-year-old female presents to her primary care provider reporting muscle weakness and cardiac abnormalities. Laboratory tests indicate that she is hypokalemic. Which of the following could be the cause of her condition? a. Respiratory acidosis b. Constipation c. Hypoglycemia d. Laxative abuse ANS: D
  • 44. NURSINGTB.COM TO GET ALL CHAPTERS EMAIL ME AT>>>>> donc8246@gmail.com Losses of potassium from body stores are usually caused by gastrointestinal and renal disorders. Diarrhea, intestinal drainage tubes or fistulae, and laxative abuse also result in hypokalemia. Acidosis is related to hyperkalemia, not hypokalemia. Constipation can occur with hypokalemia but does not cause it. Hypoglycemia is not related to muscle weakness. REF: p. 123 19. A 19-year-old male presents to his primary care provider reporting restlessness, muscle cramping, and diarrhea. Lab tests reveal that he is hyperkalemic. Which of the following could have caused his condition? a. Primary hyperaldosteronism b. Acidosis c. Insulin secretion d. Diuretic use ANS: B During acute acidosis, hydrogen ions accumulate in the ICF and potassium shifts out of the cell to the ECF, causing hyperkalemia. Primary hyperaldosteronism is associated with hypokalemia, not hyperkalemia. Insulin secretion helps reduce potassium levels in the cell; it does not cause hyperkalemia. Diuretics would cause hypokalemia, not hyperkalemia. REF: p. 122 20. A 60-year-old female is diagnosed with hyperkalemia. Which assessment finding should the nurse expect to observe? a. Weak pulse b. Excessive thirst c. Oliguria d. Constipation ANS: C NURSINGTB.COM Hyperkalemia is manifested by oliguria. Hypokalemia is manifested by a weak pulse; it is not caused by hyperkalemia. Hypokalemia is manifested by excessive thirst. Diarrhea, not constipation, is a manifestation of hyperkalemia. REF: p. 124, Table 5-6 21. Which of the following buffer pairs is considered the major plasma buffering system? a. Protein/fat b. Carbonic acid/bicarbonate c. Sodium/potassium d. Amylase/albumin ANS: B The carbonic acid/bicarbonate buffer pair operates in both the lung and the kidney and is a major extracellular buffer. Protein and fat are nutrients not related to the buffering system. Sodium and potassium are electrolytes for fluid and electrolyte balance, not the major plasma buffering system for acid-base balance. Amylase is a carbohydrate enzyme, and albumin is a protein; neither is a buffering system. REF: p. 125
  • 45. NURSINGTB.COM TO GET ALL CHAPTERS EMAIL ME AT>>>>> donc8246@gmail.com 22. A nurse recalls that regulation of acid-base balance through removal or retention of volatile acids is accomplished by the: a. buffer systems. b. skin. c. lungs. d. liver. ANS: C The volatile acid is carbonic acid (H2CO3), which readily dissociates into carbon dioxide (CO2) and water (H2O). The CO2 is then eliminated by the lungs. Buffer systems are throughout the body and operate in the extracellular and intracellular systems. Neither the liver nor the skin regulates acid-base balance. REF: p. 125 23. Which patient is most prone to metabolic alkalosis? A patient with: a. retention of metabolic acids. b. hypoaldosteronism. c. excessive loss of chloride (Cl). d. hyperventilation. ANS: C When acid loss is caused by vomiting, renal compensation is not very effective because loss of Cl stimulates renal retention of bicarbonate, leading to alkalosis. Retention of metabolic acids would lead to acidosis, not alkalosis. Hypoaldosteronism leads to hyponatremia and does not cause alkalosis. Hyperventilation leads to respiratory alkalosis, not metabolic alkalosis. REF: p. 127 NURSINGTB.COM 24. Which patient should the nurse assess for both hyperkalemia and metabolic acidosis? A patient diagnosed with: a. diabetes insipidus. b. pulmonary disorders. c. Cushing syndrome. d. renal failure. ANS: D Renal failure is associated with hyperkalemia and metabolic acidosis. Diabetes insipidus results in hypernatremia. Pulmonary disorders are a cause of respiratory acidosis or alkalosis but do not affect hyperkalemia. Cushing syndrome results in hypernatremia. REF: p. 124 | p. 127 25. For a patient experiencing metabolic acidosis, the body will compensate by: a. excreting H+ through the kidneys. b. hyperventilating. c. retaining CO2 in the lungs. d. secreting aldosterone. ANS: B
  • 46. NURSINGTB.COM TO GET ALL CHAPTERS EMAIL ME AT>>>>> donc8246@gmail.com It is the lungs hyperventilating that would compensate for metabolic acidosis by blowing off CO2, not any function associated with the kidneys. CO2 retention would increase the acidotic state. Aldosterone would conserve water but does not help compensate for acidosis. REF: p. 127 26. Which finding would support the diagnosis of respiratory acidosis? a. Vomiting b. Hyperventilation c. Pneumonia d. An increase in noncarbonic acids ANS: C Respiratory acidosis occurs with hypoventilation, and pneumonia leads to hypoventilation. Vomiting leads to loss of acids and then to alkalosis. Hyperventilation leads to respiratory alkalosis. Metabolic acidosis is caused by an increase in noncarbonic acids. REF: pp. 128-129 27. A 54-year-old male with a long history of smoking complains of excessive tiredness, shortness of breath, and overall ill feelings. Lab results reveal decreased pH, increased CO2, and normal bicarbonate ion. These findings help to confirm the diagnosis of: a. respiratory alkalosis. b. metabolic acidosis. c. respiratory acidosis. d. metabolic alkalosis. ANS: C NURSINGTB.COM A decreased pH indicates acidosis. With increased CO2, it is respiratory acidosis. The bicarbonate is normal, so it cannot be metabolic acidosis. REF: pp. 128-129 28. For a patient with respiratory acidosis, chronic compensation by the body will include: a. kidney excretion of H+ . b. kidney excretion of HCO3. c. prolonged exhalations to blow off CO2. d. protein buffering. ANS: A The kidneys excrete H+ to compensate for respiratory acidosis. The kidneys do not excrete HCO3 to compensate; this would increase acidosis. Prolonged exhalations would not be effective for compensation, especially in a chronic state. Protein buffering is intracellular and would not be effective enough to compensate for respiratory acidosis. REF: pp. 128-130 29. A 55-year-old female presents to her primary care provider and reports dizziness, confusion, and tingling in the extremities. Blood tests reveal an elevated pH, decreased PCO2, and slightly decreased HCO3. Which of the following is the most likely diagnosis? a. Respiratory alkalosis with renal compensation b. Respiratory acidosis with renal compensation
  • 47. NURSINGTB.COM TO GET ALL CHAPTERS EMAIL ME AT>>>>> donc8246@gmail.com c. Metabolic alkalosis with respiratory compensation d. Metabolic acidosis with respiratory compensation ANS: A With an elevated pH, the diagnosis must be alkalosis. Since the PCO2 is low, it is likely respiratory, with a slight decrease in HCO3 indicating renal compensation. REF: p. 130 MULTIPLE RESPONSE 1. A 60-year-old male with a 30-year history of smoking is diagnosed with a hormone-secreting lung tumor. Further testing indicates that the tumor secretes ADH. Which of the following assessment findings should the nurse expect? (select all that apply) a. Confusion b. Weakness c. Nausea d. Muscle twitching e. Increased reflexes ANS: A, B, C, D Secretion of ADH leads to water intoxication with symptoms of cerebral edema, including confusion, convulsions, weakness, nausea, and muscle twitching. Depressed reflexes are associated with water intoxication. REF: p. 119 | p. 121 NURSINGTB.COM
  • 48. NURSINGTB.COM TO GET ALL CHAPTERS EMAIL ME AT>>>>> donc8246@gmail.com Chapter 06: Innate Immunity: Inflammation and Wound Healing MULTIPLE CHOICE 1. A public health nurse is teaching the community about health promotion. Which information should the nurse include for innate immunity? Innate immunity is gained: a. following an illness. b. at birth. c. via injection of specific antibodies. d. in adulthood. ANS: B Innate immunity is present at birth. It is not dependent on illness or injection. REF: p. 134 2. Which statement indicates teaching was successful regarding collectins? Collectins are produced by the: a. kidneys. b. bowel. c. lungs. d. integument. ANS: C Collectins are produced by the lungs. Collectins are not associated with the kidneys, bowel, or integumentary system. NURSINGTB.COM REF: p. 135 3. A 20-year-old male received a knife wound to the arm during an altercation. Which of the following types of immunity was compromised? a. Innate immunity b. Inflammatory response c. Adaptive immunity d. Specific immunity ANS: A The epithelial cells of the skin are a part of innate immunity. The inflammatory response is not a type of immunity. Adaptive immunity is represented by the normal flora of the bowel. Specific immunity is a type of adaptive immunity and is not associated with a break in skin integrity. REF: pp. 134-135 4. Biochemical secretions that trap and kill microorganisms include: a. hormones. b. neurotransmitters. c. earwax. d. gastric acid.
  • 49. NURSINGTB.COM TO GET ALL CHAPTERS EMAIL ME AT>>>>> donc8246@gmail.com ANS: C Epithelial cells secrete several substances that protect against infection, including earwax. Hormones do not contain biochemical secretions that trap and kill microorganisms. Neurotransmitters carry important messages, and gastric acid helps break down food into its component parts, but neither contains biochemical secretions. REF: p. 135 5. A 25-year-old female presents to her primary care provider reporting vaginal discharge of a white, viscous, and foul-smelling substance. She reports that she has been taking antibiotics for the past 6 months. Which finding will the nurse most likely see on the microorganism report? a. Clostridium difficile overgrowth b. Decreased Lactobacillus c. Streptococcus overgrowth d. Decreased Candida albicans ANS: B Diminished colonization with Lactobacillus that occurs as a result of prolonged antibiotic treatment increases the risk for vaginal infections, such as vaginosis. Clostridium difficile and Candida albicans occur in the colon, not the vagina. Streptococcus overgrowth will occur in the mouth. REF: p. 136 6. When an aide asks the nurse about the purpose of the inflammatory process, how should the nurse respond? N R I G B.C M a. To provide specific respons U es tS owa N rd a T ntigens O . b. To lyse cell membranes of microorganisms. c. To prevent infection of the injured tissue. d. To create immunity against subsequent tissue injury. ANS: C One purpose of the inflammatory process is to prevent infection and further damage by contaminating microorganisms. Specific response toward antigens is a part of the complement system that assists in the inflammatory response, but not its purpose. Lysis of cell membranes is part of the process of phagocytosis, which removes foreign material. Immunity cannot be achieved against future tissue injury. REF: p. 137 7. A child fell off a swing and scraped his right knee. The injured area becomes painful. What else will the nurse observe upon assessment? a. Vasoconstriction at injured site b. Decreased RBC concentration at injured site c. Pale skin at injured site d. Edema at injured site ANS: D Increased vascular permeability and leakage of fluid out of the vessel cause edema at the site of injury. Vasodilation occurs, bringing increased RBCs to the site and causing redness.
  • 50. NURSINGTB.COM TO GET ALL CHAPTERS EMAIL ME AT>>>>> donc8246@gmail.com REF: p. 137 8. A nurse recalls the mast cell, a major activator of inflammation, initiates the inflammatory response through the process of: a. chemotaxis. b. endocytosis. c. degranulation. d. opsonization. ANS: C Degranulation of mast cells is a major cellular component of inflammation. Chemotaxis is the process of white cell migration. Endocytosis and opsonization are parts of phagocytosis but are not factors in mast cell response. REF: pp. 144-145 9. Which of the following individuals would be at greatest risk for an opportunistic infection? a. 18-year-old with diabetes b. 70-year-old with congestive heart failure c. 24-year-old who is immunocompromised d. 30-year-old with pneumonia ANS: C Opportunistic microorganisms can cause disease if the individual’s defenses are compromised. Diabetes, congestive heart failure, and pneumonia are not associated with immunocompromised disorders. NURSINGTB.COM REF: p. 137 10. The directional migration of leukocytes along a chemical gradient is termed: a. chemotaxis. b. endocytosis. c. margination. d. diapedesis. ANS: A Chemotaxis is the process by which leukocytes undergo directed migration. Endocytosis is a form of engulfment and a part of phagocytosis. Margination occurs when leukocytes adhere to endothelial cells in the walls of vessels. Diapedesis is the emigration of the cells through cell junctions that have loosened in response to inflammatory mediators. REF: p. 147 11. A 20-year-old male shoots his hand with a nail gun while replacing roofing shingles. Which of the following cell types would be the first to aid in killing bacteria to prevent infection in his hand? a. Eosinophils b. Neutrophils c. Leukotrienes d. Monocytes